Nervous System/Neurology Flashcards

1
Q

A 78-year old woman presents to the emergency department with acute onset of left-sided ptosis and horizontal diplopia. On examination, there is dilated nonreactive pupil on left side and normal indirect light reflex of the left side. Which one of the following is the most likely diagnosis?

A. Optic nerve palsy.
B. Third cranial nerve palsy.
C. Fourth cranial nerve palsy.
D. Sixth cranial nerve palsy.
E. Seventh cranial nerve palsy.

A

B. Third cranial nerve palsy

Patients with acute acquired third nerve palsy usually experience sudden double vision (binocular horizontal, vertical, or oblique diplopia) and a droopy eyelid.

  • Ptosis: Droopy eyelid
  • Dilated pupil (midriasis)
  • Paralysis of eye movements: Can’t move the eye inward (adduction), upward (elevation), or downward (depression)
  • Eye position: The affected eye rests in an “abducted, slightly depressed, and intorted” position (down and out).

Note: Pupillary reflex is usually intact in third nerve palsies due to ischemia (e.g., diabetes, midbrain infarcts).

  • Leads to monocular visual loss (affects one eye)
  • Symptoms: Impaired pupil light reflex, central vision loss (scotoma), color blindness (dyschromatopsia)
  • Causes binocular vertical diplopia and torsional diplopia
  • Symptoms: Weakness in the superior oblique muscle, resulting in an upward deviation of the affected eye (ipsilateral hypertropia)
  • Causes binocular horizontal diplopia that worsens when looking toward the side of the affected lateral rectus muscle
  • Symptoms: Weakness in abduction, with the affected eye often deviated inward (medial deviation)
  • Symptoms: Inability to close the eye, facial droop, inability to form forehead wrinkles, changes in taste sensation on the front two-thirds of the tongue
  • Peripheral Lesion: Affects one entire side of the face (upper and lower)
  • Central Lesion: Only the lower part of the contralateral face is affected; the forehead is spared because it receives input from both hemispheres of the brain.

Note: With an upper motor neuron lesion (central seven), the contralateral lower face is affected while the forehead remains unaffected. With a peripheral lesion, there is complete ipsilateral facial dysfunction (upper and lower face).

Patients with acute acquired third nerve palsy usually complain of the sudden onset of binocular horizontal, vertical, or oblique diplopia as well as a droopy eyelid.

On examination, patients with complete non-pupil-sparing third nerve palsy have ptosis, a midriatic (dilated) pupil, and paralysis of adduction, elevation, and depression of the eye on the affected side. The eye rests in a position of abduction, slight depression, and intorsion (down and out).

NOTE - pupillary reflex is often intact and unaffected in third nerve palsies due to ischemia such as in diabetes or midbrain infarcts.

Left third nerve palsy. The affected eye rests in ‘out and down’ position. Mild ptosis is noted. (see photo below)

Option A: An optic nerve lesion leads to monocular visual loss. Other features of optic neuropathy include:
-Impaired pupil light reflex (the afferent limb)
-Central vision loss (scotoma) on visual field testing
-Dyschromatopsia (color blindness) often out of proportion to acuity loss

Option C: Fourth nerve palsy may result in binocular (with both eyes open) vertical diplopia and subjective torsional diplopia. It causes paralysis of superior oblique muscle. On examination, an ipsilateral hypertropia is present (the involved eye is deviated upward) because the action of the superior oblique muscle (moving the eye down and inwards) is weak.

Option D: Patients with sixth nerve palsies typically complain of binocular (both eyes open) horizontal diplopia that worsens with gaze toward the paretic lateral rectus muscle. Lateral rectus muscle paralysis results in weakness of abduction of the affected eye with the affected eye often resting in medial deviation.

Option E: Seventh nerve palsy results in inability to close eye, facial droop, inability to form wrinkles on forehead and change in taste sensations in anterior two-thirds of the tongue. When one entire side of the face is weak, the lesion is usually peripheral. With a central lesion (such as stroke in one cerebral hemisphere), the forehead muscles are often spared because the part of the facial nerve nucleus supplying innervation to the forehead gets input from motor neurons of both cerebral hemispheres. The portion of the facial nerve nucleus innervating the lower face does not have the same bilateral input; its input is predominantly from the contralateral cortex.

NOTE - With upper motor neuron lesion of the facial nerve (also called central seven), the contralateral lower parts of the face are affected. The contralateral forehead remains unaffected. With peripheral lesions, complete ipsilateral dysfunction of the face muscles (upper and lower) occurs and the forehead is not spared.

Third Nerve Palsy
How well did you know this?
1
Not at all
2
3
4
5
Perfectly
2
Q

Which one of the following conditions causes ptosis and dilated pupil?

A. Myasthenia gravis.
B. Mitochondrial myopathy.
C. Horner’ssyndrome.
D. Third cranial nerve palsy.
E. Sixth cranial nerve palsy.

A

D. Third cranial nerve palsy

Third cranial nerve (oculomotor) palsy can cause:

  • Ptosis (droopy eyelid)
  • Deviation of the eye downwards and outwards
  • Dilated pupils
  • Sluggish or absent light reflex

In cases of ischemic third nerve palsy (e.g., due to diabetes or midbrain infarcts), the pupils usually remain unaffected.

  • Option A: Myasthenia Gravis
    • Causes ptosis and double vision (diplopia)
    • No pupil abnormalities
  • Option B: Mitochondrial Myopathy
    • Causes progressive eye muscle weakness and limb weakness with exercise
    • No pupil involvement
  • Option C: Horner’s Syndrome
    • Causes ptosis, miosis (constricted pupils), and loss of sweating on one side
    • No diplopia
  • Option E: Sixth Cranial Nerve Palsy
    • Causes impaired lateral eye movement
    • No pupil abnormalities

Third cranial nerve (oculomotor) palsy can cause ptosis, deviation of the eye downwards and outwards, dilated pupils and sluggish or absent light reflex. In the third nerve paralysis due to ischemia (e.g. diabetic palsy or midbrain infarcts) pupils usually remain intact.

Option A: Myasthenia gravis can cause ptosis and diplopia with no pupil abnormality.

Option B: Mitochondrial myopathy is associated with progressive ophthalmoplegia and limb weakness induced by exercise. It does not involve pupils.

Option C: Horner’s syndrome is characterized by ptosis, miosis (constricted pupils) and ipsilateral loss of sweating. There is no diplopia.

Option E: Sixth cranial nerve palsy presents with impaired lateral eye movement without affecting the pupils.

How well did you know this?
1
Not at all
2
3
4
5
Perfectly
3
Q

An 79-year-old diabetic man presents with isolated third nerve palsy. Which one of the following findings, if present, suggests diabetes as the etiology?

A. Normal pupillary reflex.
B. Normal extra ocular movements.
C. Enophthalmos.
D. Involvement of the superior oblique muscle.
E. Absence of ptosis.

A

A. Normal pupillary reflex

  • Most common causes: Diabetes mellitus, hypertension, and advanced age
  • Symptoms:
    • Ptosis (droopy eyelid)
    • Eye deviated down and out
    • Paralysis of superior rectus, inferior rectus, medial rectus, and inferior oblique muscles
    • Normal pupillary reflex (pupils unaffected)

Note: If the pupil is involved, consider aneurysmal compression until proven otherwise.

  • Option A: Normal Pupillary Reflex
    • Common in ischemic third nerve palsy (e.g., diabetes)
  • Option B: Paralysis of Eye Muscles
    • All cases of third nerve palsy affect the superior rectus, inferior rectus, medial rectus, and inferior oblique muscles
  • Option C: Enophthalmos
    • Seen with orbital floor fractures from trauma, not due to nerve paralysis
  • Option D: Superior Oblique Muscle
    • Supplied by the fourth cranial nerve, not affected in third nerve palsy
  • Option E: Ptosis
    • Present in all cases of third nerve palsy, regardless of cause

Ischaemic third nerve palsies comprise the vast majority of third nerve palsies in adults. Diabetes mellitus is the most common cause, follwoed by hypertension and advanced age.

Isolated third nerve palsy with a normal pupillary reflex and completely paralysed extra-ocular muscles innervated by this nerve (superior rectus, inferior rectus, medial rectus and inferior oblique) are the most common presenting features in diabetic third nerve palsy.

NOTE - Pupil involvement in third nerve palsy should be assumed to be due to aneurysmal compression until proven otherwise.

Option B: With third nerve palsy the function of superior rectus, inferior rectus, medial rectus and inferior oblique muscles are impaired regardless of the etiology.

Option C: Enophthalmos can be seen in palsies occurred in trauma setting and is caused by orbital floor fractures not the paralysis of the nerve.

Option D: Superior oblique muscle is supplied by the fourth cranial nerve and its function remains intact in diabetic third nerve palsy.

Option E: Ptosis can be seen in all cases of third nerve palsy regardless of the underlying etiology.

How well did you know this?
1
Not at all
2
3
4
5
Perfectly
4
Q

A 32-year-old man presents with complaint of not being able to use his right arm as usual. On examination, shoulder abduction and elbow extension on the right side are diminished. Moreover, there is loss of sensation over the right deltoid area. Which one of the following could be the cause of his problem?

A. C5 nerve palsy.
B. Injury to the brachial plexus.
C. Carpal tunnel syndrome.
D. C7 nerve root injury.
E. C6-C7 nerve root injury.

A

B. Injury to the brachial plexus

The brachial plexus is crucial for the movement and sensation of the upper limb. It consists of nerve roots from C5 to T1, forming various nerves that innervate different muscles and areas.

  • Diminished shoulder abduction and sensory loss over deltoid area: Suggests involvement of C5 and C6 nerve roots.
  • Decreased elbow extension: Indicates radial nerve involvement (C5, C6, C7, C8, T1).
  • Option A: C5 Nerve Palsy
    • Can explain shoulder abduction weakness and deltoid sensory loss.
    • Does not account for decreased elbow extension, which is radial nerve territory.
  • Option C: Carpal Tunnel Syndrome
    • Due to median nerve entrapment, affecting hand and wrist.
    • Does not affect shoulder abduction, deltoid sensation, or elbow extension.
  • Options D and E: C7 Nerve Injury
    • Justifies weakened elbow extension (triceps weakness).
    • Does not explain shoulder abduction weakness or deltoid sensory loss (supplied by axillary nerve, involving C5 and C6 roots).
  • Likely Diagnosis: Injury to the brachial plexus involving multiple nerve roots (C5, C6, and possibly others).
  • Clinical Features: Reflect deficits in shoulder abduction, deltoid sensation, and elbow extension, matching the distribution of affected nerves.

To answer this question, one should know the anatomy and distribution of brachial plexus which innervates almost all of the movements and sensation of the upper limb.

The following schematic diagram pictures the structure of the brachial plexus, the rami, trunks, divisions, cords and terminal branches and shows which spinal nerve roots contribute to formation of the nerves: - see photo below -

Diminished shoulder abduction and sensory loss over deltoid area suggest that C5 and C6 are affected. With decreased elbow extension, radial nerve (C5, C6, C7, C8 and T1) must be affected. With possible involvement of several roots, the injury to the brachial plexus is the most likely underlying cause to this man’s presentation.

Option A: C5 nerve palsy, can justify, to some extent, the loss of sensation of deltoid and shoulder abduction, but not the elbow extension which is innervated by the radial nerve.

Option C: Carpal tunnel syndrome is caused by entrapment of the median nerve in the flexor retinaculum. It does not affect elbow extension, shoulder abduction and sensory loss over the deltoid area.

Option D and E: C7 nerve injury can justify the weakened elbow extension but not the shoulder abduction and sensory loss over the deltoid area which is supplied by axillary nerve comprised of C5 and C6 nerve roots. This is through about C6-C7 nerve root deficits.

How well did you know this?
1
Not at all
2
3
4
5
Perfectly
5
Q

A 26-year-old woman sustains a closed head injury in a motor vehicle crash. She is unconscious on arrival at the emergency department. A head CT scan excluded skull fractures and hematomas. She regains consciousness after 25 minutes and now is complaining of double vision when she looks to the right. Vision ahead is normal, as is when either eye is covered. Which one of the following cranial nerves is more likely to have been injured?

A. Right third cranial nerve.
B. Right fourth cranial nerve.
C. Left fourth cranial nerve.
D. Left sixth cranial nerve.
E. Right sixth cranial nerve.

A

E. Right sixth cranial nerve

The sixth cranial nerve innervates the lateral rectus muscles; therefore, damage to the nerve produces binocular (when looked with both eyes) horizontal diplopia when the patient looks to the side of the injured nerve. This occurs because the lateral rectus muscle on the affected side fails to turn the eye to the affected side laterally, while the functioning medial rectus muscle of the contralateral eye turns it medially. The result of uncoordinated move of the two eyes produces two distinct picture on the retina and causes horizontal diplopia.

The sixth cranial nerve has a long course from the lower pons and reaches the lateral rectus muscle after emerging through the superior orbital fissure. The long course of this nerve makes it susceptible to injuries.

Option A: Typical presentation of the third cranial nerve palsy is ipsilateral ‘down and out’ gaze and fixed dilated pupil. With ischemic palsy of this nerve (e.g. diabetic palsy, midbrain stroke) the pupil reflex usually remains intact.

Option B and C: The fourth cranial nerve supplies the superior oblique muscle. The function of this muscle gives eye intorsion which is moving of the eye inwards and downwards. Damage to this nerve will lead to binocular diplopia on downward gaze.

Option D: With the left sixth cranial nerve injury, there will be binocular diplopia on looking to the left side (the affected side).

The sixth cranial nerve, also known as the abducens nerve, innervates the lateral rectus muscle of the eye. Damage to this nerve results in a specific set of symptoms:

  • Function: It controls the lateral movement of the eye towards the side of the injury.
  • Symptom: When the sixth cranial nerve is damaged on one side, the affected eye cannot move laterally towards the side of the injury. This leads to binocular horizontal diplopia (double vision when using both eyes together), particularly noticeable when looking towards the affected side.
  • Mechanism: The intact medial rectus muscle of the contralateral eye pulls the eye inward (medially), while the paralyzed lateral rectus muscle on the affected side fails to move the eye outward (laterally). This mismatch causes each eye to perceive a different image, resulting in horizontal diplopia.
  • Course and Susceptibility: The abducens nerve has a long course from the lower pons in the brainstem to the lateral rectus muscle, making it vulnerable to injuries along its path.

Comparison with Other Nerve Palsies:
- Third Cranial Nerve (Option A): Controls multiple eye muscles including the medial rectus. Damage leads to “down and out” gaze and a dilated pupil, often seen in conditions like diabetic neuropathy.
- Fourth Cranial Nerve (Options B and C): Supplies the superior oblique muscle, causing vertical diplopia specifically on downward gaze when damaged.
- Left Sixth Cranial Nerve (Option D): Specifically causes horizontal diplopia when looking towards the left side, reflecting the side of the nerve injury.

Understanding these specific symptoms and mechanisms helps in distinguishing between different cranial nerve palsies based on clinical presentations.

Extraocular Muscle Innervation
How well did you know this?
1
Not at all
2
3
4
5
Perfectly
6
Q

Aaron, 73 years old, is a diabetic patient of yours who has presented to the clinic with sudden onset of horizontal diplopia, better when he looks at a near object and worse when looking at distance. On examination, the left eye is deviated towards medial side. Which one of the following is the most likely diagnosis?
A. Left sixth cranial nerve palsy.
B. Right Sixth cranial nerve palsy.
C. Right third cranial nerve palsy.
D. Left third cranial nerve palsy.
E. Left fourth cranial nerve palsy.

A

A. Left sixth cranial nerve palsy

The sixth cranial nerve (CN-VI) palsy results in isolated weakness of abduction of the affected eye and horizontal binocular diplopia. On examination, there is an esotropia (inward deviation) that is worsened with gaze into the field of the affected lateral rectus muscle. Abduction is commonly limited on the side of the lesion. Poorly controlled diabetes is a predisposing factor.

Option B: In the right sixth cranial nerve palsy the right eye will be in an abnormal position (medially deviated), and the patient is diplopic on looking laterally to the right side.

Option C and D: The third cranial nerve supplies the levator palpebrae muscle of the eyelid and 4 extraocular muscles: the medial rectus, superior rectus, inferior rectus, and inferior oblique. These muscles adduct, depress, and elevate the eye. Patients with acute acquired third nerve palsy usually complain of the sudden onset of binocular horizontal, vertical, or oblique diplopia and a droopy eyelid. Pupil reflex remains intact in ischemic palsies (e.g. due to diabetes or midbrain infarcts).

Option E: A person with fourth nerve palsy may complain of binocular (both eyes open) vertical diplopia and/or subjective tilting of objects (torsional diplopia). The affected eye is usually extorted because the superior oblique muscle is responsible for intorsion of the eye. Objects viewed in primary position, especially in down-gaze may appear double when going down a flight of stairs so that the patient does not know which step to take first.

How well did you know this?
1
Not at all
2
3
4
5
Perfectly
7
Q

An 11-year-old boy is brought to your practice with progressive difficulty climbing stairs, walking, and running for the past few days. On examination, bilateral lower limb weakness is noted. All lower limb deep tendon reflexes (DTRs) are lost. The sensation however is intact. Which one of the following tests is most likely to confirm the diagnosis?

A. CSF analysis.
B. Forced vital capacity.
C. MRI.
D. Nerve conduction studies.
E. X-ray.

A

D. Nerve conduction studies

Asymmetrical lower limb weakness, areflexia and intact sensation, make acute inflammatory polyradiculoneuropathy (Guillain-Barre syndrome) the most likely diagnosis. Guillain-Barre syndrome (GBS) often presents acutely with rapidly progressive widespread weakness and sensory disturbances such as pain, often in the presence of a normal sensory examination. Weakness usually begins peripherally. Tendon reflexes are often lost or impaired. Some cases may have a preceding infectious disease such gastroenteritis with Campylobacter.

GBS occurs world-wide with an overall incidence of 1-2 per 100,000 per year. While all age groups can be affected, the incidence increases by approximately 20% with every 10-year increase in the age beyond the first decade of life.

The initial diagnosis of GBS is based on the clinical picture. The cardinal clinical features of GBS are progressive, mostly symmetric muscle weakness and absent or depressed deep tendon reflexes. The weakness can vary from mild difficulty walking to nearly complete paralysis of all extremities, facial, respiratory, and bulbar muscles.

The diagnosis of Guillain-Barre syndrome (GBS) is confirmed if cerebrospinal fluid (CSF) and clinical neurophysiology studies show the typical abnormalities. Therefore, lumbar puncture and clinical neurophysiology studies are performed in all patients with suspected GBS. Of these two, however, nerve conduction studies (NC) and needle electromyography (EMG) are more accurate. They are used not only for confirmation of diagnosis, but also for providing information regarding prognosis.

The typical CSF finding, known as albuminocytologic dissociation, starts approximately 48 hours after symptoms onset and is present in 50-66% of the patients in the first week, and over in 75% in the third week. Nerve conduction studies show a typical demyelinating pattern. NCV changes often lag behind those of CSF.

Patients with GBS should always be managed in an inpatient setting. Vital capacity should be monitored 4-hourly, using forced vital capacity (FVC), and if it falls to less than 20 ml/kg or is declining rapidly, the patient should be transferred to an intensive care unit. FVC monitoring is not diagnostic though. Cardiac monitoring is also recommended for such patients due to significant risk of cardiac arrhythmia.

Imaging studies (MRI, X-rays, etc.) are not of diagnostic value.

Guillain-Barre syndrome (GBS) is characterized by acute inflammatory polyradiculoneuropathy, typically presenting with rapidly progressive weakness in the limbs, areflexia (loss of reflexes), and intact sensation. It often follows an infectious illness like gastroenteritis caused by Campylobacter.

Diagnosis of GBS is primarily clinical, confirmed by findings such as progressive symmetric muscle weakness and absent or reduced deep tendon reflexes. Additional tests include cerebrospinal fluid (CSF) analysis and nerve conduction studies (NCV) with electromyography (EMG). NCV and EMG are particularly useful for confirming the diagnosis and assessing prognosis, showing characteristic demyelinating patterns.

CSF analysis may reveal albuminocytologic dissociation, a hallmark of GBS, which typically appears within 48 hours of symptom onset. This supports the clinical diagnosis.

Management of GBS involves close monitoring of vital capacity, as respiratory function can deteriorate rapidly. Patients are usually managed in an inpatient setting, with intensive care support if respiratory function declines significantly. Cardiac monitoring is also recommended due to the risk of arrhythmias associated with GBS.

Imaging studies like MRI or X-rays are generally not helpful for diagnosing GBS.

Understanding these clinical and diagnostic features is crucial for prompt recognition and appropriate management of Guillain-Barre syndrome.

How well did you know this?
1
Not at all
2
3
4
5
Perfectly
8
Q

A 22-year-old college student presents to your practice, complaining of recurrent episodes of drowsiness and feeling detached from the surrounding environment, mostly when she is in a shopping mall. Which one of the following would be the next best step in management?

A. CT scan of the head.
B. EEG.
C. Ask a witness about the episodes.
D. Polysomnography.
E. Advise relaxation techniques.

A

C. Ask a witness about the episodes

This patient has presented with episodes of transient alteration or absence of consciousness. Evaluation of such episodes is critical to diagnose epileptic seizures, parasomnias, organic enecphalopathies, psychogenic nonepileptic seizure, or sometimes a false account of the events for malingering.

A witness is a valuable source of information regarding the episode(s), and should be asked about the event if possible.

The following are examples of questions to ask:
- What did they see?
- Did the person become pale or confused before or during the attack?
- What does she/he looked like during the attack?
- How long does the episode last?
- How long did it take before consciousness returned to normal?

A thorough history in conjunction with an account of the event from a witness (if present) is always the very first step in stable patients who present with such episodes.

Option A, B and D: Investigations such as CT scan, EEG, or polysomnography can be selected for further evaluation of possible diagnoses based on history and physical exam findings.

Option E: Relaxation techniques are indicated if anxiety is found to be the underlying etiology.

Approach to the patient with transient alteration of consciousness
Medscape - 10 Steps Before You Refer for Syncope

How well did you know this?
1
Not at all
2
3
4
5
Perfectly
9
Q

One week after a knee surgery, a 24-year-old man presents with complaints of numbness and paresthesia of the right leg. On examination, there is foot drop and weakness of dorsiflexion and eversion of the right ankle. Sensation over the outer aspect of the right leg is also lost. Right ankle jerk is intact. Which one of the following nerves is most likely to have caused such presentation?

A. L4 nerve root.
B. L5 nerve root.
C. Common peroneal nerve.
D. Tibial nerve.
E. Sciatic nerve.

A

C. Common peroneal nerve

The clinical picture suggests damage to the common peroneal nerve as an adverse outcome of the knee surgery. Foot drop is the result of weakness of the ankle dorsiflexors (extensors) innervated by the deep peroneal nerve (with the exception of peroneous tertius, a weak ankle dorsiflexor located in the lateral compartment of the leg, and supplied by the superficial peroneal nerve). Sensation over the outer aspect of the leg is supplied by the common peroneal nerve and its superficial branch. These together make the common peroneal nerve injury the most likely explanation.
After branching off the sciatic nerve, the common peroneal nerve runs down laterally, wraps around the fibular head and enters the leg. The nerve is superficial in this region and susceptible to injuries during knee surgeries, with compression or in trauma.

Option D: Tibial nerve damage causes impaired ankle jerk, weak or absent plantar flexion and weak ankle inversion. Sensory impairment due to tibial nerve injury affects the back the leg and most parts of the sole of the foot.

Option E: Sciatic trunk (nerve) is above the knee, and unlikely to have been affected by the knee surgery. Moreover, sciatic damage affects the ankle and foot movements globally, and gives a clinical picture consistent with injuries of both common peroneal and tibial nerves at the same time.

L5 radiculopathy can mimic CPN injury**

How well did you know this?
1
Not at all
2
3
4
5
Perfectly
10
Q

A 45-year-old man with history of type II diabetes mellitus comes to the emergency department with diplopia and ptosis of his left eye. On examination, the left eye is deviated slightly out and down in straight-ahead gaze. Upward gaze is impaired. Pupils have normal reaction to light. Which one of the following would be the most likely underlying cause of this presentation?

A. Aneurysm of the posterior communicating artery.
B. Tumor.
C. TB meningitis.
D. Diabetes mellitus.
E. Trans-tentorial brain herniation.

A

D. Diabetes mellitus

The clinical features described are consistent with left oculomotor (3rd cranial nerve [CN-3]) palsy with sparing of the pupil.

The most common causes of third nerve palsy include:
1. Ischemia - Ischemic third nerve palsies, are the most common etiologic subset of third nerve palsies in adults. The pathogenesis is hypothesized to be microvascular.
2. Intracranial aneurysms - The most dreaded cause of a third nerve palsy is compression by an enlarging intracranial aneurysm. The most common site of an aneurysm causing a third nerve palsy is the posterior communicating artery; however, aneurysms involving the internal carotid artery and basilar artery are reported to produce third nerve palsies as well. In the setting of an acute third nerve palsy, the aneurysm is believed to be acutely enlarging and therefore at risk of imminent rupture. In this setting, subarachnoid hemorrhage can occur within hours or days of initial presentation of a third nerve palsy.
3. Trauma - Traumatic third nerve palsy usually arises only from severe blows to the head assoiciated with skull fracture and/or loss of consciousness. Thus, a third nerve palsy associated with mild head trauma should prompt evaluation for an associated pathology.
4. Migraine - Ophthalmoplegic migraine is a condition affecting children and young adults, and most commonly involves the third cranial nerve, sometimes with permanent deficits.
5. Other causes - Infections, tumors, vasculitis, and herniation are other less common causes of third nerve palsy.

NOTE - The most common causes of the oculomotor nerve palsy sparing the pupil reaction is ischemia of the nerve as a result of vascular compromise due to diabetes mellitus.

Third nerve involvement associated with impaired pupil light reaction can be caused with any of the following:
* Aneurysm of the posterior communicating artery – it can rarely spare the pupil light reaction
* Transtentorial brain herniation
* TB meningitis

3rd nerve palsy - when is light/pupillary reflexes impaired or normal?

How well did you know this?
1
Not at all
2
3
4
5
Perfectly
11
Q

A 34-year-old woman presents to the Emergency Department with complaint of sudden-onset severe unilateral headache on the left side. The pain is felt behind the orbit and is associated with nasal stiffness and conjunctival injection and lacrimation. She has the history of previous similar headaches. Which one of the following is the the treatment of choice to prevent further attacks?

A. Verapamil.
B. Oxygen100%.
C. Propranolol.
D. Sumatriptan.
E. Paracetamol.

A

A. Verapamil

The scenario describes a classic case of cluster headache, also known as migrainous neuralgia. Cluster headache is characterized by attacks of severe orbital, supraorbital, or temporal pain, accompanied by autonomic phenomena. The typical attacks may strike up to eight times a day and are relatively short-lived. Cluster headache is strictly unilateral, and the symptoms remain on the same side of the head during a single cluster attack. However, the symptoms can switch to the other side during a different cluster attack (side shift) in approximately 15%.

The pain is very severe and as opposed to migraine pain, patients with cluster headaches are restless and prefer to pace about or sit and rock back and forth. The attacks of cluster headache can be so severe and vicious that patients may commit suicide if the disease is not diagnosed or treated. Unlike migraine headache, cluster headache is NOT associated with nausea and vomiting.

Autonomic symptoms include ptosis, miosis, lacrimation, conjunctival injection, rhinorrhea, and nasal congestion. These symptoms are due to both sympathetic impairment and parasympathetic hyperactivity. In some patients, the signs of sympathetic paralysis (miosis and ptosis) persists indefinitely and increases during attacks.

The very first step in management of a cluster headache attack is oxygen 100% through a tightly-sealed face mask for 15 minutes. This treatment alleviates the headache in most patients. For those unresponsive to oxygen, the following could be tried with indefinite response:
-Sumatriptan (intramuscularly or intranasally)
-Dihydroergoramine (intramuscularly)
-Lidocaine (intranasally)

NOTE - Nasal congestion may render the intranasal route ineffective.

After cluster headache is diagnosed, preventive treatment should be started with verapamil (sustained released) as the first-line medication.
Methysergide, corticosteroids, or lithium have been used as alternatives, but not as first-line. Methysergide is associated with the serious complication of retroperitoneal, cardiac and/or pleural fibrosis.

How well did you know this?
1
Not at all
2
3
4
5
Perfectly
12
Q

A 65-year-old woman presents to the emergency department with complaint of difficulty walking for the past 24 hours. She also mentions that she has been experiencing progressive pins and needles in hands and feet for the past few days. On examination, she has a blood pressure of 130/80 mmHg, pulse rate of 90 bpm, respiratory rate of 14 breaths per minute and temperature of 37.5°C. On neurological examination, she is found to have brisk deep tendon reflexes (DTRs). There is an extensor plantar response. There is also a decrease in sensing pain, temperature, vibration and touch. Romberg test is positive. There is no visual sign or symptom. Which one of the following is the investigation of choice to reach a diagnosis?

A. Lumbar puncture.
B. Nerve conduction studies.
C. Serum vitamin B12 level.
D. Serum creatine kinase(CK).
E. MRI of the brain.

A

C. Serum vitamin B12 level

The simultaneous presence of clinical findings attributable to peripheral neuropathy and myelopathy are strong indicators of subacute combined spinal cord degeneration that occurs with vitamin B12 deficiency.

Patients with the condition complain of distal paresthesia and weakness of the extremities, followed by spastic paresis and ataxia (represented by a positive Romberg test here). On physical exam, there is a combined deficit of vibration and proprioception with pyramidal signs (plantar extension and hyperreflexia). Other sensory modalities can be affected as well.

Neuropsychiatric manifestations, such as recent memory loss with reduced attention span and otherwise normal cognition, depression, hypomania, paranoid psychosis with auditory or visual hallucinations (megaloblastic madness), violent behavior, personality changes, blunted affect, and emotional lability are possible presentations of the disease reported in some of the patients.

To establish the diagnosis, measuring the serum vitamin B12 level is the most important investigation. Treatment replacing vitamin B12.

(Extensor Plantar Response = positive Babinski reflex)

Vitamin B-12 Associated Neurological Diseases

How well did you know this?
1
Not at all
2
3
4
5
Perfectly
13
Q

Which one of the following statements is incorrect regarding Guillain-Barre syndrome?

A. Intravenous immunoglobulin (IVIG) Is the treatment of choice.
B. Plasma exchange is the first-line treatment.
C. Neuropathic pain is commonly seen during the illness.
D. Steroids have no role in treatment.
E. Neuropathic pain is responsive to tricyclic antidepressants.

A

B. Plasma exchange is the first-line treatment

Acute inflammatory polyradiculoneuropathy (Guillain-Barre syndrome) often presents acutely, with rapidly progressive widespread weakness and sensory disturbance, often in the absence of sensory signs that usually begins peripherally. Tendon reflexes are often lost or impaired. Many different clinical presentations are seen including Miller Fisher variant, manifested with extraocular paresis, ataxia and arefelxia. There is also an uncommon variant with bulbar presentation.

Some cases have an infectious trigger including Campylobacter infection.

Confirmatory tests include a cerebrospinal fluid (CSF) examination with typical finding of elevated CSF protein level with an absent or minimal cellular response and, a little while later in the course of the disease, slowing of nerve conduction on nerve conduction studies.

Patient should always be managed in an inpatient setting. Vital capacity should be monitored 4-hourly and if it falls to less than 20 ml/kg or is declining rapidly, the patient should be transferred to an intensive care unit. Cardiac monitoring is recommended for such patients due to significant risk of cardiac arrhythmia.

Intravenous immunoglobulin (IVIG) and plasma exchange are both proven treatments with similar efficacy. Both can be used as treatment options; however, plasma exchange is logistically more difficult and associated with greater potential complications making it less desirable as first-line treatment.

Option A: IVIG can be more readily available and can be used as the treatment of choice in most centers.

Option C and E: Neuropathic pain is a frequent complaint in patients with Guillain-Barre syndrome. The management is the same as for any neuropathic pain syndrome – adjuvant analgesia with a tricyclic antidepressant or antiepileptic drug, often administered conjointly with an opioid is used for pain control. Gabapentin is more effective than carbamazepine.

Option D: Steroids have no proven role in management of Guillain-Barre syndrome.

How well did you know this?
1
Not at all
2
3
4
5
Perfectly
14
Q

On neurological assessment of a patient, he is unable to copy a pentagon. Which one of the following is the site of the lesion?

A. Temporal lobe
B. Temporoparietal lobe
C. Frontal lobe
D. Dominant parietal lobe
E. Non dominant parietal lobe

A

E. Non dominant parietal lobe

Drawing intersecting pentagon assesses the ability patients in constructional praxis (ie a task require three-dimensional manipulation). This is a task of non dominant parietal lobe that leads to constructional apraxia.

TOPIC REVIEW
The Mini Mental State Examination (MMSE) is a tool that to systematically and thoroughly assess mental status. It is an 11-question measure that tests five areas of cognitive function: (1) orientation, (2) registration, (3) attention and calculation, (4) recall, and (5) language. The maximum score is 30. A score of 23 or lower is indicative of cognitive impairment. The MMSE takes only 5-10 minutes to administer, and is practical to use repeatedly and routinely.

*** Read example of MMSE on page 719

Failing to answer the questions or doing the tasks indicates dysfunction of a particular area of the cortex:

How well did you know this?
1
Not at all
2
3
4
5
Perfectly
15
Q

To perform a dilation and curettage on a woman, she is placed in lithotomy position, after which she is found to have developed a nerve injury. Which one of the muscles is most likely to be affected by this injury?

A. Extensor hallucis longus.
B. Flexor digitorum longus.
C. Tibialis posterior.
D. Soleus.
E. Quadriceps.

A

A. Extensor hallucis longus

The most commonly injured lower extremity nerve in patients undergoing surgery in lithotomy position is the common peroneal nerve. The injury is thought to be secondary to compression of the nerve between the lateral aspect the fibular head and the bar holding the leg, especially when candy cane stirrups are used (see below). Low weight, smoking and prolonged surgery are risk factors for this injury.

(See photo of nerves in lithotomy position below)

At the level of the popliteal fossa, the sciatic nerve bifurcates into tibial and common peroneal (fibular) nerves. Medial to the course of the biceps femoris muscle, common peroneal nerve runs laterally and inferiorly, wraps around the fibular head and enters the lateral compartment of the leg. It passes between the attachments of the fibularis longus muscle, where the nerve divides into the superficial and deep fibular (peroneal) nerves.

Injury to common peroneal nerve can cause dysfunction of both deep and superficial peroneal nerves that can present with the following:
- Loss of dorsiflexion and eversion of the foot (equinovarus deformity)
- Sensory manifestations along the anterolateral border of the leg and dorsum of the toes except those supplied by saphenous and sural nerves.

The following muscles of the leg are innervated by deep and superficial branches of the common peroneal nerve:
- Tibialis anterior
- Extensor digitorum longus
- Extensor hallucis longus
- Peroneous tertius
- Peroneous longus
- Peroneous brevis

Of the given options, only extensor hallucis longus can be affected as a result of common peroneal nerve injury in lithotomy position. In fact, inability to extend the greater toe is a common finding in patients with injured common peroneal nerve.

Option B, C and D Soleus (plantar flexor), tibilalis posterior (plantar flexor and ankle inverter), and flexor muscles of the foot, including flexor digitorum longus are innervated by tibial nerve; therefore, unaffected with common peroneal nerve injury.

Option E Quadriceps muscle is supplied by femoral nerve and unaffected by common peroneal injury.

NOTE : Lower leg manifestations of L5 radiculopathy can be exactly similar to the common peroneal nerve injury.

How well did you know this?
1
Not at all
2
3
4
5
Perfectly
16
Q

A 45-year-old man presents with loss of inversion, eversion, dorsiflexion and plantar flexion of his left foot. His left knee jerk is preserved, but left ankle reflex is weak. Which one of the following can be the cause of this presentation?

A. Sciatic nerve injury.
B. Common peroneal nerve injury.
C. Tibial nerve injury.
D. L4/L5 disc herniation.
E. Femoral nerve damage.

A

A. Sciatic nerve injury

Loss of ankle inversion in this man can be either due to dysfunction of deep peroneal nerve (a branch of common peroneal nerve), or tibial nerve that supply the two main ankle invertors: tibialis anterior and tibialis posterior.

Ankle eversion is the action of the muscles in the lateral compartment of the leg: peroneous (fibularis) longus, peroneous (fibularis) brevis and peroneous tertius. These muscles are supplied by the other main branch of the common peroneal nerve, superficial peroneal nerve.

Plantar flexion of the ankle is mediated mostly by the muscles in the posterior compartment of the leg, all of which are innervated by tibial nerve. On the other hand, ankle reflex in this patient is weak and suggests involvement of the S1 (and to a lesser extend S2).

In fact, the clinical picture is only justified if both common peroneal and tibial nerves are involved and affected. This can be explained by injury to sciatic nerve.

Tibial nerve and common peroneal (fibular) nerve exit from the pelvis together contained in a sheath of connective tissue to form the sciatic trunk (nerve). In fact, the sciatic nerve is the common peroneal nerve and tibial nerves running down together in the buttock and back of the thigh. In the distal thigh, these two nerves separate and run different paths.

Injury to sciatic nerve (trunk) can present with motor and sensory deficits attributable to both the tibial nerve and common peroneal nerve.

The clinical findings include:
1. Common peroneal nerve:
-Weak or absent ankle eversion
-Weak or absent ankle dorsiflexion (drop foot)
-Weak or absent toes extension
-Weak ankle inversion
-Paresthesia / impaired sensation over the lateral aspect of the leg and dorsum of the foot (except the areas supplied by saphenous and sural nerves)

  1. Tibial nerve:
    -Weak or absent plantar flexion (the patient cannot stand on the ball of his foot on the affected side)
    -Weak ankle inversion
    -Weak or absent toes flexion
    -Weak or absent ankle jerk reflex
    -Paresthesia/impaired sensation over the posterior part of the leg and sole of the foot (except those areas supplied by saphenous and sural nerves)

In summary, injury to sciatic trunk causes weakness of ankle movements in all directions, impaired or absent ankle reflex and impaired sensation below the knee except the medial aspect of the leg and the outer part of the foot. Knee reflex usually remains intact.

Option B: Common peroneal nerve damage (in isolation) explains loss of sensation over the lateral leg, as well as weakened dorsiflexion, eversion and inversion, but not the weak plantar flexion.

Option C: Tibial nerve injury (in isolation) explains the absence of the ankle plantar flexion and impaired ankle jerk, but not the other clinical aspects.

Option D: Lateral L4/L5 disc herniation affects the L5 nerve root and results in L5 radiculopathy. L5 and common peroneal nerve injuries present similarly in the lower leg. L5 radiculopathy does not cause weak plantar flexion and impaired ankle reflex.

Option E: Femoral nerve supplies all the muscles in the anterior compartment of the thigh, as well as the sensation of the thigh except the posterior part (which is supplied by the sciatic trunk) and a small medial part (supplied by the obturator nerve), and through the saphenous branch, the medial aspect of the leg and outer part of the foot.

How well did you know this?
1
Not at all
2
3
4
5
Perfectly
17
Q

Which one of the following is caused by damage to the trigeminal nerve?

A. Forehead wrinkling.
B. Vision loss.
C. Double vision.
D. Loss of taste.
E. Difficulty in opening the mouth.

A

E. Difficulty in opening the mouth

Trigeminal Nerve (CN V) Overview:

  • Function:
    • Sensory: Carries sensations (pain, temperature, touch) from the face, mucosa, teeth, and portions of the dura.
    • Motor: Controls muscles of mastication (chewing).
  • Divisions:
    1. Ophthalmic (Upper):
      • Innervates: Forehead, upper eyelid, cornea, conjunctiva, dorsum of the nose.
      • Path: Leaves through the superior orbital fissure, travels via the cavernous sinus.
    2. Maxillary (Middle):
      • Innervates: Upper lip, lateral/posterior nose, upper cheek, temple, nasal mucosa, upper jaw/teeth, roof of the mouth.
      • Path: Leaves through the foramen rotundum, travels via the inferior cavernous sinus.
    3. Mandibular (Lower):
      • Innervates: Lower lip, chin, posterior cheek, temple, external ear, lower mouth mucosa, anterior 2/3 of the tongue.
      • Path: Leaves through the foramen ovale.
  • Motor Division:
    • Supplies muscles of mastication: Masseter, temporal, pterygoid, mylohyoid, digastric.
    • Function: Controls jaw movements (elevation, depression, protrusion, retraction, side-to-side).

Key Points:
- Difficulty Opening Mouth:
- A lesion in the motor part of the trigeminal nerve affects the muscles of mastication, making it hard to open the mouth and chew.
- Unilateral Paralysis: Jaw deviates to the paralyzed side when opening.
- Bilateral Paralysis: Jaw droops, no movement possible.

  • Jaw Jerk Reflex:
    • Normal: Brisk contraction when mandible is tapped.
    • Abnormal (UMN Lesion): Hyperactive or clonus.
    • Absent (Nuclear/Infranuclear Lesion): No reflex.

Other Functions (Non-Trigeminal Nerve):
- Forehead Wrinkling: Facial nerve (CN VII).
- Vision: Optic nerve (CN II).
- Diplopia (Double Vision): Extraocular muscles (CN III, IV, VI).
- Taste: Facial (CN VII), Glossopharyngeal (CN IX), Vagus (CN X).

Important Reminder:
- Trigeminal Nerve (CN V): Provides general sensation to the tongue but not taste.

Example Question:
- Scenario: Difficulty in opening the mouth, no other cranial nerve symptoms.
- Answer: Lesion in the trigeminal nerve (CN V), specifically affecting the motor division.

Easy Memory Tip:
- Three Divisions: Think “O-M-M” (Ophthalmic, Maxillary, Mandibular).
- Muscles of Mastication: “My Time Produces Delicious Meals” (Masseter, Temporal, Pterygoid, Digastric, Mylohyoid).
- Jaw Jerk Reflex: Normal is a “quick jerk,” abnormal is “hyper or absent.”

Trigeminal nerve (NC V) has three divisions that carry pain, temperature and touch modalities from the skin of the face, the mucosa of sinuses, nose, mouth, the teeth, and portions of the dura. They also carry proprioceptive sensation from the teeth, hard palate, temporomandibular joint, and muscles of mastication. Motor fibers are included in the maxillary division of the trigeminal nerve.

The three divisions of this nerve are:
1. Ophthalmic (upper) division - Innervates forehead, upper eyelid, cornea (thus the corneal reflex - different from pupilary reflex), conjunctiva, dorsum of the nose, and the dura of some of the anterior cranial fossa.
This division leaves orbit through the superior orbital fissure. It then proceeds through the lateral wall of the cavernous sinus in close relation to the CN III, CN IV and CN VI and joins the other two divisions to form the trigeminal (semilunar, Gasserian) ganglion.

  1. Maxillary (middle) division – Innervates the upper lip, lateral and posterior portions of nose, upper cheek, anterior temple, mucosa of the nose, upper jaw, upper teeth, the roof of the mouth, and the dura of part of the middle cranial fossa. The nerve leaves the pterygopalatine fossa, passes through the foramen rotundum, traverses the inferior part of the cavernous sinus, and enters the trigeminal ganglion.
  2. Mandibular (lower) division – Supplies the lower lip, chin, posterior cheek, temple, external ear, mucosa of the lower part of mouth, anterior two-thirds of the tongue, and portions of the dura of anterior and middle cranial fossae. Proprioceptive impulses are carried largely in the motor nerve, which is incorporated into the mandibular division. It enters the cranium through the foramen ovale and goes to the trigeminal ganglion.

Sympathetic and parasympathetic fibers join the three divisions and are distributed to the pupil, to the nasal mucosa causing mucus secretion, to the lacrimal, submaxillary, and sublingual glands, and to the arterioles of the face.

The motor division of the nerve (incorporated in the mandibular division) supplies the muscles of mastication including:
-Masseter
-Temporal
-Pterygoid
-Mylohyoid
-Digastric.

These muscles produce elevation, depression, protrusion, retraction, and the side-to-side movements of the mandible; therefore, lesions of motor portion of this nerve lead to dysfunction of these muscles and difficulties in opening the mouth and chewing, and impairment of various mandible (jaw) movements. The motor division also supplies the tensor tympani and tensor palati muscles.

When there is unilateral paralysis of the masticatory muscles, on mouth opening, the mandible deviates toward the paralyzed side. This direction of the mandible is due to the action of normal pterygoids on the opposite side. The mandible droops, and no jaw movement is possible with bilateral paralysis. The involved muscles undergo atrophy in nuclear or infranuclear lesions.

The jaw jerk is one of the deep tendon or stretch reflexes. When it is normal, tapping the mandible produces a brisk contraction. When abnormal, with upper motor neuron lesions, there is a hyperactive or repeating reflex (clonus). With nuclear or infranuclear lesions, the reflex is absent.

Option A Forehead wrinkling is an action of the 7th cranial nerve (facial nerve).

Option B Carrying visual stimuli is a function of 2nd cranial nerve (optic nerve).

Option C Diplopia (squint) is mostly caused by dysfunction of extraocular muscles innervated by CN III, CN IV and CN VI.

Option D Sense of taste is mediated by three cranial nerves:
1. Facial nerve (CN VII)
2. Glossopharyngeal nerve (CN IX)
3. Vagus nerve (CN X)

Facial nerve (chorda tympani and the greater petrosal branch nerves) provides the sense of taste in the anterior 2/3 of the tongue. Glossopharyngeal nerve provides that of the posterior 1/3 of the tongue. The superior laryngeal branch of vagus (CN X) innervates taste buds in the laryngeal surface of the epiglottis. The trigeminal nerve provides the general sensation of the tongue not the taste sense; therefore, CN V lesions do not result in taste loss.

See Page 732 for photo of Tongue Nerves

How well did you know this?
1
Not at all
2
3
4
5
Perfectly
18
Q

One week after a knee surgery, a 24-year-old man presents with complaints of numbness and paresthesia of the right leg. On examination, there is foot drop and weakness of dorsiflexion and eversion of the right ankle. Sensation over the outer aspect of the right leg is also lost. Right ankle jerk is intact. Which one of the following nerves is most likely to have caused such presentation?

A. L4 nerve root.
B. L5 nerve root.
C. Common peroneal nerve.
D. Tibial nerve.
E. Sciatic nerve.

A

C. Common peroneal nerve

The clinical picture suggests damage to the common peroneal nerve as an adverse outcome of the knee surgery. Foot drop is the result of weakness of the ankle dorsiflexors (extensors) innervated by the deep peroneal nerve (with the exception of peroneous tertius, a weak ankle dorsiflexor located in the lateral compartment of the leg, and supplied by the superficial peroneal nerve). Sensation over the outer aspect of the leg is supplied by the common peroneal nerve and its superficial branch. These together make the common peroneal nerve injury the most likely explanation.
After branching off the sciatic nerve, the common peroneal nerve runs down laterally, wraps around the fibular head and enters the leg. The nerve is superficial in this region and susceptible to injuries during knee surgeries, with compression or in trauma.

Option A L4 nerve root is in the lumbar area. L4 radiculopathy can cause weakened knee reflex and partially impaired ankle inversion, but does not explain other manifestations.
Option B L5 nerve root damage has a very similar clinical presentation in lower leg to that of the common peroneal nerve injury; however, the history of knee surgery makes the latter more likely.
Option D Tibial nerve damage causes impaired ankle jerk, weak or absent plantar flexion and weak ankle inversion. Sensory impairment due to tibial nerve injury affects the back the leg and most parts of the sole of the foot.
Option E Sciatic trunk (nerve) is above the knee, and unlikely to have been affected by the knee surgery. Moreover, sciatic damage affects the ankle and foot movements globally, and gives a clinical picture consistent with injuries of both common peroneal and tibial nerves at the same time.

How well did you know this?
1
Not at all
2
3
4
5
Perfectly
19
Q

A 75-year-old woman underwent total left hip replacement surgery yesterday. Today, she has developed weakness and numbness of her left foot. On examination, there is weakness of all the left ankle movements including dorsiflexion, plantar flexion, eversion and inversion, and numbness over the dorsum and sole of her foot and lateral aspect of her leg. The left ankle jerk is absent. Which one of the following is the most likely site of the lesion?

A. Left common peroneal nerve.
B. Left femoral nerve.
C. Left obturator nerve.
D. Left sciatic nerve.
E. Left tibial nerve.

A

D. Left sciatic nerve

Ankle dorsiflexion and eversion are actions of the deep peroneal and superficial peroneal nerves, respectively. Inversion is supplied by deep peroneal nerve (anterior tibialis muscle) and tibialis nerve (posterior tibialis muscle). Plantar flexion and ankle reflex are supplied by the tibial nerve. The efferent limb of the ankle jerk reflex is innervated by S1 and to lesser extend S2 fibers within this nerve. The pattern of the sensory impairment indicates that both nerves are affected.

As a matter of fact, this woman has a clinical presentation consistent with both common peroneal and tibial nerves injuries. When such clinical presentation is encountered, injury to the sciatic nerve is the most likely explanation. This can be the most likely diagnosis given the history of hip surgery.

The sciatic nerve is derived from lumbosacral plexus. Once formed, it leaves the pelvis and enters the gluteal region via greater sciatic foramen. It emerges inferiorly to the piriformis muscle, and enters the posterior thigh by passing deep to the long head of the biceps femoris, and descends in an inferolateral direction. Within the posterior thigh, the nerve gives off branches to the hamstring muscles and adductor magnus. When the sciatic nerve reaches the apex of the popliteal fossa, it terminates by bifurcating into the tibial and common fibular (peroneal) nerves.

NOTE - The sciatic nerve can be described as two individual nerves bundled together in a sheath of connective tissue. These two nerves are the tibial and common peroneal nerves. These two nerves usually separate at the apex of the popliteal fossa, however in a minority or persons, they separate as they leave the pelvis.

Sciatic nerve injury can occur during hip surgeries. Stretch, compression, ischemia, and direct damage are primary mechanisms. The lithotomy, frog leg, and sitting positions have been implicated in perioperative injury to this nerve. Regional anesthetic techniques and hip arthroplasty may also cause injury. The common peroneal component is more common to be affected because it is more superficial compared with the tibial component.

Sciatic nerve Injury manifests as paralysis of the hamstring muscles (knee flexion weakness) and all the muscles below the knee. All sensation below the knee is affected with the exception of the sensation of the medial aspect of the leg and lateral aspect of the foot, which remains intact because these two areas are supplied by saphenous nerve (a sensory branch of the femoral nerve) and sural nerve, respectively.

OPTION A : Left common peroneal nerve injury results in foot drop (loss of ankle dorsiflexion), weak inversion and inability of the patient to evert the ankle. Sensory impairment occurs over the lateral aspect of the leg and dorsum of the left foot (except the lateral margin of left foot).

OPTION B : Left femoral nerve injury causes global weakness of all muscles in the anterior compartment of the thigh, impaired knee jerk and sensory disturbances of the lateral, and anteromedial surfaces of the thigh, as well as the medial aspect of the leg.

OPTION C : Obturator nerve consists of L2, L3, and L4 nerve roots. It innervates the muscles adductor longus, adductor brevis, adductor magnus, gracilis, and obturator externus. It provides, along with the femoral nerve, the sensation of the medial aspect of the thigh. Obturator nerve injury presents with medial thigh or groin pain, weakness of leg adduction and sensory loss in the medial thigh.

OPTION E : Tibial nerve damage results in weak plantar flexion. Impaired ankle jerk and sensory disturbances over the posterior aspect of the leg and sole of the foot are other manifestations.

How well did you know this?
1
Not at all
2
3
4
5
Perfectly
20
Q

A 26-year-old-man develops weakness of foot plantar flexion and inversion. Which nerve is most likely to have been damaged?

A. Common peroneal nerve.
B. Tibial nerve.
C. Sciatic nerve.
D. Lumbosacral plexus.
E. Lumbar nerve roots.

A

B. Tibial nerve

Plantar flexion is provided by the ankle flexor muscles in the posterior compartment of the leg, all of which are innervated by the tibial nerve. Ankle inversion is the action of two muscles: tibialis anterior and tibialis posterior Tibialis anterior is supplied by deep peroneal nerve, while tibialis posterior in the posterior leg compartment is innervated by tibial nerve. This movement can be affected with injuries to the tibial or deep peroneal nerves.

OPTION A : With involvement of the common peroneal nerve, foot drop (caused by weakness of the ankle extensors) and sensory disturbances over the outer surface of the leg is expected. Ankle jerk reflex remains intact.

OPTION C : With sciatic nerve injury, global weakness of all ankle and foot movements occurs. Ankle reflex is also affected. Weakness of the knee flexion may be noticed as well.

OPTION D : Lumbosacral plexus problems have various presentations depending on the affected nerve root. S1 radiculopathy presents with similar manifestation seen in tibial nerve injury; however, back pain radiating from low back and the buttock down to the posterior aspect of the leg into the foot is a feature not mentioned here. It makes lumbosacral plexus involvement less likely.

OPTION E : Lumbar nerve root problems cause deficits in the movements and sensation of the hip, thigh and knee extension and reflex.

*http://www.medscape.com/viewarticle/758724_7
*http://www.orthobullets.com/anatomy/10115/sciatic-
*http://www.orthobullets.com/anatomy/10116/tibial-n
*http://ceaccp.oxfordjournals.org/content/early/201
*http://www.uptodate.com/contents/acute-lumbosacral

How well did you know this?
1
Not at all
2
3
4
5
Perfectly
21
Q

A 26-year-old man develops weakness of foot dorsiflexion and eversion. Which nerve is most likely to have been damaged?

A. Common peroneal nerve.
B. Tibial nerve.
C. Sciatic nerve.
D. Lumbosacral plexus.
E. Lumbar nerve roots.

A

A. Common peroneal nerve

Ankle eversion is an action of three muscles in the lateral compartment of the leg, including:
-Peroneous (fibularis) longus
-Peroneous (fibularis) brevis
-Peroneous tertius

These muscles are innervated by the superficial peroneal nerve which is one of the two main branches of the common peroneal nerve.

Ankle dorsiflexion is the action of the following:
-Tibialis anterior
-Extensor hallucis longus
-Extensor digitorum longus
-Peroneous (fibularis) tertius

Peroneous tertius is innervated by the superficial peroneal nerve. The other three are supplied by deep peroneal nerve, the other main branch of the common peroneal nerve. With both ankle dorsiflexion and eversion affected, the common peroneal nerve is most likely to be involved.

OPTION B: Tibial nerve innervates the posterior compartment of the leg. Muscles in this compartment are mostly ankle plantar flexors.
With tibial nerve injury plantar flexion and ankle jerk will be affected.

OPTION C : With sciatic nerve involvement, global weakness and impairment of all ankle and foot movements is expected. Ankle reflex will be affected as well. Weakness of the knee flexion could also be problem.

OPTION D : Clinical findings associated with lumbosacral plexus problems vary depending on the affected nerve root. S1 radiculopathy presents similar to tibial nerve injury with impaired plantar flexion, ankle jerk and sensory disturbances in the back of the leg and most parts of the sole of the foot.

OPTION E : Lumbar nerve root problems cause deficits in the movements and sensation of the hip, thigh and knee extension and deep tendon reflex.

How well did you know this?
1
Not at all
2
3
4
5
Perfectly
22
Q

Which one of the following can be a presentation associated with L5/S1 disc prolapse?

A. Absent knee reflex.
B. Absent ankle reflex.
C. Impaired ankle dorsiflexion.
D. Impaired ankle eversion.
E. Impaired sensation over the lateral aspect of the leg.

A

B. Absent ankle reflex

In L5/S1 disc prolapse, which often occurs laterally, the herniated disc compresses the S1 nerve root causing S1 radiculopathy. S1 radiculopathy presents with pain radiating down the posterior aspect of the leg into the foot from the back.
On examination, weakness of plantar flexion is specific. Weakness of leg extension (gluteus maximus) and toe flexion are other possible features. Sensation is generally reduced on the posterior aspect of the leg and the lateral edge of the foot. Loss of the ankle reflex is typical.

OPTION A : Knee reflex is an action of L3 and L4 nerve roots.

OPTION C, D and E : Ankle dorsiflexion, eversion, and sensation over the lateral aspect of the leg are innervated by L5 that can be affected in L4/L5 disc prolapse.

How well did you know this?
1
Not at all
2
3
4
5
Perfectly
23
Q

Which one of the following is most suggestive of sciatic nerve injury?

A. Absent ankle reflex.
B. Foot drop.
C. Inability to flex the hip.
D. Decreased sensation over the anterior thigh and medial leg.
E. Intervertebral disc prolapse at L2/L3 level.

A

A. Absent ankle reflex

The sciatic nerve is derived from the lumbosacral plexus. Once formed, it leaves the pelvis and enters the gluteal region via greater sciatic foramen. It emerges inferiorly to the piriformis muscle, and enters the posterior thigh by passing deep to the long head of the biceps femoris, and descends in an inferolateral (down and out) direction. Within the posterior thigh, the nerve gives off branches to the hamstring muscles and adductor magnus. When the sciatic nerve reaches the apex of the popliteal fossa, it terminates by bifurcating into the tibial and common fibular nerves.

The sciatic nerve can be described as tibial and common peroneal nerves bundled together in a sheath of connective tissue.

Sciatic nerve Injury manifests as paralysis of the hamstring muscles (knee flexion weakness) and all the muscles below the knee. All sensation of the leg except the medial aspect is impaired. Absent or weak ankle reflex is most specific to sciatic nerve injury among other options.

OPTION B : Foot drop is seen in sciatic as well as common peroneal nerve injuries and is not specific to sciatic nerve injury.

OPTION C : Hip flexion is the action of L1-L2 nerve roots. Hip flexion is not an action of the sciatic nerve which is formed by L4, L5, S1, S2, and S3 nerve roots.

OPTION D : Decreased sensation over the anterior thigh and medial leg is caused by femoral nerve injuries.

OPTION E : Intervertebral disc lesion at L2/L3 compromises the action of L3 nerve root, which does not contribute to the formation of the sciatic nerve.

How well did you know this?
1
Not at all
2
3
4
5
Perfectly
24
Q

A 65-year-old man is being seen in the Emergency Department because of severe headache starting two hours ago. On a scale of 1 to 10, the patient scores the pain as 10 in intensity. There is no history of similar headaches in the past. Relatives accompanying the patient inform you that he lost consciousness 30 minutes after the headache started but he was lucid again few minutes later. Which one of the following could be the most likely cause of the headache?

A. Subdural hematoma.
B. Epidural hematoma.
C. Cerebral abscess.
D. Subarachnoid hemorrhage.
E. Migraine.

A

D. Subarachnoid hemorrhage

Of the options, subarachnoid hemorrhage (SAH) is most consistent with the clinical scenario. Symptoms of SAH typically begin abruptly. The most common and premier symptom is a sudden, severe headache occurring in 97% of patients. Patients often describe the headache as the “worst headache of my life.” The onset of the headache may be associated with a brief loss of consciousness, seizure, nausea, or vomiting. Signs of meningeal irritation may be present but may not develop until several hours after the bleed, because it is caused by the breakdown of blood products within the CSF and the consequent aseptic meningitis.

Meningismus can be elicited by a positive Kernig’s sign (extending the patient’s knee while the hip is flexed at 90° causes pain) and/or Brudzinski’s sing (flexing the patient’s head while the chest is restrained with the physician’s other hand causes flexion of the hip and the knees). Neurological deficits such as hemiplegia, third cranial nerve palsy, etc. may be found on physical examination as well.

A non-contrast CT scan is the initial investigation of choice and should be performed as soon as possible to confirm the diagnosis.

OPTION A : Subdural hematoma is due to a venous bleed between the dura and the arachnoid. It follows injuries that may be trivial, especially in the elderly. It can be acute (less than two days), subacute (2-14 days), or chronic (more than 14 days).

Subdural hematoma should be considered if a person presents with the following:
-Personality changes
-Slowness and unsteadiness of movements
-Headache
-Irritability
-Fluctuating level of consciousness

CT scan of the head is the confirmatory diagnostic tool of choice for subdural hematoma.

OPTION B : Epidural hematoma is a life-threatening head injury that is caused by arterial bleeding between the skull bone and dura mater. The typical clinical scenario is loss of consciousness following trauma to the head which is usually followed by a short lucid interval before consciousness deteriorates further. The patient with epidural hematoma is usually restless, confused, irritable, and may develop severe headache, seizures, ipsilateral pupil dilatation and facial weakness. Lumbar puncture is contraindicated in this situation. CT scan of the head is used to confirm the diagnosis.

OPTION C : In cerebral abscess, symptoms are more insidious and usually develop over two weeks or so. Presentation is usually with fever, confusion and decreased responsiveness.

OPTION E : Classic features of migraine include headache (frontal or temporal) radiating to retro-orbital and/or occipital area. The pain is aggravated by tension and relieved by rest particularly in a dark silent environment. Loss of consciousness is not a feature.

How well did you know this?
1
Not at all
2
3
4
5
Perfectly
25
Q

A 52-year-old man presents to your clinic for evaluation of spasticity and weakness in his left lower limb. The condition has progressively worsened over the past three months and made walking difficult for him. On examination, fasciculation in the left deltoid muscle and small muscles of the hand are noted; however, the muscle power, reflexes and sensation of the upper limbs are within the normal range. In the lower limb, bilateral extensor plantar reflexes, left-sided foot drop and muscle weakness and spasticity are noted. Lower limb sensation is intact. Which one of the following investigations is the most appropriate one to reach a diagnosis?

A. CT scan of the brain.
B. MRI of the cervical spine.
C. Electromyography.
D. Lumbar puncture (LP).
E. MRI of the brain.

A

C. Electromyography

While fasciculation of the deltoid and small muscles of the hand and the foot drop in this patient suggest a lower motor neuron (LMN) problem, increased muscle tone (spasticity) and extensor plantar reflex are pointers toward an upper motor neuron lesion. The combination of LMN and UMN findings in the same area of the body as well as the intact sensation puts amyotrophic lateral sclerosis (ALS), the most common motor neuron disorder, at the top of the differential diagnoses list.

Motor neuron disorders (MNDs) are a clinically and pathologically heterogeneous group of neurologic diseases characterized by progressive degeneration of motor neurons. Either or both of the following two sets of motor neurons can be affected:
1. Upper motor neurons (UMNs) – These neurons originate from the primary motor cortex of the cerebrum (precentral gyrus). These neurons have long axons that form corticospinal and corticobulbar tracts.
2. Lower motor neurons (LMNs) – These neurons originate from the brainstem (cranial nerve [CN] motor nuclei) and spinal cord (anterior horn cells) and directly innervate skeletal muscles.

MNDs can be classified into those affecting primarily the UMNs, those affecting primarily the LMNs, and those affecting both. The patient’s symptoms vary, depending on which set of motor neurons is involved.

Motor neuron disorders include:
-Amyotrophic lateral sclerosis (ALS)
-Primary lateral sclerosis (PLS)
-Hereditary spastic paraparesis (HP)
-Progressive bulbar palsy (PBP).
-Spinal muscular atrophy.
-X-linked spinobulbar muscular atrophy.
-Post-polio syndrome

ALS, the most common neurodegenerative disease of adult onset, is a fatal disorder and is characterized by progressive skeletal muscle weakness and wasting or atrophy (i.e., amyotrophy), spasticity, and fasciculations as a result of degeneration of both the UMNs and LMNs. ALS eventually results in respiratory paralysis. Both LMN and UMN abnormalities should be observed in a single area before ALS could be considered a highly likely diagnoses. ALS is sporadic in 90-95% of cases and hereditary in only 5-10% of affected individuals.

ALS should be suspected as a diagnosis when there is insidious loss of function or gradual, slowly progressive, painless weakness in one or more regions of the body, without changes in sensation, and there is no other explanation for to this presentation. With time, patients with ALS develop manifestations of both LMN and UMN involvement.

Findings indicating UMN involvement include the following:
-Stiffness (spasticity)
-Brisk tendon reflexes (hyperreflexia) – but maybe diminished if there is concurrent LMN
-Presence of abnormal reflexes (e.g., Babinski, Chaddock, or Hoffman signs)
-Loss of dexterity in the presence of normal strength
-Muscle spasms

Findings indicating LMN dysfunction include the following:
-Twitching muscles (fasciculations) – this can be an early manifestation most commonly seen in the tongue and limbs
-Reduction of muscle bulk (atrophy)
-Foot drop (or wrist drop in case of upper limb involvement)
-Depressed reflexes
-Breathing difficulties

The diagnosis of ALS is primarily clinical. When the disease has progressed far in its course and involves many parts of the body, the patient’s appearance and neurologic examination findings often provide sufficient evidence for the diagnosis. However, if the patient presents early in the course of the disease, the diagnosis is not straightforward, and often exclusion of other possible causes is required before a diagnosis of ALS is made because there is no pathognomonic test for ALS (or other motor neuron disorders) and the symptoms are initially nonspecific. Often, several months (average 14 months) are needed before a definite diagnosis of ALS is made.

Of the investigations currently in use for workup and assessment of ALS (and other motor neuron disorders), electromyography (EMG) and nerve conduction studies (NCS) are most useful for confirming the diagnosis of ALS and for exclusion of peripheral conditions that resemble ALS. The role of EMG/NCS in diagnosis of ALS is so crucial that some consider it as an extension to physical examination.

Imaging studies such as CT scan (option A) or MRI (option B and E) may be primarly used in some patients for assessment of other conditions, such as multiple sclerosis (MS), spinal canal stenosis or other spinal lesions that can mimic initial presentations of ALS. Imaging studies are usually normal in patients with ALS, and not diagnostic for the condition.

OPTION D : LP and examination of the CSF is not necessary unless the patient has a pure UMN or pure LMN presentation, in which case it can be of diagnostic value in ruling out inflammatory conditions, neoplastic infiltrations, or infection. With a combination of both UMN and LMN manifestations, this patient will not benefit from LP as a diagnostic tool.

How well did you know this?
1
Not at all
2
3
4
5
Perfectly
26
Q

Alan, 45 years, is a new patient to your clinic. He has presented with complaint of frequent tripping of his right foot for the past few months, which has increased in frequency in the past month. On examination, interosseous muscles wasting of the right foot is noted. Plantar reflex is equivocal on the right but extensor on the left. Lower limb deep tendon reflexes are normal. There is foot drop on the right side. Which one of the following tests is most likely to make a diagnosis?

A. MRI of the head and cervical spine.
B. Acetylcholine receptor antibodies.
C. CT scan of the head and cervical spine.
D. Lumbar puncture (LP).
E. Electromyography (EMG).

A

E. Electromyography (EMG)

Alan has signs of both lower motor neuron (LMN) and upper motor neuron (UMN) involvement in different parts of his body. He has foot drop and muscle wasting in his right foot (LMN signs), while his left foot shows an upward plantar reflex (Positive Babinski sign, an UMN sign). His right plantar reflex is unclear, making it hard to determine if the right foot issues are due to LMN or UMN problems. Overall, he has symptoms of both LMN and UMN lesions in different places.

The most likely diagnosis for Alan is amyotrophic lateral sclerosis (ALS), a type of motor neuron disease (MND). ALS typically presents with painless motor problems and normal sensations. Over time, it shows features of both LMN (e.g., muscle twitching, cramps, atrophy, foot drop) and UMN (e.g., spasticity, hyperreflexia, abnormal reflexes) involvement.

ALS usually starts in the limbs, with 75-80% of cases showing limb involvement first. Lower limb onset may cause tripping, stumbling, or foot drop, while upper limb onset can lead to reduced finger dexterity and hand weakness. Bulbar involvement (20-25% of cases) affects speech and swallowing.

Diagnosis often relies on clinical examination, with electromyography (EMG) and nerve conduction studies (NCS) being crucial for confirming ALS and excluding similar conditions.

Other tests, such as checking for acetylcholine antibodies (for myasthenia gravis) or lumbar puncture (for inflammatory conditions or infections), are not as relevant here given Alan’s combination of LMN and UMN symptoms. Imaging studies typically appear normal in ALS.

In summary, ALS is the most likely diagnosis given Alan’s mix of LMN and UMN signs, painless progression, and the pattern of neurological deficits.

Alan has signs of lower motor neuron (LMN) involvement of his right foot including the foot drop and muscle wasting. While his left foot has an upward plantar reflex (Positive Babinski) indicating upper motor neuron lesion, the right plantar reflex is equivocal making it hard to tell if the right foot signs and symptoms are LMN or UMN in origin. However, in general he has clinical findings suggesting both LMN and UMN lesions at different sites.

Among the differential diagnoses justifying such presentation, a motor neuron disorder (MND) is the most likely diagnosis. Spinal canal stenosis can be a differential diagnosis but is far less likely in this scenario because firstly chronic pain is the most common presenting symptom in spinal canal stenosis which is absent here. Secondly, stenotic lesions cause UMN manifestations at the level of stenosis and LMN features below that. The pattern of neurological deficits here does not match that of a stenotic spinal canal.

Given the pattern of clinical findings, amyotrophic lateral sclerosis (ALS) as the most common MND could be the most likely diagnosis in this case. ALS often presents with insidious onset of painless motor neurological deficits and intact sensations. With time, features of both LMN and UMN will be present in the affected area. While LMN involvement manifests with fasciculations (twitching muscles), muscle cramps, muscle atrophy, foot drop (or wrist drop) and depressed reflexes, features of UMN include spasticity (stiffness), hyperreflexia, presence of abnormal reflexes (such as Babinski, Chaddock, or Hoffman signs), loss of dexterity in the presence of normal muscle strength and muscle spasms.

In ALS, symptoms begin with limb involvement in 75-80% of cases and bulbar symptoms in 20-25%. Individuals with upper limb onset may experience reduced finger dexterity, cramping, stiffness, and weakness or wasting of intrinsic hand muscles. This may lead to difficulty with actions such as buttoning clothes, picking up small objects, or turning a key. Wrist drop may develop.

There is equal likelihood for presentation in either lower extremity. Patients who have lower limb onset initially may complain of tripping, stumbling, or awkwardness when running. Foot drop is common, and patients may report a “slapping” gait.

Bulbar involvement presents with slurred speech, hoarseness, or decreased volume of speech, or aspiration or choking during a meal. Hypernasality of the voice and eventually loss of speech, swallowing difficulty (usually starting with liquids), and drooling occurs later in the course of the disease. Women have a greater frequency of bulbar (speech dysfunction) onset than men.

Neurologic examination often provides sufficient evidence for the diagnosis.

Of the investigations currently in use for workup and assessment of ALS (and other motor neuron disorders), electromyography (EMG) and nerve conduction studies (NCS) are confirmatory for ALS and for exclusion of peripheral conditions that resemble ALS. The role of EMG/NCS in diagnosis of ALS is so crucial that some consider it as an extension to physical examination.

OPTION B : Assessment of acetylcholine antibodies is used when myasthenia gravis is suspected as a diagnosis. This disease typically affects the extraocular, bulbar, or proximal limb muscles. Droopy eyelids or double vision is the most common symptom at initial presentation (more than 75%). These symptoms progress from mild to more severe over weeks to months. Difficulty in swallowing, slurred or nasal speech, difficulty chewing, and facial, neck, and extremity weakness often follow.
Imaging studies are usually normal in patients with ALS.

OPTION D : LP and examination of the CSF is not necessary unless the patient has a pure UMN or pure LMN presentation, in which case it can be of diagnostic value in ruling out inflammatory conditions, neoplastic infiltrations, or infection. With a combination of both UMN and LMN manifestations, this patient will not benefit from LP as a diagnostic tool.

NOTE - For patients with a new focal presentation, the differential diagnoses by region include the following:
1. Upper motor neuron (UMN) bulbar signs - Brainstem lesions including syrinx, mass, stroke, and demyelinating forms of other degenerative diseases
2. Lower motor neuron (LMN) bulbar signs - Cranial nerve palsies
3. Limb UMN signs - Cervical myelopathy, cord tumor, hereditary spastic paraparesis, transverse myelopathy, HIV-related myelopathy, syrinx
4. Limb LMN signs - Radiculopathy, plexopathy, neuropathy

Differential diagnoses for patients with more advanced disease most commonly include the following:
i) UMN signs - Compressive myelopathy, syrinx
ii) LMN signs - Chronic inflammatory demyelinating polyradiculoneuropathy; multifocal motor, toxic, or metabolic neuropathies or myopathies such as inclusion body myositis or polymyositis

The following table summarizes the most important differential diagnoses of MNDs and their diagnostic tests:

(Table in page 744)

How well did you know this?
1
Not at all
2
3
4
5
Perfectly
27
Q

A 68-year-old woman presents with left hemiplegia. A CT scan of the brain is obtained and is shown in the following photograph. Past medical history is significant for a melanoma that was excised 3 years ago. Which one of the following could be the most likely diagnosis?

A. Metastasis from the melanoma.
B. Glioma.
C. Cerebral hemorrhage.
D. Cerebral abscess.
E. Meningioma.

A

A. Metastasis from the melanoma

The CT scan shows a round hyperattenuated lesion in the right hemisphere. These features are suggestive of either an abscess or a tumor. Since the patient is afebrile, abscess is less likely as the diagnosis, and a brain tumor will top the differential diagnoses list.
With history of melanoma, a metastatic brain tumor from the melanoma could be the most likely diagnosis.

Although, most cases of malignant melanoma are diagnosed at an early stage, when surgical excision can be curative, a few patients have metastatic disease at presentation, or develop metastases after the initial definitive surgical excision. Melanoma can metastasise to distant sites. Brain and lung are the most common sites of metastasis.

Melanoma is one of the three most common malignancies metastasising to the brain. The other two are breast cancer and lung cancer.

The following factors are associated with increased risk of systemic metastasis (including brain metastasis):
-Male gender
-Melanomas arising on mucosal surfaces or the skin of the trunk, head, or neck
-Wide, thick, deeply invasive, or ulcerated primary lesions
-Acral lentiginous or nodular lesions on histologic examination
-Involvement of more than three regional lymph nodes, either at diagnosis or relapse

If amenable, surgical resection of the metastatic brain tumors is the best appropriate treatment option; however, it is not the case often.

How well did you know this?
1
Not at all
2
3
4
5
Perfectly
28
Q

Lindsay, 35 years of age, presents to your practice with a 48-hours history of facial weakness. She has no remarkable past medical history, is on no medications currently, and has been fit and well to date. On examination, she is asked to smile. Her appearance while smiling is shown in the accompanying photograph. She has no tinnitus, vertigo, or hearing loss. Otoscopic exam is normal. Which one of the following options can be the most likely diagnosis?

A. Acoustic neuroma.
B. Bell’s palsy.
C. Central facial nerve lesion.
D. A parotid glad tumor.
E. Ramsay – Hunt syndrome.

A

B. Bell’s palsy

In the photograph, the forehead lines and nasolabial fold on the right side are lost on smiling. Also, the right mouth corner has failed to be elevated while she is smiling. These features suggest the complete right-sided facial palsy. With no clue on exam pointing toward a specific underlying cause, as well as the absence of vertigo or hearing problems, Bell’s palsy (idiopathic facial nerve palsy) is the most likely diagnosis.

Bell’s palsy is the most common cause of unilateral facial paralysis. This idiopathic condition, is one of the most common neurologic disorders of the cranial nerves. In most patients, Bell palsy gradually resolves over time.

Signs and symptoms of Bell palsy include the following:
-Acute onset of unilateral upper and lower facial paralysis (over a 48-hour period)
-Posterior auricular pain
-Decreased tearing
-Hyperacusis (increased hearing sensitivity)
-Taste disturbances (anterior 2/3 of the tongue is innervated by the facial nerve)
-Otalgia (ear pain)
-Weakness of the facial muscles
-Poor eyelid closure
-Aching of the ear or mastoid
-Tingling or numbness of the cheek/mouth
-Excessive watering of the eye
-Ocular pain
-Blurred vision
-Flattening of forehead and nasolabial fold on the side affected by palsy
-When patient raises eyebrows, palsy-affected side of forehead remains flat
-When patient smiles, face becomes distorted and lateralizes to side opposite the palsy

OPTION A : Acoustic neuroma presents with hearing loss, tinnitus and vertigo, none of which Lindsay has. Acoustic neuromas, however, can cause facial paralysis due to pressure on the facial nerve. This occurs later in the course of the disease. It is very unlikely that facial nerve palsy is the first presentation of an acoustic neuroma.

OPTION C : A central lesion of the facial nerve, e.g. caused by a lacunar stroke, spares the forehead due to bilateral motor neuron innervation. With an affected forehead, the palsy is always peripheral.

OPTION D : Malignant parotid tumors can invade the facial nerve in its course through the superficial and deep layers of the affected parotid gland and cause paralysis of the facial nerve. A painless enlarging mass is the typical initial presentation, the absence of which makes a parotid tumor as the cause a less likely diagnosis.
OPTION E : Ramsay – Hunt syndrome (herpes zoster infection of the facial nerve) is caused by the reactivation of dormant varicella zoster virus in the geniculate ganglion. It typically presents with peripheral facial palsy, ear pain, loss of taste, dry eyes and a vesicular rash. Although Ramsay- Hunt syndrome shares many clinical features with Bell’s palsy, the absence of vesicular rash in the ear canal (the otoscopic exam is normal) makes it a less likely diagnosis.

How well did you know this?
1
Not at all
2
3
4
5
Perfectly
29
Q

Peter, 48 years old, is on your office concerned about facial weakness, unpleasantly increased perception in the left ear, and that everything tastes bland. His symptoms have developed and progressed since yesterday. He denies hearing loss and tinnitus. He is an accountant and otherwise healthy with no previous health problems but an upper respiratory viral infection 10 days ago. He does not take any medications except supplemental vitamins. On examination, there is right-sided facial weakness. The following photograph illustrates his facial expression when he is asked to smile. Neurological examination is otherwise unremarkable. Hearing is unaffected. An otoscopic examination reveals normal ear canal and tympanic membrane. Which one of the following diagnostic investigations should be considered for him at this stage?

A. Nerve conduction studies (NCS).
B. CT scan of the head.
C. MRI of the head.
D. Swallowing test by a speech pathologist.
E. No investigation is required at this stage.

A

E. No investigation is required at this stage

Features evident in the photograph are loss of forehead wrinkles, failure of the right mouth corner to rise, absence of laugh lines, and failure of the eye to tighten up, all on the right side. These features suggest peripheral (lower motor neuron) right facial nerve palsy (in upper motor neuron palsy the forehead is spared due two dual innervation). Additional features of Hyperacusis and loss of taste are also confirmatory findings. Peter’s physical findings are otherwise normal and there is no significant past medical history to point towards any specific underlying etiology. These make idiopathic facial nerve palsy (Bell palsy) the most likely diagnosis.
Bell palsy is the most common cause of unilateral facial paralysis. Bell palsy gradually resolves over time. The cause is unknown; however, herpes simplex type 1 virus and herpes zoster virus have been implicated by some authors.

Clinical features of Bell palsy include:
-Acute onset of unilateral upper and lower facial paralysis (over a 48-hour period)
-Posterior auricular pain
-Decreased tearing
-Hyperacusis (increased hearing sensitivity)
-Taste disturbances (anterior 2/3 of the tongue is innervated by the facial nerve)
-Otalgia (ear pain)
-Weakness of the facial muscles
-Poor eyelid closure
-Aching of the ear or mastoid
-Tingling or numbness of the cheek/mouth
-Excessive watering of the eye
-Ocular pain
-Blurred vision
-Flattening of forehead and nasolabial fold on the side affected by palsy
-When patient raises eyebrows, palsy-affected side of forehead remains flat
-When patient smiles, face becomes distorted and lateralizes to side opposite the palsy

In many cases, the history and physical examination establish the diagnosis of Bell palsy, and no further investigation including imaging studies, laboratory tests and nerve conduction studies are required because most patients recover within 6- 8 weeks. Investigations are only indicated if the diagnosis is uncertain or the paralysis last longer 6-8 weeks.

OPTION A : NCS and electromyography (EMG) have prognostic value in Bell palsy but are most useful if performed 3-10 days after the onset of paralysis. Generally speaking, NCS/EMG is a tool used for research rather than diagnosis in Bell palsy.

NOTE - Most NCS/EMG studies do not show an abnormality for 3 weeks following a peripheral nerve injury.

OPTION B : Ct scan of the head is indicated if there are other associated physical findings or if the paresis is progressive and unremitting. CT scan is used for assessment of the temporal bone architecture, and, if used with contrast enhancement, for detection of tumors compressing the facial nerve (MRI is a better option for this purpose).

OPTION C : In patients with Bell palsy, MRI may show enhancement of the facial nerve at or near the geniculate ganglion. MRI, however, is not indicated unless the diagnosis is in doubt, symptoms do not resolve within 6-8 weeks, or symptoms are progressive. MRI is the preferred imaging modality for assessment of cerebellopontine angle, or if, based on the history and/or clinical findings, a tumor compressing the facial nerve is suspected. Such tumors include acoustic neuroma (Shcwannoma), hemangioma, meningioma, and sclerosing hemangioma. Acoustic neuroma is the most common tumor associated with facial nerve palsy.

OPTION D : Facial nerve has not a significant role in swallowing and swallowing remains mostly unaffected. Therefore, swallowing tests for assessment of dysphagia in patients with facial nerve palsy are not indicated.

How well did you know this?
1
Not at all
2
3
4
5
Perfectly
30
Q

A 58-year-old man presents with complaint of progressive difficulty in walking for the past few months. He denies any pain or pins and needles. On examinations, he drags his left foot on walking. On the left side, ankle tendon reflexes are exaggerated and the strength of plantar flexion and dorsiflexion are decreased to 4/5. Sensation is intact. The right foot is completely normal. Plantar reflexes are equivocal bilaterally. The rest of the physical examination including neurologic exam of the face and upper extremities are unremarkable. Lesions of which one of the following parts could be the mostly likely cause to this presentation?

A. Common peroneal nerve.
B. L5/S1nerve root.
C. Cervical spine.
D. Brainstem.
E. Cerebral cortex.

A

E. Cerebral cortex

One of important points to consider in dealing with patients who have presented with motor neurological deficits, such as this patient, is the possible origin(s) of such presentation. Distinguishing lower motor neuron lesions from those of upper motor neuron is of paramount importance.

Upper motor neurons (UMNs) neurons originate from the primary motor cortex of the cerebrum (precentral gyrus). These neurons have long axons that form corticospinal and corticobulbar tracts.
Lesions of UMN presents with:
-Stiffness (spasticity)
-Brisk tendon reflexes (hyperreflexia)
-Presence of abnormal reflexes (e.g., Babinski, Chaddock, or Hoffman signs)
-Loss of dexterity in the presence of normal strength
-Muscle spasms

Lower motor neuron (LMNs), on the other hand, originate from the brainstem (cranial nerve [CN] motor nuclei) and spinal cord (anterior horn cells) and directly innervate skeletal muscles.
Findings consistent with LMNs lesions include:
-Fasciculations (twitching muscles)
-Reduction of muscle bulk (atrophy)
-Decreased muscle strength
-Depressed deep tendon reflexes

While the weakness of muscles responsible for weak ankle flexion and dorsiflexion indicates an LMN lesion, the increased ankle tendon reflexes on the same side reflects an UMN problem. Therefore, this patient has both LMN and UMN disorders at the same time. Additionally, he has no sensory impairment. This constellation of findings makes a motor neuron disease (MND), more specifically, amyotrophic lateral sclerosis, the most likely diagnosis. Of the given options, only lesions of cerebral cortex can give rise to this scenario.

Injuries to the common peroneal nerve (option A) can weaken the ankle dorsiflexion and disturbed sensation over the anterolateral aspect of the leg and lateral aspect of the foot. However, such lesions are associated with a depressed ankle reflex. Moreover, with such injuries, sensation of the lateral aspect of the lower leg and foot that are innervated by the superficial branch of this nerve would be impaired.

Lesions of the L5/S1 nerve root (option B) can cause weak ankle dorsiflexion and drop foot (L5) and decreased plantar flexion (S1). Atrophy of muscles innervated by these nerve roots is another feature, but since the lesions are of lower motor origin, decreased reflexes would be expected. This patient has also features consistent with UMN deficits that makes this diagnosis unlikely.

Cervical spine lesions (option C ) caused by conditions such as multiple sclerosis (MS), or spinal canal stenosis/compression can cause signs and symptoms consistent with UMN lesions. But with cervical spine involvement, neurological deficits are expected to be more pronounced in upper extremities than the lower extremities. Normal neurologic exam findings in the face and arms make cervical spine lesions a less likely diagnosis.

OPTION D : UMN lesions of the brainstem origin is almost an unlikely diagnosis in this scenario due to lack of signs and symptoms suggestive of bulbar involvement such as slurred speech and oropharyngeal dysphagia, especially for liquids, or facial involvement.

How well did you know this?
1
Not at all
2
3
4
5
Perfectly
31
Q

A 56-year-old man presents to the emergency department with complaint of diplopia. On examination, he has right-sided ptosis. His visual acuity and visual fields are unaffected. His right eye is depressed inferiorly and laterally and he is not able to look at to the left side. The pupillary light reflex of the right eye is sluggish. The neurological examination is otherwise unremarkable. Which one of the following can be the most likely cause to this presentation?

A. Midbrain infarct.
B. Right 6th nervepalsy.
C. Posterior communicating artery aneurysm.
D. Right 4th nerve palsy.
E. A lesion in the occipital cortex.

A

C. Posterior communicating artery aneurysm

This patient has the classic presentation of a third cranial nerve palsy. The third cranial nerve supplies the levator muscles of the eyelid and four extraoccular muscles: (1) the medial rectus (eye adduction), (2) superior rectus (eye elevation), (3)
inferior rectus (eye depression), (4) and inferior oblique (eye elevation).

Additionally, the third cranial nerve constricts the pupil through its parasympathetic fibers that supply the smooth muscles of the ciliary body and the sphincter of the iris. The third nerve begins as a nucleus in the midbrain that consists of several subnuclei innervating the individual extraoccular muscles, the eyelids, and the pupils.

The etiology of the third cranial nerve palsy is vast and includes conditions such as:

  • Ischemia (e.g. diabetes and midbrain infarcts)
  • Compressive effects of aneurysms such as those of the posterior communicating artery, internal carotid artery and basilar artery.
  • Trauma
  • Infections
  • Infiltrative diseases (e.g. neoplasms)
  • Demyelination
  • Space occupying tumors
  • Meningitis
  • Herniation
  • Inflammatory diseases

Of the given options, only a posterior communicating aneurysm can cause a third nerve palsy that involves the pupilary reflex. Compression of the third nerve by an enlarging intracranial aneurysm is the most dreaded etiology. The most common site of an aneurysm causing a third nerve palsy is the posterior communicating artery.

OPTION A : Midbrain infarcts can cause third cranial nerve palsy. However, since the etiology is ischemic (such as in diabetes) the pupillary light reflex is expected to be unaffected most of the time. Furthermore, with a midbrain infarct, an isolated third nerve palsy as the sole presenting symptom is very unlikely and other signs and symptoms related to a midbrain infarct are expected.

OPTION B : Right 6th nerve palsy presents with horizontal binocular diplopia upon looking laterally to the affected side. Furthermore, lesions of the sixth cranial nerve do not cause ptosis.

OPTION D : The fourth cranial nerve palsy presents with binocular vertical diplopia. Ptosis is not a feature.

OPTION E : Lesions of occipital cortex are associated with impaired vision and visual fields deficits, none of which are present here. Furthermore, ptosis is not a feature.

How well did you know this?
1
Not at all
2
3
4
5
Perfectly
32
Q

Which one of the following is the most likely sensory disturbance associated with the following x-ray?

A. Paresthesia over the deltoid.
B. Decreased sensation of the ulnar aspect of the forearm and hand.
C. Decreased strength of handgrip.
D. Decreased sensation over dorsum of the hand.
E. Decreased sensation over the 2/3 radial side of the palm.

A

D. Decreased sensation over dorsum of the hand

The X-ray shows a fracture in the mid-shaft of the left humerus. Radial nerve injury and palsy associated with fracture of the shaft of the humerus is the most common nerve injury complicating fractures of long bones. Rarely, median or ulnar nerves can be affected. Injury to the axillary nerve is seen in the fractures of humeral neck.

The primary motor function of the radial nerve is to innervate the muscles responsible for wrist and metacarpophalangeal (MCP) joints extensors, and abduction and extension of the thumb; therefore, proximal radial nerve injuries results in wrist drop.

Radial nerve also provides the sensation of dorsum of the hand and fingers at the 2/3 radial portion (thumb, index, middle and half of the ring fingers), the web space between the thumb and index fingers, most parts of the posterior aspect of the forearm. Depending on the anatomical site of the injury, sensory disturbances over these areas can be noted.

Of the options, decreased sensation over dorsum of the hand can be caused by radial nerve injury.

OPTION A : Paresthesia over the deltoid area is a sign of damage to the axillary nerve. Axillary nerve damage can be an associated injury in fractures of the humeral neck, not the shaft.

OPTION B : Decreased sensation over the ulnar aspect of the forearm and hand is associated with ulnar nerve injuries. Compared to the radial nerve injuries, ulnar nerve injuries are rare in humeral shaft fractures.

OPTION C : The strength of grip is the action of flexor muscles of the wrist and fingers. These muscles are innervated by median (mostly) and ulnar nerves.

OPTION E : Decreased sensation over the 2/3 radial side of the palm can be caused by injuries of the median nerve.

Dorsum of The Hand
How well did you know this?
1
Not at all
2
3
4
5
Perfectly
33
Q

Which one of the following could the most likely finding in a patient with mid-shaft humerus fractures?

A. Numbness of the little finger.
B. Inability to extend the thumb.
C. Numbness of the ventral aspect of the thumb.
D. Decreased grip strength.
E. Decreased wrist flexion.

A

B. Inability to extend the thumb

Radial nerve injury and palsy associated with fracture of the shaft of the humerus is the most common nerve injury complicating fractures of long bones. Rarely, median or ulnar nerves can be affected. Injury to the axillary nerve is seen in the fractures of humeral neck.

The primary motor function of the radial nerve is to innervate the muscles responsible for wrist and metacarpophalangeal (MCP) joints extensors, and abduction and extension of the thumb; therefore, proximal radial nerve injuries results in wrist drop or the decreased or absent thumb extension and abduction.

OPTION A : The sensation of the little finger and half of the ring fingers are supplied by terminal braches of the ulnar nerve which is very less like to be damaged in humeral mid-shaft fracture compared to the radial nerve.

OPTION C : The sensation of the ventral aspect of the thumb is provided by the terminal branches of the median nerve. Compared to the radial nerve, the median nerve is less likely to be affected in a mid-shaft humeral fracture.

OPTION D and E : The motor innervation of hand grip and wrist flexion is supplied by the median nerve.

How well did you know this?
1
Not at all
2
3
4
5
Perfectly
34
Q

A 71-year-old man, who has been diagnosed with bilateral osteoarthritis of his knees few years ago, presents to the GP clinic you work at with complaint of paresthesia and numbness of the lateral aspect of his right leg. The condition is worse at night and improves after walking for about 10 minutes. Which one of the following is the most important physical exam finding to consider in this patient?

A. Positive straight leg raise test.
B. Any tenderness over the right lateral condyle.
C. Any loss of sensation over the lateral side of the right leg.
D. Weakened dorsiflexion of the right foot.
E. Weak distal pulses of the right leg.

A

D. Weakened dorsiflexion of the right foot

The chief complaint of this man is paresthesia and numbness below the knee over the lateral side of the right leg. This presentation can be caused by common peroneal nerve injury (CPN). CPN is one of the two main nerves comprising the sciatic nerve. Along with the tibial nerve, CPN runs down from the buttock down the posterior part of the thigh to the apex of the popliteal fossa where it changes its path lateral to reach the knee. After wrapping around the fibular head, CPN enters the lateral compartment of the leg. There, this nerve divides in ‘deep peroneal’ and ‘superficial peroneal’ nerves. The proximal lateral aspect of the leg is innervated by CPN, while the distal part is innervated by superficial peroneal nerve.

(See photo below)

CPN through its branches supplies the muscles of ankle dorsiflexion, eversion and inversion in the anterior and lateral compartments of the leg. CPN nerve injury can cause weak ankle dorsiflexion (extension) resulting in foot drop and ankle and foot eversion, as well as toes extension.

This patient has presented with sensory deficits without any obvious motor weakness. He has osteoarthritis and bony processes (osteophytes) can compress the CPN in the lateral aspect of the knee, resulting in nerve irritation and damage. It should be noted that motor weakness, if subtle, might not be always appreciated by the patient. Therefore, a thorough physical examination is required to reveal any motor deficits that have remained unnoticed by the patient. In this case, weakened dorsiflexion of the right foot makes the diagnosis almost certain. Weakened dorsiflexion is the characteristic feature of CPN nerve injury as well as L5 radiculopathy.

OPTION A : Lesions of L5 nerve root and CPN nerve in the leg present similarly. However, these two can be differentiated by other historical and clinical findings. L5 radiculopathy is often associated with low back pain as a prominent feature. The pain radiates down from the lateral aspect of the thigh, is aggravated by walking (especially uphill) and bending, and is relived by recumbency and lying down. Pain relieved by walking and aggravated during sleep in unusual for L5 nerve root lesions. If in doubt, tests such as straight leg raise (SLR) or slump test can reproduce the symptoms in L5 radiculopathy, but not in CPN lesions.

OPTION B : While tenderness over the lateral condyle may be caused by compressing an injured CPN, it could also be just due to osteoarthritis itself, and is not a distinguishing feature.

OPTION C : With this presentation, loss of sensation over the lateral aspect of the leg is very likely as the main complaint is paresthesia and numbness of this area. Such finding adds no significant diagnostic benefit.

OPTION E : Weak distal pulses of the leg can be an important finding pointing towards vascular problems as a concomitant condition. It does not justify or adds any diagnostic clue to the presentation.

  • Medscape - Peroneal Mononeuropathy
35
Q

Mother of an 8-year-old girl has brought her for genetic testing for Huntington disease because the father has been recently diagnosed with the disease. She knows that the disease can have been passed to her daughter as well and she is very worries about this issue. Which one of the following is the most appropriate action in this situation?

A. Do not perform the genetic test.
B. Counsel the daughter about the disease and genetic testing. C. Perform genetic testing both for the mother and daughter.
D. Genetic counselling.
E. Perform the genetic test only for the daughter.

A

A. Do not perform the genetic test

Huntington disease is an autosomal dominant genetic disorder affecting approximately 1 in 15000 Australian population. The disease affects males and females equally and is found in all ethnic groups but it is among people of European descent.
The symptoms often starts between 30 and 50 years of age and can include:
-Involuntary muscle movements affecting the face, hands, limbs and body that are irregular, jerky and uncoordinated.
-Slurred speech and difficulties with swallowing.
-Disruptions to thought processes, including short term memory loss, difficulty with making decisions, lack of motivation, reduced insight and awareness of social boundaries.
-Unsteadiness and falls.
-Difficulty with emotional control, frustration, agitation and mood swings.
-Depression and anxiety.

HD progresses slowly. After developing the first symptoms, affected people usually live 15 to 20 years.

For HD, there is genetic testing that predicts whether a person with a family member affected with HD will develop the disease in the future.

According to Australian national guideline, in concordance with those of WHO, eligibility criteria for such test are:
1. The age of the individual is at least 18 years
AND
2. There is at least one blood-relative with definite diagnosis of HD in the pedigree.

This means no genetic testing for HD is performed on request of a person, or the carers (e.g. parents) of a person under the age of 18 years.
But the report added, “It seems appropriate and even essential, however, that the child be informed of his or her at-risk status upon reaching the age of reason.”
In the same year, a working party of the Clinical Genetics Society concluded that although discussion and counselling could and should be offered to minors, “formal genetic testing should generally wait until the ‘children’ request such tests for themselves, as autonomous adults. Testing should be waited either until the person affected is adult or “is able to appreciate not only the genetic facts of the matter but also the emotional and social consequences.”

The following summarizes the current guidelines for genetic testing for HD:
-No genetic testing is performed for children under 18 years upon parents’ (or other legal carers’) request
-Genetic counselling (different from testing) will not be performed for the child alone on parents’ request.
-No genetic testing will be performed upon the request of a consenting (competent) minor, but they can be offered genetic counselling (different from testing) provided that a belief is formed. On reasonable grounds, that the young person can handle the emotional and social consequences

Based on the guideline, genetic testing requested by the mother should not be performed for this girl until the age of 18. On the other hand, her very young age excludes her from genetic counselling (not testing) for her. She is too young to understand and handle the given information or handle the social and emotional ramifications.

*https://www.ncbi.nlm.nih.gov/pmc/articles/PMC1115457/
*http://www.genetics.edu.au/publications-and-resources/booklets-and-pamphlets/huntington-disease-and-genetic-testing * https://www.huntingtonswa.org.au/information/genetics/

36
Q

A 16-year-old girl comes to your GP clinic requesting genetic testing for Huntington disease. She is very distressed because her father was diagnoses with the disease two months ago. She surfed the internet about the disease and has known that the disease is inheritable and she may develop it later in life. She says that this is her decision and her parents do not know anything about her request. She does not want to inform them. Which one of the following is the most appropriate action in this situation?

A. Arrange the genetic test for her.
B. Tell her that she needs to come back for the test when she is 18years old.
C. Arrange for the whole family to have genetic counselling.
D. Tell her parents’ consent is required for genetic testing.
E. Seeking a court order before proceeding to the test.

A

B. Tell her that she needs to come back for the test when she is 18years old

Current national Australian guidelines, in agreement with WHO guidelines, recommend predictive genetic testing for Huntington disease (HD) only for people aged 18 years or older who have at least one blood relative with definite diagnosis of HD.

In cases where a consenting minor fulfilling the criteria for Gillick competence rule requests for such testing, the rule still holds true; however, genetic testing for HD will not be performed for such minor, even if he/she is independent and seems to understand, and can handle the social and emotional consequences of a positive test result for HD and the fact that he/she will eventually develop the disease and probably die of it. In such instances, the child is needed to be informed that he/she can request such a test at the age of 18 years or older. However, the child can be offered referral to an accredited genetic unit for genetic counselling (not testing), provided that the clinician, based on reasonable ground, form a belief that this minor can handle the situation.

This girl should be told that she cannot have the genetic test until she is 18 years old.

OPTION A : Arrangement for the genetic test for her is incorrect and she can have the test at the age of 18 years or older.

OPTION C : Arrangement for offspring of this family is only performed upon their request provided that they fulfill the criteria of predictive genetic testing for HD.

OPTION D : Parents cannot consent to or request for HD predictive genetic testing on the minors’ behalves. Once the minor are 18 years no parental consent for testing is required.

OPTION E : A competent adult can request for the test. A court order is required where guidelines are unclear for decision making.

37
Q

An eight-year-old boy is brought to your practice by his mother for what she calls funny turns in the past several weeks. According to the mother, the boy have episodes of sudden stares along with fidgeting of the right hand fingers and right arm, lip smacking and chewing movements while his head twitches to the right side. He has 3 to 4 episodes every day with each lasting 60-90 seconds. There is always spontaneous resolution after which the child is confused for 1 to 2 minutes before he becomes completely himself and back to whatever he was doing before. Which one of the following is the most likely diagnosis?

A. Temporal lobe epilepsy.
B. Juvenile myoclonic epilepsy.
C. Absence seizure.
D. Hypsarrhythmia.
E. Rolandic epilepsy.

A

A. Temporal lobe epilepsy

The clinical picture in this child is highly suggestive of temporal lobe epilepsy as the most likely diagnosis. Temporal lobe epilepsy is categorized under complex partial seizures meaning that the seizure has a particular focus in the brain (partial) and the awareness is impaired during an attack (complex).

Temporal lobe epilepsy is characterized by automatism including fidgeting, chewing, lip smacking, fumbling, or complex acts such as speed driving, kissing, violence or other bizarre behaviors. The duration of an attack is often 1-3 minutes. There is a postictal state of 1 to 2 minutes associated with confusion. The patient has no recollection of the events during the attack.

Other features include memory disturbances such as deja vu or jamais vu, hallucinations, emotional disturbances such as a sense of sudden terror, panic, anger or drealization. Ictal or post-ictal aphasia can be a feature. Abdominal rising sensation (abdominal aura) with or without nausea and vomiting may occur. Patients may have bizarre delusions.

OPTION B : Juvenile myoclonic epilepsy, also known of myoclonic epilepsy of Janz, is the triad of: (1) myoclonic jerks, (2) tonic-clonic seizures, and (3) absence seizures. Myoclonic and tonic-clonic seizures often occur early in the morning after waking. Typically the onset is around puberty but it may develop earlier in life. The condition is often life-long and responds well to sodium valproate. There is no cognitive impairment.

OPTION C : Absence seizure, formerly known as petit mal seizure, is characterized by often over a hundred daily attacks of sudden onset alteration of awareness and blank stare. The common age for absence seizure is 5 to 7 years. There might be automaticity of face and other body parts. Ethosuximide is the first-line therapy.

OPTION D : Hypsarrhythmia, also known as infantile spasms or West syndrome, are generalized tonic seizures featured by sudden flexion of the arms, forward flexion of the trunk and extension of the legs lasting only a few seconds. The age of onset is often between 3 and 7 months and commonly restricted to the first three years of life before being replaced by other types of seizures later on. There are different cognitive outcomes but the prognosis is often poor. First-line treatment is with corticotropin (ACTH) intramuscularly. Prednisolone, vigabatrin, sodium valproate and benzodiazepines has been tried as second-line therapies. The age of the child and the seizure features are not consistent with such diagnosis.

OPTION E : Benign Rolandic epilepsy is a simple partial seizure often associated with a family history of epilepsy. It affects children between 2 and 13 years with a peak age of 5 to 8 years. There are motor or somatosensory disturbances involving the mouth and face during sleep and a characteristic ‘glugging’ sound (the sound a liquid pouring from a bottle). The child often wakes up and goes to the parents with a twisted face and inability to speak. Progression to tonic-clonic seizures is likely but there is often a good prognosis and remission occurs around the puberty. Carbamazepine is the first-line medication for treatment.

Drug of choice: Carbamazapine

38
Q

A 68-year-old man presents with a history of headache for the past several months. The headache is worse on waking up, coughing, sneezing and bending forward. He also mentions that he has been suffering from cough for the past 3 months. His history is significant for smoking of 45 cigarettes a day for the past 45 years. Significant finding on examination is left hemiparesis. A chest X-ray reveals an opacity of 3x2 cm in size in the left lung field. A contrast CT scan of the head is obtained which is shown in the following picture. Which one of the following could be the most likely diagnosis?

A. Primary brain tumor.
B. Metastatic brain tumor.
C. Brain abscess.
D. Cerebral hemorrhage.
E. Ischemic stroke.

A

B. Metastatic brain tumor

The clinical picture of headache worse in the morning, and by sneezing, coughing and bending forward is suggestive of a space-occupying lesion in the brain that can be either a tumor or abscess. Considering the age of the patient, history of heavy smoking and the cough, this patient has lung cancer with high certainty.

Lung cancer is one of the three primary cancers that tend to metastasize to the brain (21%). The other two are melanomas (40%) and breast cancer (9%).
With this history and chest X-ray findings, the lesion is most likely to be a metastatic brain lesion from a primary lung cancer.
Metastatic brain tumors are the most common form of brain tumors. Of all brain tumors, more than a half are metastatic.

OPTION A : With the high probability of an underlying lung cancer in this patient, this tumor is more likely to be metastatic than primary.

OPTION C : Brain abscess has similar appearance on contrast CT scan (ring enhancement); however, with absence of fever and considering the history, a brain abscess is not a likely diagnosis.

OPTION D and E : The appearance of the lesion as well as the insidious onset and progression of the symptoms make cerebral hemorrhage and ischemic stroke unlikely diagnoses.

39
Q

A 65-year-old man is brought to your clinic for evaluation after his carer noticed that his movements have been progressively slowing down for the past few weeks. He has been on chlorpromazine for the past 8 weeks for treatment of psychotic features he developed after his wife passed away. Which one of the following options would suggest a diagnosis of drug induced Parkinsonism rather than Parkinson’s disease?

A. Restlessness of the arms and legs.
B. Bilaterality of the symptoms.
C. Rigidity.
D. Masked face.
E. Bradykinesia.

A

B. Bilaterality of the symptoms

Drug-induced Parkinsonism (DIP) is the second-most-common etiology of Parkinsonism in the elderly after Parkinson’s disease (PD). Many patients with DIP may be misdiagnosed with PD because the clinical features of these two conditions are indistinguishable.

Typical antipsychotics such as phenothiazines (e.g. chlorpromazine) and haloperidol are the leading cause of DIP. Metoclopramide is another drug frequently associated with DIP. Other medications with potential ability to cause DIP include calcium channel blockers, atypical antipsychotics, and antiepileptic drugs.

The clinical manifestations of DIP are classically described as bilateral and symmetric Parkinsonism without tremor at rest.

OPTION C, D and E : Rigidity , masked face and bradykinesia/hypokinesia are shared features of both PID and Parkinson’s disease. Restlessness of arms and legs is not seen in any of the conditions.

Parkinsonism is defined as the triad of: (1) bradykinesia/hypokinesia, (2) rigidity, and (3) tremors. Atypical

TOPIC REVIEW

Parkinsonism is defined as syndromes characterized by some features of Parkinsonism PLUS additional features (Parkinson plus).

Early clinical features that suggest an atypical Parkinsonism rather than Parkinson’s disease include the following:

  • Falls at presentation or early in the disease
  • Poor response to levodopa
  • Symmetry at disease onset
  • Rapid disease progression
  • No tremor
  • Dysautonomia (e.g., urinary incontinence, fecal incontinence, catheterization for urinary retention, persistent erectile failure, prominent symptomatic orthostatic hypotension)

The atypical parkinsonisms are usually associated with little or no tremor, relatively early speech and balance difficulty, and little or no response to dopaminergic medications.

Parkinson plus syndromes include:

  • Progressive supranuclear palsy (PSP) - early falls and postural instability; vertical gaze paralysis; rigidity of trunk > limbs; symmetrical onset; speech and swallowing problems; tremor is unusual
  • Multiple system atrophy (MSA, Shy-Dragger syndrome) - early autonomic features (postural hypotension, bladder dysfunction; cerebellar and pyramidal signs; rigidity > tremor
  • Cortico-basal degeneration (CBD) - akinetic rigidity involving one limb; cortical sensory loss (e.g. astereognosis); apraxia (in the extreme there may be autonomous interfering activity of affected limb – the ‘alien limb’ phenomenon)
  • Lewy body dementia - early dementia with fluctuating cognition and visual hallucinations.
  • Vascular Parkinsonism -Parkinsonism worse in legs than arms; pyramidal signs; prominent gait abnormality.
40
Q

Which one of the following, if present, is in favor of Parkinson’s disease rather than drug induced Parkinsonism?

A. Tremors.
B. Rigidity.
C. Symmetrical symptoms.
D. Masked face.
E. Bradykinesia.

A

A. Tremors

Drug-induced Parkinsonism (DIP) is the second-most-common etiology of Parkinsonism in the elderly after Parkinson’s disease (PD). Many patients with DIP may be misdiagnosed with PD because the clinical features of these two conditions are indistinguishable.

Typical antipsychotics such as phenothiazines (e.g., chlorpromazine) and haloperidol are the leading cause of DIP. Metoclopramide is another drug frequently associated with DIP. Other medications with potential ability to cause DIP include calcium channel blockers, atypical antipsychotics, and antiepileptic drugs.

The clinical manifestations of DIP are classically described as bilateral and symmetric Parkinsonism without tremor at rest. Although approximately 50% of DIP patients show asymmetrical Parkinsonism and tremor at rest, bilaterality of symptoms is in favor of DIP rather than Parkinson’s disease.

Of the option, presence of tremors is more (not definitely) suggestive of Parkinson’s disease rather than DIP.

OPTION B, D, E : Rigidity, masked face, and bradykinesia/hypokinesia are shared features in both PID and Parkinson’s disease.

OPTION C : Symmetrical symptoms is in favor of DIP rather than Parkinson’s disease.

41
Q

A 12-year-old boy is brought to your office after accidentally cutting his left wrist with a pocketknife. On examination, there is a deep 2-cm laceration at the base of the thenar eminence. To test the motor injury to the median nerve you would have the patient:

A. Extend the thumb and fingers.
B. Oppose the thumb and little finger.
C. Flex the wrist.
D. Abduct the thumb and index finger.
E. Adduct the thumb and index finger.

A

B. Oppose the thumb and little finger

The scenario describes a cut in the thenar area of this child.

There are three muscles in the thenar eminence:
1. abductor pollicis brevis
2. flexor pollicis brevis
3. opponens pollicis.

Median nerve injury at wrist level results in the dysfunction of these three muscles (as well as the lumbrical muscles of the digits 2 and 3).

The first two muscles are conjoined with fibers from their counterparts originating from forearm, namely abductor pollicis longus and flexor pollicis longus. While median nerve injury at wrist level causes the dysfunction of thenar region muscles, thumb abduction and flexion remain preserved due to compensatory action of abductor pollicis longus and flexor pollicis longus muscles. However, thumb opposition becomes severely impaired due to denervation of the opponens pollicis muscle. Isolated impairment of thumb opposition is the most likely finding in median nerve injuries at wrist level.

OPTION A : Extension of the thumb and fingers is an action of radial nerve and is not affected by median nerve injuries.

OPTION C : Wrist flexion is mainly the action of flexor carpi radialis and flexor carpi ulnaris muscles, innervated by the median and ulnar nerves, respectively, in the forearm. Median and ulnar nerve injuries in the wrist will not affect wrist flexion; therefore, this movement will be unaffected in this patient.

OPTION D : While median nerve injury at wrist can impair the function of abductor pollicis brevis in the thenar eminence, thumb abduction is preserved to a great extend, because the fibers of abductor pollicis longus are innervated in the forearm by the anterior interosseous nerve (a branch of median nerve) and unaffected by injury to the median nerve at wrist level. Therefore, this action is preserved. Abduction of other fingers than the thumb are provided by dorsal interosseous muscles and abductor digiti minimi, all supplied by the ulnar nerve; hence unaffected in this patient.

OPTION E : Thumb adduction is mainly provided by the muscle adductor pollicis that is innervated by the deep branch of the ulnar nerve. Median nerve injury at wrist level is unlikely to significantly affect this movement. Adduction of the other fingers other than the thumb are the action of palmar interosseous muscles all innervated by ulnar nerve and is not expected to be impaired in this patient.

42
Q

A 35-year old woman presents to your practice with complaint of decreased ability to hold small objects between his right thumb and index finger. He noticed it few months ago when he was unable to properly hold the door or car keys. Which one of the following conditions could have led to this presentation?

A. Radial nerve injury.
B. Carpal tunnel syndrome.
C. Anterior interosseous nerve injury.
D. Ulnar nerve injury.
E. Posterior interosseous nerve injury.

A

D. Ulnar nerve injury

Types of Pinching:

  1. Tip-to-Tip Pinch:
    • Thumb tip touches the tip of the index or middle finger.
    • Used for holding small objects.
  2. Pad-to-Pad Pinch:
    • Pad of the thumb touches the pad of the index or middle finger.
    • Used for holding larger objects needing more stability.
  3. Three-Jaw-Chuck Pinch:
    • Thumb touches the tips of the index and middle fingers.
    • Used for holding larger or heavier objects, like a piece of chalk or a pencil.
  4. Lateral/Key Pinch:
    • Thumb pulp touches the side of the index finger.
    • Used for actions like turning a key in a lock.

Key Muscles in Pinching:
- Adductor Pollicis and Dorsal Interosseous Muscle of the index finger.
- Both are supplied by the ulnar nerve and provide strength for pinching.

Common Nerve Injuries Affecting Pinching:

  1. Ulnar Nerve Injury:
    • Symptoms: Difficulty with pinch strength, especially with the lateral/key pinch.
    • Froment’s Sign: When asked to hold a piece of paper, the thumb flexes at the distal joint to compensate for weak adductor pollicis, creating a “clamp” grip instead of the normal “tongs” grip.
  2. Radial Nerve Injury:
    • Symptoms: Affects the extensor muscles of the forearm, wrist, and fingers.
    • Note: Does not cause weakness in pinching between the thumb and index finger.
  3. Anterior Interosseous Nerve (AIN) Syndrome:
    • Symptoms: Inability to make an “O.K.” sign due to impaired flexion of the thumb and index finger.
    • Pinch Test: Cannot pinch a sheet of paper properly, resulting in a “tongs” appearance instead of a “clamp.”
    • Affected Muscles: Flexor pollicis longus (thumb), flexor digitorum profundus (index/middle fingers), and pronator quadratus.
  4. Carpal Tunnel Syndrome:
    • Symptoms: Sensory disturbances in the palmar side of the thumb, index finger, middle finger, and half of the ring finger, and weakness of the thenar muscles.
    • Note: While thumb opposition is affected, pinching between thumb and index finger is not significantly impaired since key muscles (flexor pollicis longus and flexor digitorum profundus) are innervated by AIN, which is not affected by carpal tunnel syndrome.
  • Ulnar Nerve Injury: Affects pinch strength, particularly the lateral/key pinch, with signs like Froment’s sign.
  • Radial Nerve Injury: Does not significantly impact pinching ability.
  • AIN Syndrome: Impairs the ability to make an “O.K.” sign and perform effective pinching, but does not cause sensory symptoms.
  • Carpal Tunnel Syndrome: Affects sensation and thumb opposition but does not majorly impair pinching.

Understanding these nerve injuries and their impact on pinching helps in diagnosing and managing conditions effectively.

In order to understand clearly the nerve injury associated with this presentation, it is essentinal to know the pinching and its types.

Four types of pinching have been described:
(PICTURES OF TYPES PINCHES in PAGE 762-764)

  1. Tip-to-tip pinch
    Tip to tip pinch (pulp of the thumb against that of the index/middle finger) – this type of pinch is used for holding and positioning of small objects
  2. Pad-to-pad pinch
    Pad to pad pinch (pad of the thumb against index / middle finger) – this type of pinch is used for larger objects or objects that need more stabilization
  3. Three-jaw-Chalk pinch
    Three jaw chalk pinch (thumb against tips of index and middle fingers) – it is used for larger or heavier objects e.g., holding a piece of chalk while writing on the blackboard or holding a pencil
  4. Lateral/key pinch
    Lateral / key pinch (pulp of thumb against radial/lateral aspect of the index finger) – object is aligned with forearm and with supination/pronation the object is moved through the space (using a key to open a door)

Different muscles work together to provide precise alignment and power of different types of pinch (pincer grip); however, the two key muscles providing the strength of a grip are adductor pollicis and dorsal interosseous muscle of the index finger, both of which are supplied by the ulnar nerve. With decreased power of pinching, an ulnar nerve injury is more likely to be the underlying cause of this presentation.

If a patient with ulnar nerve injury is asked to grasp a piece of paper between the thumb and lateral aspect of the index finger, loss of power to adductor pollicis muscle will cause flexion of the distal thumb as the flexor pollicis longus compensates. Patient makes a “clamp” instead of the naturally-occurring “tongs” to hold the paper (Froment’s sign).

OPTION A - Radial nerve and its branch - the posterior interosseous supply the extensor muscles of the forearm, wrist and fingers. Radial nerve lesions do not cause weakness of pinching between the thumb and index finger.

OPTION C - For tip-to-tip approximation during pinching between the thumb and index finger (thumb-index finger pincer grip) the two key muscles involved are flexor pollicis longus (flexing the interphalangeal joint of the thumb) and flexor digitorum profundus (flexing the distal interphalangeal joint of the index finger). Both flexor pollicis longus and the flexor digitorum profundus muscles of fingers II and III are innervated by anterior interosseous nerve (AIN), a motor branch of the median nerve. Patients with AIN lesions typically fail to make an “O.K.” sign, because flexion of the interphalangeal joint of the thumb and the distal interphalangeal joint of the index finger are impaired. Another sensitive test is the pinch test: a patient with AIN lesions will also not be able to pinch a sheet of paper between his thumb and index finger, instead they hold the sheet between his extended thumb and index fingers, giving an appearance similar to a “tong” rather than a “clamp”. The anterior interosseous nerve, however, does not contribute as significantly to the power of the pincer grip as do adductor pollicis and dorsal interosseous muscle.

NOTE - Anterior interosseous nerve innervates the following muscles:
a) Flexor pollicis longus – main flexor of the thumb
b) Flexor digitorum profundus of digits II and III (index and middle fingers) – flexor of the distal interphalangeal joints of forefinger and middle finger
c) Pronator quadratus – along with the pronator teres, and while the elbow is flexed to a right angle, turns the hand so that the palm of the hand faces the floor.

OPTION B and E : Carpal tunnel syndrome is caused by compression of the median nerve in the carpal tunnel, and presents with sensory disturbances in palmar aspect of the thumb, index finger, middle finger and half of the ring (4th) finger. It also causes the weakness of the thenar muscles the abductor pollicis brevis, flexor pollicis brevis and opponens pollicis. While thumb opposition can be significantly affected in patients with carpal tunnel syndrome, pinching between thumb and index fingers are NOT markedly impaired because the key muscles to the action, which are flexor pollicis longus and flexor digitorum profundus muscle of the index finger are supplied by AIN that does not run in the carpal tunnel; therefore, unaffected in carpal tunnel syndrome.

43
Q

A 27-year-old woman, typist by profession, presents to your practice with complaint of right wrist pain and numbness of the index, middle and radial half of the ring fingers. On examination, she is unable to appropriately do the opposition of the thumb and little fingers on the right side. Which one of the following is the most likely diagnosis?

A. Ulnar nerve lesion.
B. Radial nerve lesion.
C. Carpal tunnel syndrome.
D. Anterior interosseous nerve syndrome.
E. Tendonitis.

A

C. Carpal tunnel syndrome

What is CTS?
Carpal Tunnel Syndrome is a condition where the median nerve, which runs through the carpal tunnel in the wrist, gets compressed. This causes various symptoms in the hand.

Symptoms:
- Pain and Paresthesia: Pain and numbness/tingling in the thumb, index finger, middle finger, and half of the ring finger.
- Night Symptoms: Symptoms are often worse at night, causing people to wake up. Many find relief by shaking their hands, wringing them, or running them under warm water.
- Activity-Related Symptoms: Activities like typing, reading, driving, and holding a phone can make symptoms worse.
- Radiating Pain: Sometimes, pain and numbness can spread to the wrist, entire hand, forearm, or even up to the shoulder.
- Motor Symptoms: In severe cases, weakness in the hand can occur, making it hard to hold objects, turn keys, open jars, or button clothes.
- Thenar Muscle Weakness: The muscles at the base of the thumb can become weak.
- Late-Stage Sensory Loss: Sensory loss usually affects the fingers but not the palm, as the palm is supplied by a different branch of the median nerve that doesn’t pass through the carpal tunnel.

Differential Diagnoses:
- Ulnar Nerve Lesion (Option A): Causes sensory issues in the little finger and half of the ring finger, and affects finger abduction/adduction.
- Radial Nerve Lesion (Option B): Affects sensation on the back of the thumb, index, and middle fingers, but not thumb opposition.
- Anterior Interosseous Nerve (AIN) Syndrome (Option D): Causes forearm pain and difficulty making an “O.K.” sign with the thumb and index finger. It affects motor function but not sensation.
- Tendonitis (Option E): Causes pain but no sensory disturbances.

  • Primary Symptom: Pain and tingling in the first three fingers and half of the fourth finger.
  • Exacerbating Factors: Activities involving wrist movement.
  • Severe Cases: Muscle weakness and difficulty with hand movements.

CTS is typically managed with lifestyle changes, wrist splints, anti-inflammatory medications, and in some cases, surgery to relieve pressure on the median nerve. Understanding these basics can help recognize CTS and differentiate it from other conditions with similar symptoms.

The clinical presentation is highly suggestive of carpal tunnel syndrome (CTS) as the most likely diagnosis.
CTS refers to a constellation of signs and symptoms caused by compression of the median nerve as it travels through the carpal tunnel. CTS presents with pain and paresthesia, and less commonly weakness in the median nerve distribution in hand.

Classic CTS is associated with pain or paresthesia (numbness and tingling) in the distribution of the median nerve territory in the hand (the first three digits and the radial half of the fourth digit.) Symptoms are typically worse at night and often awaken patients. The patients often shake or wring their hands or place them under warm running water to alleviate the symptoms. Symptoms of CTS are usually limited to fingers innervated by the median nerve; however, the pain and paresthesia may be localized to the wrist or involve the entire hand. It is even possible that the sensory symptoms radiate proximally into the forearm, or less frequently to above the elbow or even the shoulder.

CTS symptoms often worsen with activities involving wrist flexion/extension or arm raising. Some of such activities include typing, reading, driving, and holding a telephone.

In more severe cases, motor involvement follows and results in weakness or clumsiness when using the hands, such as difficulty holding objects, turning keys or doorknobs, buttoning, or opening jar lids. Clinical signs may include weakness of thumb abduction and opposition, and atrophy of the thenar eminence.
Fixed sensory loss is usually a late finding characterized by a distinctive clinical pattern characterized by involvement of the fingers supplied by the median nerve, while the thenar eminence is spared. This pattern occurs because the sensation of the palm is supplied by a sensory cutaneous nerve coming off the median nerve proximal to the wrist. This nerve over the carpal tunnel, not through it.

OPTION A : Ulnar nerve lesions result in sensory disturbances in the sensory territory of the ulnar nerve. In hand, the ulnar nerve provides sensation of the little finger and 1⁄2 of the ring finger, as well as ulnar parts of the palm and dorsum of the hand. Motor involvement results in decreased ability to abduct/adduct fingers (an action of interosseous muscles) as well as weak pincer grip.

OPTION B : Radial nerve lesions, presents with disturbed sensation over the dorsal aspect of the thumb, index finger, middle finger and radial 1⁄2 half of the ring finger. Thumb opposition is provided by the action of opponens pollicis that is innervated by median nerve and is not affected in radial nerve injuries.

OPTION D : Anterior interosseous nerve (AIN) syndrome is a rare condition associated with entrapment of the anterior interosseous nerve. AIN syndrome presents with forearm pain, inability to make an ‘O.K. sign’ with the thumb and index finger, and positive pinch test in which the patient pinches in between the thumb and index finger in a fashion similar to tongs rather than clamps. AIN is a pure motor nerve branching off the median nerve; therefore, its injuries do not cause sensory symptoms.

OPTION E : Tendonitis is not associated with sensory disturbances.

44
Q

Lesions of which of the following nerves can lead to wrist drop?

A. Median nerve at the wrist.
B. Median nerve at the elbow.
C. Radial nerve at the wrist.
D. Radial nerve at the elbow.
E. Ulnar nerve at elbow.

A

**D. Radial nerve at the elbow*

Wrist extension primarily involves the following muscles:
- Extensor Carpi Radialis Longus
- Extensor Carpi Radialis Brevis
- Extensor Carpi Ulnaris
- Extensor Digitorum

These muscles originate around the lateral epicondyle of the humerus and are innervated by the radial nerve near the elbow. Therefore, an injury to the radial nerve at the elbow can lead to wrist drop.

Option A: Median Nerve Injury at the Wrist
- Affects the thenar muscles: flexor pollicis brevis, abductor pollicis brevis, and opponens pollicis.
- Does not affect wrist extension.

Option B: Median Nerve Injury at the Elbow
- Affects muscles innervated by the median nerve and its motor branch, the anterior interosseous nerve.
- Includes most forearm flexors and thenar muscles, plus the 1st and 2nd lumbricals.
- Does not affect wrist extension.

Option C: Radial Nerve Injury at the Wrist
- Only causes sensory deficits (dorsal thumb, index and middle fingers, radial half of ring finger, dorsal aspect of first web space).
- No motor function impairment.

Option E: Ulnar Nerve Injury at the Elbow
- Affects flexion of the 4th and 5th fingers and wrist flexion (flexor carpi ulnaris muscle).
- Results in radial deviation of the wrist when attempting flexion.
- Affects the 3rd and 4th lumbricals, interosseous muscles, and hypothenar muscles.
- Sensory deficits in the ulnar nerve’s territory.
- Does not affect wrist extensors.

  • Radial nerve injury at the elbow is the main cause of wrist drop due to its role in wrist extensor muscle innervation.
  • Other nerve injuries (median, ulnar) do not affect wrist extension and lead to different deficits.

Wrist extension is mainly the action of extensor carpi radialis longus, extensor carpi radialis brevis, extensor carpi ulnaris, and extensor digitorum, all of which originating from around the lateral epicondyle of the humerus and innervated in the vicinity of the elbow. Therefore, radial nerve injury at the level of the elbow can be associated with wrist drop.

OPTION A : Median nerve injury at the wrist causes the weakness of thenar eminence muscles i.e. flexor pollicis brevis, abductor pollicis brevis and opponens pollicis. These muscles are not involved in wrist extension.

OPTION B : Median nerve injury at elbow results in dysfunction of all the muscles innervated by the median nerve and its motor branch the anterior interosseous nerve. These muscles include all muscles in the anterior compartment of the forearm (except flexor carpi ulnaris that is innervated by the ulnar nerve) as well as muscles of thenar eminence and the 1st and 2nd lumbrical muscles. Wrist extension is not affected by median nerve injuries at elbow.

OPTION C : Radial nerve injury at wrist is not associated with any motor function impairment and only causes sensory deficit in the territory or radial nerve that includes the dorsal aspect of the thumb, index and middle fingers, the radial half of the ring finger, and the dorsal aspect of the first web space.

OPTION E : Ulnar nerve injury at elbow affects the flexion of fingers 4th and 5th, wrist flexion (paralysis of the flexor carpi ulnaris muscle and radial deviation when flexion is attempted), the 3rd and 4th lumbrical muscles, interosseous muscles of the hand and muscles in the hypothenar eminence. Sensory deficits in the territory of the ulnar nerve can be another physical finding. None of the wrist extensor muscles are innervated by the ulnar nerve and remain unaffected by ulnar nerve injuries.

45
Q

Which one of the following can cause weakness in small muscles of the hand while sparing the thenar eminence?

A. Ulnar nerve injury.
B. Median nerve injury.
C. Radial nerve injury.
D. Anterior interosseous nerve injury.
E. Posterior interosseous nerve injury.

A

A. Ulnar nerve injury

Ulnar Nerve:
- Innervates: All interosseous muscles, 3rd and 4th lumbricals, muscles in hypothenar eminence.
- Injury: Causes weakness in these muscles, but thenar eminence is spared (supplied by the median nerve).

Other Nerves:

  1. Median Nerve (Low/Distal Injury):
    • Affects: Thenar muscles (flexor pollicis brevis, abductor pollicis brevis, opponens pollicis).
  2. Radial Nerve:
    • Supplies: Muscles in the posterior forearm, no small hand muscles, thenar, or hypothenar.
  3. Anterior Interosseous Nerve (branch of median nerve):
    • Supplies: Flexor pollicis longus, flexor digitorum profundus (2nd and 3rd fingers), pronator quadratus (all in forearm).
  4. Posterior Interosseous Nerve (branch of radial nerve):
    • Supplies: Muscles from the posterior forearm compartment, no small hand muscles, thenar, or hypothenar.

A 45-year-old woman has weakness in her hand muscles, sparing the thenar eminence.

  1. What is the most likely nerve injury?
    • A) Median nerve (low)
    • B) Ulnar nerve
    • C) Radial nerve
    • D) Anterior interosseous nerve
    • E) Posterior interosseous nerve
    Answer: B) Ulnar nerveExplanation: Ulnar nerve injury affects most intrinsic hand muscles but spares the thenar eminence (supplied by the median nerve).
    • Why not A): Median nerve injuries affect thenar muscles.
    • Why not C, D, E): These nerves do not supply the intrinsic hand muscles.

A patient presents with weakness in the thenar muscles (flexor pollicis brevis, abductor pollicis brevis, opponens pollicis).

  1. Which nerve is most likely affected?
    • A) Ulnar nerve
    • B) Radial nerve
    • C) Median nerve (low)
    • D) Anterior interosseous nerve
    • E) Posterior interosseous nerve
    Answer: C) Median nerve (low)Explanation: Low median nerve injury affects thenar muscles.
    • Why not A): Ulnar nerve injury spares the thenar eminence.
    • Why not B, D, E): These nerves do not innervate thenar muscles.

A patient has an injury affecting the posterior compartment of the forearm muscles.

  1. Which nerve is most likely injured?
    • A) Ulnar nerve
    • B) Radial nerve
    • C) Median nerve
    • D) Anterior interosseous nerve
    • E) Posterior interosseous nerve
    Answer: B) Radial nerveExplanation: The radial nerve supplies the muscles in the posterior compartment of the forearm.
    • Why not A, C, D, E): These nerves do not primarily innervate the posterior forearm muscles.

A patient has difficulty flexing the distal phalanges of the 2nd and 3rd fingers.

  1. Which nerve is most likely affected?
    • A) Ulnar nerve
    • B) Radial nerve
    • C) Median nerve (high)
    • D) Anterior interosseous nerve
    • E) Posterior interosseous nerve
    Answer: D) Anterior interosseous nerveExplanation: The anterior interosseous nerve innervates the flexor pollicis longus and flexor digitorum profundus for the 2nd and 3rd fingers.
    • Why not A, B, C, E): These nerves do not specifically innervate the flexor digitorum profundus for the 2nd and 3rd fingers.

Of the options, ulnar nerve injury can cause weakness of most intrinsic hand muscles while sparing the thenar eminence.
In hand, ulnar nerve innervates all interosseous muscles, the 3rd and 4th lumbrical muscles, and muscles in the hypothenar eminence. While ulnar nerve injuries affect the function of these muscles, thenar eminence muscles are spared because they are supplied by the median nerve.

OPTION B : Low (distal) median nerve injuries affect the thenar muscles flexor pollicis brevis, abductor pollicis brevis and opponens pollicis.

OPTION C : Radial nerve supplies the muscles in the posterior compartment of the forearm, but no small muscle in the hand, thenar, or hypothenar eminence.

OPTION D : Anterior interosseous nerve is a purely motor branch of the median nerve and supplies the muscles flexor pollicis longus, flexor digitorum profundus (2nd and 3rd fingers), and pronator quadratus, all of which originating from the forearm, not the hand.

OPTION E : Posterior interosseous nerve (PIN) is a branch of the radial nerve that supplies the muscles originating from the posterior compartment of forearm. PIN injuries do not affect the function of any of the small hand muscles, thenar, or hypothenar eminence.

46
Q

Which one of the following nerves supplies the interosseous muscles of the hand?

A. Radial nerve.
B. Median nerve.
C. Ulnar nerve.
D. Anterior interosseous nerve.
E. Posterior interosseous nerve.

A

C. Ulnar nerve

Interosseous muscles are intrinsic hand muscles that abduct (dorsal interosseous muscles) and adduct (palmar interosseous muscles) the fingers. These muscles are innervated by deep branch of the ulnar nerve.

  • Orthobullets - Dorsal interossei
  • Orthobullets - Palmar interossei
47
Q

After a left elbow injury, a 55-year-old man develops weakness of the long flexors of the thumb and index finger. He is also unable to do forearm pronation on the same side. Which one of the nerves is involved?

A. Radial nerve.
B. Median nerve.
C. Anterior interosseous nerve.
D. Posterior interosseous nerve.
E. Ulnar nerve.

A

C. Anterior interosseous nerve

  • AIN: Purely motor branch of the median nerve.
  • Muscles Innervated by AIN:
    • Flexor pollicis longus (thumb flexor)
    • Radial part of flexor digitorum profundus (index and middle finger flexor)
    • Pronator quadratus (forearm pronation)
  • Weakness in thumb flexion
  • Weakness in index finger flexion
  • Difficulty in forearm pronation
  • Radial Nerve Injury: Affects thumb and finger extensors, forearm supination.
  • Distal Median Nerve Injury: Affects thenar muscles (thumb opposition).
  • Posterior Interosseous Nerve Injury: Affects finger extension.
  • Ulnar Nerve Injury: Affects 4th and 5th finger flexion, wrist flexion.

A 30-year-old male presents with weakness in thumb and index finger flexion after a forearm injury.

  1. Which nerve is most likely injured?
    • A) Radial nerve
    • B) Distal median nerve
    • C) Anterior interosseous nerve
    • D) Posterior interosseous nerve
    • E) Ulnar nerve
    Answer: C) Anterior interosseous nerveExplanation: Weakness in thumb and index finger flexion suggests AIN injury.
    - Why not A): Radial nerve affects extension, not flexion.
    - Why not B): Distal median nerve affects thenar muscles.
    - Why not D): Posterior interosseous nerve affects finger extension.
    - Why not E): Ulnar nerve affects 4th and 5th finger flexion.

A patient presents with suspected AIN injury.

  1. Which clinical test would confirm the diagnosis?
    • A) Froment’s sign
    • B) Pinch grip test
    • C) Tinel’s sign
    • D) Phalen’s test
    • E) Finkelstein’s test
    Answer: B) Pinch grip testExplanation: Pinch grip test evaluates the function of flexor pollicis longus and flexor digitorum profundus.
    - Why not A): Froment’s sign tests ulnar nerve function.
    - Why not C): Tinel’s sign tests for carpal tunnel syndrome.
    - Why not D): Phalen’s test is for carpal tunnel syndrome.
    - Why not E): Finkelstein’s test is for de Quervain’s tenosynovitis.

A patient with suspected AIN injury needs imaging.

  1. What is the first imaging study to order?
    • A) MRI of the forearm
    • B) X-ray of the forearm
    • C) Ultrasound of the forearm
    • D) CT scan of the forearm
    • E) Bone scan
    Answer: B) X-ray of the forearmExplanation: X-ray helps rule out fractures or bony abnormalities that might be compressing the nerve.
    - Why not A): MRI is more detailed but not the first step.
    - Why not C): Ultrasound is useful but not the primary imaging.
    - Why not D): CT scan is more detailed but not the first step.
    - Why not E): Bone scan is not typically used for initial evaluation of nerve injuries.

A patient is diagnosed with AIN injury.

  1. What is the best initial treatment?
    • A) Surgical decompression
    • B) Physical therapy
    • C) NSAIDs and rest
    • D) Corticosteroid injections
    • E) Nerve graft
    Answer: C) NSAIDs and restExplanation: Initial conservative treatment includes NSAIDs and rest.
    - Why not A): Surgery is considered if conservative treatment fails.
    - Why not B): Physical therapy is adjunctive but not the initial treatment.
    - Why not D): Steroid injections are not first-line for AIN injury.
    - Why not E): Nerve grafts are for severe cases with nerve damage.

A patient is undergoing treatment for AIN injury.

  1. How should the progress be monitored?
    • A) Serial nerve conduction studies
    • B) Weekly physical examinations
    • C) Repeat MRI every month
    • D) Blood tests for inflammatory markers
    • E) Regular X-rays
    Answer: A) Serial nerve conduction studiesExplanation: Nerve conduction studies help monitor nerve recovery.
    - Why not B): Physical exams are useful but less precise.
    - Why not C): MRI is not typically repeated frequently.
    - Why not D): Blood tests are not specific for nerve injury.
    - Why not E): X-rays do not monitor nerve recovery.

A patient presents with weakness in thumb flexion and index finger flexion without trauma.

  1. Which condition is least likely?
    • A) Anterior interosseous nerve injury
    • B) Carpal tunnel syndrome
    • C) Cervical radiculopathy
    • D) Brachial plexopathy
    • E) Radial tunnel syndrome
    Answer: E) Radial tunnel syndromeExplanation: Radial tunnel syndrome affects the radial nerve, not AIN.
    - Why not A): AIN injury fits the symptoms.
    - Why not B): Carpal tunnel syndrome can cause median nerve symptoms.
    - Why not C): Cervical radiculopathy can cause similar symptoms.
    - Why not D): Brachial plexopathy can cause similar symptoms.

A patient with untreated AIN injury.

  1. What is a potential complication?
    • A) Permanent muscle atrophy
    • B) Chronic pain syndrome
    • C) Loss of sensation in the hand
    • D) Recurrent infections
    • E) Joint deformity
    Answer: A) Permanent muscle atrophyExplanation: Untreated AIN injury can lead to muscle atrophy.
    - Why not B): Chronic pain is less specific.
    - Why not C): AIN is purely motor.
    - Why not D): Infections are not a direct complication.
    - Why not E): Joint deformity is less specific.

A patient with wrist pain and weakness in thumb flexion.

  1. Which finding differentiates AIN injury from carpal tunnel syndrome?
    • A) Weakness in thumb flexion
    • B) Positive Phalen’s test
    • C) Negative Tinel’s sign
    • D) Sensory loss in the first three fingers
    • E) Pain radiating to the forearm
    Answer: D) Sensory loss in the first three fingersExplanation: AIN injury does not cause sensory loss.
    - Why not A): Both can cause thumb flexion weakness.
    - Why not B): Phalen’s test is for carpal tunnel.
    - Why not C): Tinel’s sign can be positive in both.
    - Why not E): Both can cause radiating pain.

A patient with persistent AIN injury symptoms after conservative treatment.

  1. What is the next step in management?
    • A) Continue conservative treatment
    • B) Physical therapy
    • C) Surgical exploration
    • D) Corticosteroid injections
    • E) Nerve grafting
    Answer: C) Surgical explorationExplanation: Surgery is indicated if conservative treatment fails.
    - Why not A): Continuing conservative treatment may not be effective.
    - Why not B): Physical therapy alone may not resolve the issue.
    - Why not D): Steroid injections are not first-line for persistent symptoms.
    - Why not E): Nerve grafting is considered after exploration.

A patient asks about the prognosis of AIN injury.

  1. What is the typical prognosis with appropriate treatment?
    • A) Full recovery in most cases
    • B) Partial recovery with some residual weakness
    • C) No recovery without surgery
    • D) Chronic pain and disability
    • E) High risk of recurrent injury
    Answer: A) Full recovery in most casesExplanation: With appropriate treatment, most AIN injuries fully recover.
    - Why not B): Full recovery is common with proper care.
    - Why not C): Surgery is not always required

The clinical presentation is suggestive of right anterior interosseous nerve injury (AIN).
AIN is a purely motor branch of the median nerve that innervates all deep muscles of the anterior compartment of forearm except the ulnar part of the muscle flexor digitorum profundus. AIN branches from median nerve 4cm distal to medial epicondyle and runs along the front of the interosseous membrane of the forearm in the space between the flexor pollicis longus and flexor digitorum profundus. AIN ends distally in the pronator quadratus and wrist joint.

Muscles innervated by AIN include:
-Flexor pollicis longus (the long flexor of thumb)
-The radial part of flexor digitorum profundus (flexor of the index and (sometimes) middle fingers)
-Pronator quadratus (forearm pronation resulting in palm of the hand facing down)

OPTION A : Radial nerve supplies thumb and fingers extensors and forearm supinator muscles. Radial nerve injuries or neuropathies do not cause weakness of fingers flexion or forearm pronation.

OPTION B : Median nerve injuries distal to where AIN branches off cause weakness of thenar muscles including flexor pollicis brevis, abductor pollicis brevis and opponens pollicis. With compensatory action of flexor pollicis longus (supplied by the AIN) and abductor pollicis longus (supplied by the radial nerve) flexion and abduction remain almost unaffected; however, thumb opposition becomes impaired. Median nerve injuries proximal to AIN origin result in dysfunction of all median- innervated muscles in the forearm and hand.

OPTION D : Posterior interosseous nerve innervates the common and deep extensors of the fingers. It branches off from the radial nerve at the radiohumeral joint line and travels down in the posterior compartment of the forearm. Injuries or neuropathies of this nerve cause extension abnormalities, the presentation in this case scenario.

OPTION E : With ulnar nerve involvement, there would be weak flexion of the 4th and 5th fingers, weak flexion of the wrist, and sensory disturbances in the territory of the ulnar nerve.

48
Q

A 45-year-old man presents to your clinic complaining of weak right wrist flexion. On examination, small muscles of his right hand are wasted. He is not able to flex his wrist against resistance. A lesion of which of the following nerves could have led to this presentation?

A. Right ulnar nerve at the wrist.
B. Right median nerve at the wrist.
C. Right median nerve at the elbow.
D. Right ulnar nerve at the elbow.
E. Right anterior interosseous nerve.

A

D. Right ulnar nerve at the elbow

Most small muscles of the hand including dorsal and palmar interossei, lumbrical muscles of digits 4 and 5, and hypothenar eminence muscles are innervated by the ulnar nerve, the paralysis of which results in wasting of most small hand of the hand (lumbrical muscles of digits 2 and 3 and muscles of thenar eminence are innervated by the median nerve). Ulnar nerve also supplies flexor carpi ulnaris which is a wrist flexor. Ulnar nerve injuries or neuropathies at the level of the elbow can cause weakness and wasting of most of the small muscles of the hand and weakness of the wrist flexion.

Another important muscle involved in wrist flexion is flexor carpi radialis that is innervated by the median nerve.

OPTION A : Although ulnar nerve injuries at the wrist can affect the function of small muscles of the hand, flexor carpi ulnaris is innervated proximal to the wrist, and ulnar nerve injuries at wrist does not cause weak wrist flexion.

OPTION B : Median nerve injuries at the wrist results in weakness of thenar muscles and the first two lumbricals but does not affect the action of flexor carpi radialis because it is innervated proximal to the wrist.

OPTION C : With median nerve injuries at the elbow, all of the muscles innervated by the median nerve and its branches including anterior interosseous nerve are affected. However, with the exception of the first two lumbrical muscles and muscles of the thenar eminence, all other small muscles of the hand remain intact as they are supplied by the ulnar nerve.

OPTION E : Anterior interosseous nerve does not innervate any of the small muscles of the hand or main flexors of the wrist. Muscles supplies by the anterior interosseous nerve include flexor pollicis longus, the radial half of flexor digitorum profundus and pronator quadratus.

49
Q

Robert is a 72-year-old patient of yours, who was diagnosed with Parkinson disease 10 years ago. Recently he developed agitation, for which he was prescribed haloperidol. Today, he is brought to you office by his son with complaint of marked increase in his tremors. Which one of the following is the most appropriate management?

A. Do nothing.
B. Increase the dose of haloperidol.
C. Increase the dose of anti-Parkinson medications.
D. Decrease the dose of haloperidol.
E. Switch to risperidone.

A

D. Decrease the dose of haloperidol

Haloperidol is a first-generation antipsychotic that acts by inhibition of dopamine receptors in CNS. On the other hand, the pathophysiology of Parkinson disease is dopamine depletion from the basal ganglia. Consequently, haloperidol results in worsening of extrapyramidal symptoms of Parkinson disease such as bradykinesia and tremors.

All antipsychotic drugs are capable of producing extrapyramidal effects in a dose-dependent fashion. Aripiprazole, clozapine and quetiapine are probably exceptions. When extrapyramidal effects develop, it is an indication that the dose of drug has exceeded the optimal therapeutic range for a given patient. In such circumstances, the next best step would be decreasing the dose of the antipsychotic.

While decreasing the dose of haloperidol is very likely to reduce tremor, doing nothing would be an incorrect option.

OPTION B : By increasing the dose of haloperidol, the condition becomes worse.

OPTION C : Increasing the dose of anti-Parkinson drugs is not an appropriate option because the condition is caused by high doses of haloperidol and can be reversed by dose reduction.

OPTION E : Switching to another antipsychotic with the same side effect is not an appropriate option when the problem is likely to resolve with decreasing the dose of haloperidol.

50
Q

A 28-year-old man presents with distal weakness and atrophy of the small muscles of both hands including interossei and lumbricals as well as thenar and hypothenar eminences. Which one of the following is the most likely cause of his presentation?

A. Multiple sclerosis.
B. Bilateral median nerve palsy.
C. Syringomyelia.
D. Bilateral ulnar nerve palsy.
E. Brainstem infarction.

A

C. Syringomyelia

  • Syringomyelia: Development of a fluid-filled cavity (syrinx) within the spinal cord.
  • Symptoms:
    • Bilateral distal muscle weakness and wasting.
    • Involves both thenar and hypothenar eminences.
    • Loss of pain and temperature sensation.
    • Preserved light touch, vibration, and position senses initially.
    • Shawl-like distribution of sensory loss.
  • Progression: Can extend to involve the posterior columns affecting position and vibration senses.

A 45-year-old male presents with bilateral distal arm weakness and wasting of the thenar and hypothenar eminences.

  1. What is the most likely diagnosis?
    • A) Cervical spine multiple sclerosis
    • B) Bilateral carpal tunnel syndrome
    • C) Syringomyelia
    • D) Bilateral ulnar nerve palsy
    • E) Brainstem infarction
    Answer: C) SyringomyeliaExplanation: Bilateral involvement and muscle wasting of both thenar and hypothenar eminences suggest syringomyelia.
    - Why not A): Multiple sclerosis typically presents with asymmetric symptoms.
    - Why not B): Carpal tunnel syndrome affects the median nerve only.
    - Why not D): Ulnar nerve palsy would not affect the thenar eminence.
    - Why not E): Brainstem infarction usually presents with contralateral symptoms.

A patient with syringomyelia.

  1. What type of sensory loss is characteristic?
    • A) Loss of all sensory modalities
    • B) Loss of light touch and vibration sense
    • C) Loss of pain and temperature sensation
    • D) Loss of proprioception
    • E) No sensory loss
    Answer: C) Loss of pain and temperature sensationExplanation: Syringomyelia disrupts the spinothalamic tract, leading to loss of pain and temperature sensation.
    - Why not A): Not all sensory modalities are lost initially.
    - Why not B): Light touch and vibration sense are preserved early on.
    - Why not D): Proprioception is initially preserved.
    - Why not E): Sensory loss is a key feature.

A patient with suspected syringomyelia.

  1. What is the initial imaging study of choice?
    • A) X-ray of the spine
    • B) CT scan of the spine
    • C) MRI of the spine
    • D) Ultrasound of the spine
    • E) Bone scan
    Answer: C) MRI of the spineExplanation: MRI is the best imaging modality to visualize the syrinx within the spinal cord.
    - Why not A): X-ray cannot show the syrinx.
    - Why not B): CT scan is less detailed for soft tissue.
    - Why not D): Ultrasound is not effective for spinal cord imaging.
    - Why not E): Bone scan is not relevant for diagnosing syringomyelia.

A patient diagnosed with syringomyelia.

  1. What is the initial management approach?
    • A) Immediate surgical intervention
    • B) Physical therapy
    • C) NSAIDs and rest
    • D) Observation and follow-up MRI
    • E) Corticosteroid injections
    Answer: D) Observation and follow-up MRIExplanation: Initial management may involve observation and monitoring with follow-up imaging to assess progression.
    - Why not A): Surgery is considered if there is significant progression or symptoms.
    - Why not B): Physical therapy is supportive but not initial management.
    - Why not C): NSAIDs are for pain management but not primary treatment.
    - Why not E): Steroid injections are not standard treatment.

A patient with untreated syringomyelia.

  1. What is a potential complication?
    • A) Permanent muscle atrophy
    • B) Chronic pain syndrome
    • C) Loss of sensation in the hand
    • D) Recurrent infections
    • E) Joint deformity
    Answer: A) Permanent muscle atrophyExplanation: Untreated syringomyelia can lead to permanent muscle atrophy due to motor neuron damage.
    - Why not B): Chronic pain can occur but atrophy is more specific.
    - Why not C): Sensory loss can occur but muscle atrophy is significant.
    - Why not D): Infections are not a direct complication.
    - Why not E): Joint deformity can occur but atrophy is primary.

A patient with bilateral hand weakness and sensory loss.

  1. Which condition is least likely?
    • A) Cervical spine multiple sclerosis
    • B) Carpal tunnel syndrome
    • C) Syringomyelia
    • D) Bilateral ulnar nerve palsy
    • E) Brainstem infarction
    Answer: D) Bilateral ulnar nerve palsyExplanation: Bilateral ulnar nerve palsy would not explain thenar eminence involvement.
    - Why not A): Multiple sclerosis is less likely but possible.
    - Why not B): Carpal tunnel syndrome affects the median nerve but not bilaterally usually.
    - Why not C): Syringomyelia fits the bilateral symptoms.
    - Why not E): Brainstem infarction usually presents with contralateral symptoms.

A patient presents with suspected syringomyelia based on clinical symptoms.

  1. What is the confirmatory test?
    • A) Nerve conduction studies
    • B) Spinal X-ray
    • C) CT myelography
    • D) MRI of the spine
    • E) Lumbar puncture
    Answer: D) MRI of the spineExplanation: MRI is the definitive test to confirm syringomyelia by visualizing the syrinx.
    - Why not A): Nerve conduction studies are not definitive for syringomyelia.
    - Why not B): X-ray cannot visualize the syrinx.
    - Why not C): CT myelography is invasive and less preferred.
    - Why not E): Lumbar puncture does not diagnose syringomyelia.

A patient with syringomyelia and worsening symptoms.

  1. When is surgery indicated?
    • A) Initial diagnosis
    • B) Mild symptoms
    • C) Worsening neurological deficits
    • D) No improvement with NSAIDs
    • E) Asymptomatic syrinx
    Answer: C) Worsening neurological deficitsExplanation: Surgery is indicated when there are progressive or severe neurological deficits.
    - Why not A): Surgery is not always needed at initial diagnosis.
    - Why not B): Mild symptoms do not necessitate surgery.
    - Why not D): NSAIDs are not a primary treatment.
    - Why not E): Asymptomatic syrinx does not always require surgery.

A patient with syringomyelia under observation.

  1. How should the patient be monitored?
    • A) Weekly physical examinations
    • B) Annual MRI scans
    • C) Monthly X-rays
    • D) Regular nerve conduction studies
    • E) Frequent blood tests
    Answer: B) Annual MRI scansExplanation: Regular MRI scans help monitor the progression of the syrinx.
    - Why not A): Physical exams are useful but less specific.
    - Why not C): X-rays are not useful for syringomyelia monitoring.
    - Why not D): Nerve conduction studies do not visualize the syrinx.
    - Why not E): Blood tests are not relevant for syringomyelia monitoring.

A patient asks about the prognosis of syringomyelia with appropriate treatment.

  1. What is the typical prognosis?
    • A) Full recovery in most cases
    • B) Partial recovery with some residual symptoms
    • C) No recovery without surgery
    • D) Chronic pain and disability
    • E) High risk of recurrent syrinx
    Answer: B) Partial recovery with some residual symptomsExplanation: Most patients experience improvement but may have some residual symptoms.
    - Why not A): Full recovery is possible but not guaranteed.
    - Why not C): Some recovery can occur without surgery.
    - Why not D): Chronic pain is possible but not in all cases.
    - Why not E): Recurrent syrinx is possible but not the most likely outcome.

This man has bilateral global weakness of distal arms and muscle wasting of both thenar (median-innervated) and hypothenar (ulnar-innervated) eminences. Of the options, syringomyelia best justified the bilaterality of the signs and symptoms, as well as multiple nerves involvement.

Syringomyelia is the development of a fluid-filled cavity (syrinx) within the spinal cord. Syrinx extension into the anterior horns of the spinal cord damages motor neurons (lower motor neuron) and causes diffuse muscle atrophy that begins in the hands and progresses proximally to include the forearms and shoulder girdles. Claw-hand may develop.
Syrinx also interrupts the decussating spinothalamic fibers that mediate pain and temperature sensibility, resulting in loss of these sensations, while light touch, vibration, and position senses are preserved because their fibers are located in the posterior column that are not affected unless late in the course of the disease (dissociated sensory loss). When the cavity enlarges to involve the posterior columns, position and vibration senses are lost as well. The sensory disturbances usually occur in a shawl-like distribution.

OPTION A : Cervical spine multiple sclerosis can cause muscle weakness, exaggerated reflexes and decreased sensation of the upper limb; however, bilateral symptoms of this patient makes multiple sclerosis a less likely diagnosis. Bilateral median nerve palsy can cause bilateral atrophy of thenar eminence but not that of hypothenar eminence.

OPTION D : Bilateral ulnar nerve palsy can cause bilateral atrophy of hypothenar eminence but not that of thenar eminence.

OPTION E : Brainstem infarction is associated with contralateral limb weakness. Bilaterality is against brainstem infarction as a likely diagnosis.

51
Q

A 5-year-old boy is brought to your practice by his parents for assessment. According to the parents, he has several episodes of blank stare while watching TV every day, which are accompanied by bizarre movements of the head and right hand. The trunk and head turn before starts fidgeting the fingers of his right hand and smacking his lips. He seems lost during an attack. There are up to 10 episodes almost every day, each lasting about one minute before spontaneous resolution occurs. He seems confused and drowsy for 1 to 2 minutes after each attack. Which one of the following is the most appropriate treatment option for him?

A. Phenytoin.
B. Sodium valproate.
C. Ethosuximide.
D. Carbamazepine.
E. Phenobarbital.

A

D. Carbamazepine

  • Temporal Lobe Epilepsy: A type of complex partial seizure.
  • Characteristics:
    • Automatisms: fidgeting, chewing, lip-smacking, complex behaviors.
    • Duration: 1-3 minutes per attack.
    • Post-ictal confusion for 1-2 minutes.
    • Memory disturbances: déjà vu, jamais vu.
    • Hallucinations and emotional disturbances.
    • Abdominal aura: rising sensation with or without nausea.
    • No recollection of events during the attack.
  • Treatment:
    • First-line: Carbamazepine (fewer side effects, better tolerance).
    • Newer AEDs: Gabapentin, topiramate, lamotrigine, etc., preferred due to fewer side effects.
    • Surgery: Temporal lobectomy for medically intractable cases.
    • Other AEDs: Vigabatrin, felbamate for intractable epilepsy (serious side effects).

A 25-year-old male presents with episodes of fidgeting, lip-smacking, and confusion lasting 1-3 minutes. He has no recollection of these events.

  1. What is the most likely diagnosis?
    • A) Absence seizures
    • B) Temporal lobe epilepsy
    • C) Generalized tonic-clonic seizures
    • D) Simple partial seizures
    • E) Psychogenic non-epileptic seizures
    Answer: B) Temporal lobe epilepsyExplanation: Automatisms and post-ictal confusion suggest temporal lobe epilepsy.
    - Why not A): Absence seizures are brief and lack post-ictal confusion.
    - Why not C): Generalized tonic-clonic seizures involve convulsions.
    - Why not D): Simple partial seizures do not impair awareness.
    - Why not E): Psychogenic seizures lack consistent neurological findings.

A patient diagnosed with temporal lobe epilepsy.

  1. What is the first-line treatment?
    • A) Phenytoin
    • B) Carbamazepine
    • C) Ethosuximide
    • D) Phenobarbital
    • E) Vigabatrin
    Answer: B) CarbamazepineExplanation: Carbamazepine is the first-line treatment due to fewer side effects.
    - Why not A): Phenytoin is effective but has more side effects.
    - Why not C): Ethosuximide is used for absence seizures.
    - Why not D): Phenobarbital is not preferred for temporal lobe epilepsy.
    - Why not E): Vigabatrin is reserved for intractable epilepsy.

A patient with temporal lobe epilepsy not responding to medications.

  1. What is the next step in management?
    • A) Increase the dose of current AED
    • B) Add a second AED
    • C) Temporal lobectomy
    • D) Cognitive behavioral therapy
    • E) Vagus nerve stimulation
    Answer: C) Temporal lobectomyExplanation: Surgery is considered for medically intractable cases.
    - Why not A): Increasing dose may not be effective if already optimized.
    - Why not B): Adding a second AED may help but surgery is more definitive.
    - Why not D): Therapy is supportive, not primary treatment.
    - Why not E): Vagus nerve stimulation is an adjunctive therapy.

A patient presents with sudden episodes of deja vu, lip-smacking, and abdominal discomfort.

  1. Which feature is NOT typical of temporal lobe epilepsy?
    • A) Déjà vu
    • B) Lip-smacking
    • C) Abdominal aura
    • D) Sudden onset without warning
    • E) Post-ictal confusion
    Answer: D) Sudden onset without warningExplanation: Temporal lobe epilepsy often has auras before seizures.
    - Why not A): Déjà vu is a common feature.
    - Why not B): Lip-smacking is common.
    - Why not C): Abdominal aura is characteristic.
    - Why not E): Post-ictal confusion is typical.

A patient with well-controlled temporal lobe epilepsy.

  1. What is the best approach for monitoring?
    • A) Annual EEG
    • B) Regular MRI scans
    • C) Monthly blood tests
    • D) Yearly neurological exams
    • E) Routine video EEG monitoring
    Answer: D) Yearly neurological examsExplanation: Regular exams monitor control and detect side effects.
    - Why not A): EEG is used if breakthrough seizures occur.
    - Why not B): MRI is not routine unless new symptoms arise.
    - Why not C): Blood tests monitor AED levels, not seizure control.
    - Why not E): Video EEG is for diagnostic purposes, not routine monitoring.

A patient on carbamazepine for temporal lobe epilepsy.

  1. What side effect should be monitored?
    • A) Weight gain
    • B) Gingival hyperplasia
    • C) Hyponatremia
    • D) Hirsutism
    • E) Tremors
    Answer: C) HyponatremiaExplanation: Carbamazepine can cause hyponatremia.
    - Why not A): Weight gain is more common with valproate.
    - Why not B): Gingival hyperplasia is associated with phenytoin.
    - Why not D): Hirsutism is associated with phenytoin.
    - Why not E): Tremors are less specific.

A patient with temporal lobe epilepsy not tolerating carbamazepine.

  1. Which AED is a suitable alternative?
    • A) Ethosuximide
    • B) Lamotrigine
    • C) Phenobarbital
    • D) Topiramate
    • E) Felbamate
    Answer: B) LamotrigineExplanation: Lamotrigine is effective with fewer side effects.
    - Why not A): Ethosuximide is for absence seizures.
    - Why not C): Phenobarbital has more side effects.
    - Why not D): Topiramate is an alternative but less preferred.
    - Why not E): Felbamate is reserved for intractable epilepsy.

A patient experiences confusion and disorientation after a seizure.

  1. What is the best immediate management?
    • A) Administering lorazepam
    • B) Monitoring and reassurance
    • C) IV glucose administration
    • D) Initiating CPR
    • E) Performing a lumbar puncture
    Answer: B) Monitoring and reassuranceExplanation: Post-ictal confusion is typically self-limiting.
    - Why not A): Lorazepam is for active seizures, not post-ictal state.
    - Why not C): IV glucose is not indicated without hypoglycemia.
    - Why not D): CPR is not needed unless there is cardiac arrest.
    - Why not E): Lumbar puncture is not indicated in post-ictal state.

A patient with temporal lobe epilepsy undergoing evaluation for surgery.

  1. What pre-surgical evaluation is necessary?
    • A) Genetic testing
    • B) Neuropsychological testing
    • C) Skin biopsy
    • D) Liver function tests
    • E) Muscle biopsy
    Answer: B) Neuropsychological testingExplanation: Neuropsychological testing assesses cognitive function and surgical risks.
    - Why not A): Genetic testing is not routine for surgical evaluation.
    - Why not C): Skin biopsy is irrelevant.
    - Why not D): Liver function tests monitor AED side effects, not surgical risks.
    - Why not E): Muscle biopsy is not indicated.

A patient asks about the prognosis after temporal lobectomy.

  1. What is the typical outcome?
    • A) Full recovery in all cases
    • B) Partial seizure control with some residual symptoms
    • C) No improvement
    • D) Complete disability
    • E) High risk of recurrent seizures
    Answer: B) Partial seizure control with some residual symptomsExplanation: Many patients achieve significant improvement but may have some residual symptoms.
    - Why not A): Full recovery is possible but not guaranteed.
    - Why not C): Surgery often improves seizure control.
    - **Why not

The clinical picture in this child is highly suggestive of temporal lobe epilepsy as the most likely diagnosis. Temporal lobe epilepsy is categorized under complex partial seizures meaning that the seizure has a particular focus in the brain (partial) and the awareness is impaired during an attack (complex). Temporal lobe epilepsy is characterized by automatism including fidgeting, chewing, lip smacking, fumbling, or complex acts such as speed driving, kissing, violence or other bizarre behaviors. The duration of an attack is often 1-3 minutes. There is a post-ictal state of 1 to 2 minutes associated with confusion. The patient has no recollection of the events during the attack. Other features include memory disturbances such as deja vu or jamais vu, hallucinations, emotional disturbances such as a sense of sudden terror, panic, anger or drealization. Ictal or post-ictal aphasia can be a feature. Abdominal rising sensation (abdominal aura) with or without nausea and vomiting may occur. Patients may have bizarre delusions.

Many antiepileptic drugs (AEDs) have been used successfully for treatment of temporal lobe epilepsy. The three major AEDs, carbamazepine, phenytoin and sodium valproate have equal efficacy in controlling the seizures; however, of these carbamazepine is the drug of choice because it is associated with fewer side effects and better tolerance by patients.

NOTE - The newer AEDs (not mentioned as an option), such as gabapentin, topiramate, lamotrigine, levetiracetam, oxcarbazepine, pregabalin, lacoasmide, and zonisamide, have similar efficacy than the older AEDs, but they are preferred over older AEDs because they have far less adverse effects and less drug-drug interactions. If present in the options, they should be selected as the most appropriate treatment option.

Vigabatrin and felbamate are reserved for intractable epilepsy patients due to the potential of serious side effects.

NOTE - Temporal lobectomy is the definitive treatment for medically intractable temporal lobe epilepsy, as it has a high seizure-free rate.

OPTION C : Ethosuximide is a treatment option for absence seizure and phenobarbital is not a reasonable option for treatment of temporal lobe epilepsy.

52
Q

A 35-year-old woman presents with chief complaint of headaches. She describes that she has been having these headaches for the past 8 months. The headaches occurr more often in the evening hours and she took Panadol, ibuprofen, and sometimes codeine for pain relief. Recently, the headache that is varying in intensity comes early in the morning and wakes her up. The ache is felt in the frontal area bilaterally and it seems that the her regular pain killers are not that effective with only a partial relief lasting not more than 2 to 3 hours. She denies nausea and vomiting, light and noise sensitivity, or visual problems. She is otherwise in good shape, smokes 5-10 cigarettes a day, and drinks within healthy limit. Physical examination is inconclusive. Which one of the following condition is most likely to have led to this presentation?

A. Pseudotumor cerebri.
B. Migraine.
C. Tension headaches.
D. Drug rebound headache.
E. Cerebral tumor.

A

D. Drug rebound headache

  1. Medication Overuse Headache:
    • Common in those with a long history of tension or migraine headaches.
    • Frequent or daily headaches, often early morning.
    • Partial relief with analgesics, but pain returns as medication wears off.
    • No focal neurological symptoms.
  2. Other Headache Types:
    • Tension Headache: Bilateral, band-like pressure, worse in the evening, responds well to analgesics.
    • Migraine: Unilateral, pulsating, associated with nausea/vomiting, sensitivity to light/noise.
    • Cerebral Tumor: Early morning headache, may have projectile vomiting, focal neurological findings.
    • Idiopathic Intracranial Hypertension (IIH): Bilateral headache, papilledema, visual disturbances, pulsatile tinnitus.

A 45-year-old woman with a 10-year history of tension headaches now has daily morning headaches that partially improve with analgesics but return later.

  1. What is the most likely diagnosis?
    • A) Tension headache
    • B) Migraine
    • C) Medication overuse headache
    • D) Idiopathic intracranial hypertension
    • E) Cerebral tumor
    Answer: C) Medication overuse headacheExplanation: Daily morning headaches that improve with analgesics but return later suggest medication overuse headache.
    - Why not A): Tension headaches typically worsen in the evening.
    - Why not B): Migraine is usually unilateral with associated symptoms.
    - Why not D): IIH has visual symptoms and papilledema.
    - Why not E): Cerebral tumors often present with focal neurological findings.

A patient diagnosed with medication overuse headache.

  1. What is the next step in management?
    • A) Increase dose of current analgesics
    • B) Discontinue all analgesics
    • C) Start prophylactic migraine medication
    • D) Refer to neurologist for imaging
    • E) Prescribe stronger pain medication
    Answer: B) Discontinue all analgesicsExplanation: Discontinuing all analgesics is crucial in breaking the cycle of medication overuse headaches.
    - Why not A): Increasing dose exacerbates the problem.
    - Why not C): Prophylactic medication can be considered later.
    - Why not D): Imaging is not required without new neurological findings.
    - Why not E): Stronger pain medication worsens medication overuse.

A patient asks about risk factors for medication overuse headache.

  1. Which of the following is a risk factor?
    • A) Infrequent use of analgesics
    • B) Long-term prophylactic medication use
    • C) Recent head trauma
    • D) Frequent use of triptans
    • E) Family history of headaches
    Answer: D) Frequent use of triptansExplanation: Frequent use of triptans is a known risk factor for medication overuse headaches.
    - Why not A): Infrequent use does not lead to overuse headaches.
    - Why not B): Prophylactic medications aim to prevent headaches, not cause them.
    - Why not C): Head trauma may cause other types of headaches but not medication overuse.
    - Why not E): Family history may predispose to primary headache disorders but not directly to medication overuse.

A patient presents with bilateral, band-like headache that worsens in the evening.

  1. What is the most likely diagnosis?
    • A) Migraine
    • B) Medication overuse headache
    • C) Tension headache
    • D) Cluster headache
    • E) Sinus headache
    Answer: C) Tension headacheExplanation: Bilateral, band-like headache worsening in the evening is characteristic of tension headaches.
    - Why not A): Migraine is unilateral and pulsating.
    - Why not B): Medication overuse headache occurs more frequently in the morning.
    - Why not D): Cluster headaches are unilateral and excruciating.
    - Why not E): Sinus headache involves facial pain and congestion.

A patient presents with early morning headaches and projectile vomiting.

  1. What is the next step in management?
    • A) Start migraine prophylaxis
    • B) Refer for urgent MRI of the brain
    • C) Prescribe stronger analgesics
    • D) Recommend lifestyle modifications
    • E) Order a lumbar puncture
    Answer: B) Refer for urgent MRI of the brainExplanation: Early morning headaches and projectile vomiting suggest a cerebral tumor, requiring urgent imaging.
    - Why not A): Migraine prophylaxis is not indicated without a clear diagnosis of migraine.
    - Why not C): Stronger analgesics do not address the underlying cause.
    - Why not D): Lifestyle modifications are supportive but not primary management.
    - Why not E): Lumbar puncture is indicated after imaging to rule out increased intracranial pressure.

A patient with a history of morning headaches and recent visual disturbances.

  1. What is the initial investigation?
    • A) EEG
    • B) MRI of the brain
    • C) Fundoscopy
    • D) Lumbar puncture
    • E) Blood glucose testing
    Answer: C) FundoscopyExplanation: Fundoscopy checks for papilledema, indicating increased intracranial pressure.
    - Why not A): EEG is used for seizure activity.
    - Why not B): MRI is the next step if fundoscopy is abnormal.
    - Why not D): Lumbar puncture follows imaging to avoid complications.
    - Why not E): Blood glucose is not relevant to this presentation.

A patient presents with headaches, visual disturbances, and pulsatile tinnitus.

  1. What is the most likely diagnosis?
    • A) Migraine
    • B) Tension headache
    • C) Idiopathic intracranial hypertension
    • D) Cluster headache
    • E) Sinus headache
    Answer: C) Idiopathic intracranial hypertensionExplanation: Headaches, visual disturbances, and pulsatile tinnitus are characteristic of IIH.
    - Why not A): Migraine lacks pulsatile tinnitus.
    - Why not B): Tension headaches do not have visual disturbances.
    - Why not D): Cluster headaches do not present with pulsatile tinnitus.
    - Why not E): Sinus headaches involve facial pain and congestion.

A patient presents with bilateral, evening headaches relieved by analgesics.

  1. What is the initial management?
    • A) MRI of the brain
    • B) Start prophylactic medication
    • C) Recommend stress management techniques
    • D) Prescribe triptans
    • E) Order a lumbar puncture
    Answer: C) Recommend stress management techniquesExplanation: Stress management is crucial in managing tension headaches.
    - Why not A): MRI is not indicated without alarming symptoms.
    - Why not B): Prophylactic medication is for frequent, severe headaches.
    - Why not D): Triptans are for migraines, not tension headaches.
    - Why not E): Lumbar puncture is not indicated for tension headaches.

A patient presents with severe, unilateral headaches, tearing, and nasal congestion.

  1. What is the most likely diagnosis?
    • A) Migraine
    • B) Tension headache
    • C) Cluster headache
    • D) Sinus headache
    • E) Medication overuse headache
    Answer: C) Cluster headacheExplanation: Severe unilateral headaches with tearing and nasal congestion are typical of cluster headaches.
    - Why not A): Migraine does not have tearing and nasal congestion.
    - Why not B): Tension headache is bilateral.
    - Why not D): Sinus headache is less severe and constant.
    - Why not E): Medication overuse headache is daily and bilateral.

A patient with frequent migraines not responding to acute treatment.

  1. What is the best long-term management?
    • A) Increase dose of acute medication
    • B) Start prophylactic medication
    • C) Recommend regular exercise
    • D) Advise dietary modifications
    • E) Schedule regular follow-ups
    Answer: B) Start prophylactic medicationExplanation: Prophylactic medication is indicated for frequent migraines.
    - Why not A): Increasing acute medication can lead to medication over

Although early morning headache is characteristic of headaches caused by cerebral tumors (option E)
no history projectile vomiting or absence of focal neurological findings makes such diagnoses less likely.

Migraine (option B) headache is usually unilateral and felt behind the orbit and/or temporal regions. However, it can progress to affect the entire head and neck. In this patient, the pain is bilateral and other features of migraine such as nausea and vomiting and sensitivity to noise and light are absent. With these, migraine is an unlikely diagnosis.

Tension headache (option C) is very common. The pain is bilateral and described as band-like pressure. Nausea and vomiting are not prominent features and patient is not sensitive to light and noise. One important characteristic of tension headache is the fact that it occurs mainly towards the end of the day (evening hours). Adequate response to analgesics is another distinctive feature of tension headaches.

Although the headache in this woman is bilateral and not associated with nausea and vomiting and resembles tension headache, the time of onset (early in the morning) and partial and inadequate response to analgesia makes this diagnosis less likely for this woman.

Idiopathic intracranial hypertension (IIH), also known as pseudotumor cerebri, is a disorder of unknown etiology. It can occur in all age groups but is most common in obese women of childbearing age. The primary problem in IIH is chronically elevated intracranial pressure (ICP). With increased ICP, the most important neurological finding is papilledema that can result in progressive optic atrophy, visual loss, and potentially blindness. In general and as the name implies, sign and symptoms of IIH are similar to any space-occupying brain lesion such as a cerebral tumor, including:

  • Headache (typically nonspecific and varying in type, location, intensity, and frequency)
  • Nausea and vomiting
  • dizziness
  • Visual loss (typically visual field but rarely visual acuity loss)
  • Brief episodes of blindness lasting only a few seconds and affecting one or both eye (visual obscuration)
  • Diplopia (typically horizontal due to non-localizing sixth nerve palsy but rarely vertical)
  • Pulsatile tinnitus – ringing in the ears that pulses in time with the heartbeat
  • Pain in the neck, shoulders or arms (not very common)

With just headache in the history and absence of other presenting features of pseudotumor cerebri, such diagnosis is unlikely.

As a matter of fact, with prolonged use of analgesics for chronic headache and absence of alarming findings in the history and physical examinations, this clinical scenario is most consistent with drug rebound headache, also known as medication overuse headache. Classically, a typical patient is described as a 30-60 years old woman with a history of more than a decade of tension or migraine headache and pain killer use for relief; however, it may affect patients of different ages with a history of infrequent analgesic use for headache even as short as days or weeks. Therefore, a high index of suspicion should be held for every patient presenting with frequent headaches. The characteristic features of medication overuse headache is not uniform but often is similar to quality of pain in the original headache, for which medications are taken. The condition is more prevalent in patient who use prophylaxis for their headaches. In General, medication overuse headache occurs every day or almost every day, often waking up the patient early in the morning when the effect of the painkillers has worn off overnight. The pain is partially improved with medications but returns as the medication wears off again.

Medication overuse headache is not associated with focal or lateralizing neurological symptoms. It is, however, common for patients with migraine with aura to experience aura if they develop medication overuse headaches. It is important to note that if the patient’s symptoms have been stable months or years, imaging or neurological examination is not required provided that there is no neurological findings in between headaches.

53
Q

A 30-year-old woman presents to your GP clinic with complaint of headaches 3 to 5 days before her periods. The headache often starts from the back of her head (occipital area) and radiates to her left temple, and is aggravated by walking or other types of physical activity. She denies visual, auditory, or taste disturbances, but mentions that during attacks she cannot tolerate noisy or crowded places. According to her, every episode of headache lasts from two to several hours. Physical examination, including a full neurological exam is inconclusive. Which one of the following could be the most likely diagnosis?

A. Migraine with aura.
B. Migraine without aura.
C. Tension vascular headache.
D. Cerebral tumor.
E. Premenstrual tension headache.

A

B. Migraine without aura

The description of headache in the patient’s words implies a unilateral nature for the pain (from occipital area radiating to the left temple). The headache is aggravated by walking and physical activity and the patient is intolerant of noise and crowded places. These, collectively, make migraine the most likely diagnosis. She also denies visual, taste, or auditory disturbances (aura). So she is very likely to be suffering from migraine without aura. Relation to menstruation (hormonal changes as a trigger) is also another supporting evidence for such diagnosis.

Over 50% of women with migraine report an association between migraine and menstruation. Such association has been hypothesized to be due to hormonal changes around menstruation.

OPTION A : Absence of visual, taste or auditory disturbances makes migraine with aura (classic migraine) an unlikely diagnosis.

OPTION C : Tension-vascular headache is a term used for headaches that cannot be categorized as tension-type headache or migraine headache due to overlapping symptoms. In such patients both migraine and tension headaches are present. Findings in this woman are nearly classic for migraine without aura.

The following tables outline differences between tension headache and migraine:

(TABLE IN PAGE 776)

OPTION D : Headache caused by space-occupying brain lesions such as cerebral tumors characteristically occur early in the morning and are associated with forceful (projectile) vomiting and focal neurological findings, absence of which in the scenario make a space-occupying lesion as the cause of the headaches less likely. Also, headache associated with such lesions do not occur just prior to menstruation and in a cyclic fashion.

OPTION E : Although this woman has headaches before menstruation, the features of headache are more consistent with migraine rather than tension headache.

54
Q

Joan, 60 years of age, is in the Emergency Department with complaint of acute onset of a spinning sensation at home followed by severe nausea and vomiting. She has never had a similar episode. She also denies any recent history of upper respiratory tract infections. On examination, she has a blood pressure of 164/104 mmHg, heart rate of 103 bpm and regular, respiratory rate of 17 breaths per minute and temperature of 37.0°C. Remarkable exam findings include nystagmus, decreased hearing of the left ear, left-sided Horner syndrome, ipsilateral sensory loss of the face and contralateral sensory deficits of the limbs. No motor dysfunction of the limbs is noted but ataxia is present. Based on the clinical picture, a diagnosis of stroke is made. Which one of the following arteries is most likely to be occluded?

A. Anterior communicating artery.
B. Posterior cerebral artery.
C. Anterior cerebral artery.
D. Posterior inferior cerebellar artery.
E. Internal carotid artery.

A

D. Posterior inferior cerebellar artery

Key Points:
1. Cause: Most often due to occlusion of the vertebral artery or posterior inferior cerebellar artery (PICA).
2. Symptoms:
- Ipsilateral:
- Ataxia and dysmetria
- Horner syndrome (ptosis, miosis, anhidrosis)
- Loss of facial pain and temperature sensation
- Nystagmus
- Dysphagia and dysarthria
- Hypoacusis
- Contralateral:
- Loss of pain and temperature sensation in the body and extremities
3. Other Findings: Tachycardia, dyspnea, palatal myoclonus.

A 60-year-old man presents with vertigo, decreased hearing, Horner syndrome, and sensory loss on the left face and right limbs.

  1. What is the most likely diagnosis?
    • A) Anterior cerebral artery stroke
    • B) Posterior cerebral artery stroke
    • C) Anterior communicating artery stroke
    • D) Lateral medullary syndrome
    • E) Internal carotid artery occlusion
    Answer: D) Lateral medullary syndromeExplanation: The combination of ipsilateral face and contralateral body sensory loss, along with Horner syndrome, suggests lateral medullary syndrome.
    - Why not A): ACA strokes cause personality changes and lower limb weakness.
    - Why not B): PCA strokes present with vision loss and memory problems.
    - Why not C): Anterior communicating artery strokes involve personality changes and leg symptoms.
    - Why not E): Internal carotid artery occlusion involves vision loss and a mix of ACA and MCA symptoms.

A patient presents with symptoms suggestive of lateral medullary syndrome.

  1. What is the initial investigation?
    • A) MRI of the brain
    • B) CT scan of the brain
    • C) Carotid Doppler ultrasound
    • D) EEG
    • E) Blood glucose testing
    Answer: A) MRI of the brainExplanation: MRI is the best imaging modality for detecting brainstem strokes.
    - Why not B): CT scan is less sensitive for posterior circulation strokes.
    - Why not C): Carotid Doppler is used for evaluating carotid artery stenosis.
    - Why not D): EEG is used for seizure activity.
    - Why not E): Blood glucose testing is not relevant to this presentation.

A patient diagnosed with lateral medullary syndrome.

  1. What is the next step in management?
    • A) Aspirin
    • B) Thrombolysis
    • C) Anticoagulation with warfarin
    • D) Surgical decompression
    • E) High-dose steroids
    Answer: A) AspirinExplanation: Aspirin is indicated for most ischemic strokes unless contraindications exist.
    - Why not B): Thrombolysis is considered in acute settings within the time window.
    - Why not C): Anticoagulation is for specific indications like atrial fibrillation.
    - Why not D): Surgical decompression is not indicated for small brainstem strokes.
    - Why not E): Steroids are not indicated for ischemic strokes.

A patient with lateral medullary syndrome presents with ptosis, miosis, and anhidrosis.

  1. Which syndrome is this?
    • A) Horner syndrome
    • B) Brown-Séquard syndrome
    • C) Carpal tunnel syndrome
    • D) Guillain-Barré syndrome
    • E) Raynaud’s syndrome
    Answer: A) Horner syndromeExplanation: Ptosis, miosis, and anhidrosis are characteristic of Horner syndrome.
    - Why not B): Brown-Séquard syndrome involves spinal cord hemisection.
    - Why not C): Carpal tunnel syndrome involves median nerve compression.
    - Why not D): Guillain-Barré syndrome involves ascending paralysis.
    - Why not E): Raynaud’s syndrome involves vasospastic attacks in extremities.

A patient presents with vertigo, dysphagia, and sensory loss.

  1. Which diagnosis should be considered?
    • A) Benign paroxysmal positional vertigo (BPPV)
    • B) Ménière’s disease
    • C) Vestibular neuritis
    • D) Lateral medullary syndrome
    • E) Labyrinthitis
    Answer: D) Lateral medullary syndromeExplanation: Vertigo with dysphagia and sensory loss suggests a brainstem stroke.
    - Why not A): BPPV causes positional vertigo without other neurological symptoms.
    - Why not B): Ménière’s disease involves hearing loss and tinnitus without neurological signs.
    - Why not C): Vestibular neuritis causes vertigo without sensory loss.
    - Why not E): Labyrinthitis involves hearing loss and vertigo without sensory loss.

A patient presents with acute vision loss, confusion, and paresthesia.

  1. What is the most likely diagnosis?
    • A) Anterior cerebral artery stroke
    • B) Posterior cerebral artery stroke
    • C) Lateral medullary syndrome
    • D) Middle cerebral artery stroke
    • E) Carotid artery stenosis
    Answer: B) Posterior cerebral artery strokeExplanation: Vision loss, confusion, and paresthesia are indicative of PCA stroke.
    - Why not A): ACA stroke presents with lower limb weakness and personality changes.
    - Why not C): Lateral medullary syndrome involves vertigo and sensory loss.
    - Why not D): MCA stroke presents with hemiparesis and aphasia.
    - Why not E): Carotid artery stenosis typically causes transient ischemic attacks.

A patient presents with lower limb weakness and personality changes.

  1. What is the most likely diagnosis?
    • A) Posterior cerebral artery stroke
    • B) Middle cerebral artery stroke
    • C) Anterior cerebral artery stroke
    • D) Lateral medullary syndrome
    • E) Basilar artery stroke
    Answer: C) Anterior cerebral artery strokeExplanation: Lower limb weakness and personality changes are characteristic of ACA stroke.
    - Why not A): PCA stroke involves vision loss and confusion.
    - Why not B): MCA stroke presents with upper limb and facial weakness.
    - Why not D): Lateral medullary syndrome involves vertigo and sensory loss.
    - Why not E): Basilar artery stroke causes bilateral symptoms and cranial nerve deficits.

A patient presents with right-sided facial weakness and aphasia.

  1. What is the most likely diagnosis?
    • A) Anterior cerebral artery stroke
    • B) Posterior cerebral artery stroke
    • C) Lateral medullary syndrome
    • D) Middle cerebral artery stroke
    • E) Basilar artery stroke
    Answer: D) Middle cerebral artery strokeExplanation: Facial weakness and aphasia are indicative of MCA stroke.
    - Why not A): ACA stroke presents with lower limb weakness and personality changes.
    - Why not B): PCA stroke involves vision loss and confusion.
    - Why not C): Lateral medullary syndrome involves vertigo and sensory loss.
    - Why not E): Basilar artery stroke causes bilateral symptoms and cranial nerve deficits.

A patient presents with acute stroke symptoms.

  1. What is the initial management?
    • A) Aspirin
    • B) Thrombolysis
    • C) Anticoagulation with warfarin
    • D) High-dose steroids
    • E) Surgical decompression
    Answer: B) ThrombolysisExplanation: Thrombolysis is the initial treatment for eligible patients with acute ischemic stroke.
    - Why not A): Aspirin is used if thrombolysis is not indicated.
    - Why not C): Anticoagulation is for specific indications like atrial fibrillation.
    - Why not D): Steroids are not indicated for ischemic strokes.
    - Why not E): Surgical decompression is for hemorrhagic strokes or large infarcts causing herniation.

A patient presents with vision loss, hemiparesis, and sensory loss.

  1. What is the most likely diagnosis?
    • A) Anterior cerebral artery

The clinical findings of vertigo, decreased hearing, Horner syndrome, and the pattern of loss of sensation in the face and limbs (ipsilateral and contralateral respectively) is mostly consistent with lateral medullary syndrome, also known as Wallenberg syndrome.

This syndrome is most often due to vertebral artery and less commonly to posterior inferior cerebellar artery (PICA) occlusion. Patients with the lateral medullary syndrome present with nausea, vomiting, and vertigo from involvement of the vestibular system. Other findings may include:

  1. Ipsilateral:
    -Ataxia and dysmetria, due to damage to the inferior cerebellar peduncle and cerebellum
    -Horner syndrome (ptosis, miosis, hypohidrosis or anhydrosis), due to damage to descending sympathetic fibers
    -Loss of facial pain and temperature sensation
    -Reduced corneal reflex due to damage to the descending spinal tract and nucleus of CN V
    - Nystagmus
    -Hypoacusis (involvement of cochlear nucleus)
    -Dysarthria
    -Dysphagia
    -Paralysis of the pharynx, palate, and vocal cord
    -Loss of taste from the posterior third of the tongue (nuclei or fibers of CN IX and X)
  2. Contralateral:
    -Loss of pain and temperature sense in the body and extremities, indicating involvement of the lateral spinothalamic tract.

Other findings include tachycardia and dyspnea (dorsal nucleus of CN X) and palatal myoclonus, which is a rhythmic involuntary jerking movement of the soft palate, pharyngeal muscles, and diaphragm. Palatal myoclonus sometimes follows infarction of the dentate nucleus of the cerebellum and inferior oliva.

OPTION A : Anterior communicating artery is an artery in the circle of Willis that connects the right and left anterior cerebral arteries. Aneurysms of this artery are the most common circle of Willis aneurysms. The ischemic strokes due to occlusion of this artery are not that common but if they occur, presentation will be similar to that of anterior cerebral artery strokes with personality changes, motor weakness, and sensory disturbances which are more prominent in the lower limbs.

OPTION B : The posterior cerebral arteries (PCAs) are paired arteries, usually branching from the top of the basilar artery. They curve laterally, posteriorly, and superiorly around the midbrain. The PCAs supply parts of the midbrain, subthalamic nucleus, basal nucleus, thalamus, mesial inferior temporal lobe, and occipital and occipitoparietal cortices. In addition, the PCAs, via the posterior communicating arteries (PCOM), may become important sources of collateral circulation for the middle cerebral artery (MCA) territory.

Patients with PCA stroke/ TIA often present with the following symptoms:
-Acute vision loss
-Confusion
-New onset posterior cranium headache
-Paresthesia
-Limb weakness
-Dizziness
-Nausea
-Memory loss
-Language dysfunction

OPTION C : Anterior cerebral artery (ACA) supplies mostly the frontal lobes and parasagittal areas. Strokes caused by ACA stroke/ TIA may present with lower limb weakness and sensory disturbances, personality changes (inappropriate behavior), and apraxia and aphasia. The scenario is inconsistent with ACA strokes.

OPTION E : With internal carotid occlusion, the presentation is a combination of signs and symptoms seen in both anterior and middle cerebral arteries PLUS vision loss (i.e., amaurosis fugax).

55
Q

A 45-year-old man presents to the Emergency Department with complaints of acute-onset vertigo, right eyelid drooping, and numbness of the lower right half of the face. He also complains of blurred vision and decreased hearing in the right ear. On examination, he has a blood pressure of 176/95 mmHg, heart rate of 94 bpm, respiratory rate of 20 breaths per minute, temperature of 36.8°C, and oxygen saturation of 96% on room air. There is right-sided miosis, ptosis and anhidrosis. He has nystagmus and ataxic movements and sensory loss of the left upper and lower limbs. Rinne and Weber tests show sensorineural deafness of the right ear. Which one of the following could be the most likely cause of this constellation of symptoms?

A. Right-sided Horner syndrome.
B. Left vertebral artery thrombosis.
C. Right vertebral artery thrombosis.
D. Obstruction of the anterior communicating artery.
E. Obstruction of the posterior cerebral artery.

A

C. Right vertebral artery thrombosis

Here’s a simple way to understand and remember the key points about vertebrobasilar stroke:

  • The patient has a stroke affecting the vertebrobasilar system, which supplies blood to important parts of the brain like the brainstem, cerebellum, and the back of the brain (occipital lobe).
  1. Vertigo: The patient feels dizzy and like the room is spinning, due to the stroke affecting the part of the brain that controls balance (vestibular system).
  2. Horner Syndrome: The patient has drooping of the eyelid (ptosis), a smaller pupil in one eye (miosis), and no sweating on one side of the face (anhidrosis). This happens because the stroke affects nerves in the brainstem.
  3. Hearing Loss on One Side: The stroke also affects the nerves related to hearing, leading to hearing loss in one ear.
  4. Blurred Vision and Numbness: The patient experiences blurred vision and numbness on one side of the face and the opposite side of the body.
  • Involvement of the Brainstem: The brainstem controls many basic functions and is where the symptoms like vertigo, Horner syndrome, and hearing loss come from.
  • Crossover of Symptoms: Symptoms like numbness on one side of the face and the opposite side of the body are classic signs of a brainstem stroke, which is part of the vertebrobasilar system.
  • Cerebellar Signs: Problems with coordination and balance (ataxia) are common because the cerebellum, which controls these functions, is affected.
  • Horner Syndrome alone doesn’t explain all these symptoms, so it’s not just that.
  • If the stroke was on the left side of the vertebral artery, symptoms would be on the left side of the face, not the right.
  • Anterior communicating artery strokes mostly affect personality, legs, and cause more general weakness, which doesn’t match this case.
  • Posterior cerebral artery strokes usually cause vision loss and memory problems but don’t cause the full range of symptoms seen here.

Think of vertebrobasilar strokes as affecting the back of the brain and brainstem. If the symptoms involve dizziness, mixed-sided numbness, coordination problems, and something like Horner syndrome, it’s likely a vertebrobasilar stroke.

Constellation of clinical findings in this patient, including vertigo, right Horner syndrome indicated by miosis, ptosis and anhidrosis, right-sided sensorineural hearing loss, blurred vision, and numbness of the right side of the face and left side of the body is highly suggestive of right-sided vertebrobasilar stroke.

The vertebral arteries arise from the subclavian arteries, and as they course cephalad in the neck, they pass through the costotransverse foramina of C6 to C2. They enter the skull through the foramen magnum and merge at the pontomedullary junction to form the basilar artery. Each vertebral artery usually gives off the posterior inferior cerebellar artery (PICA). At the top of the pons, the basilar artery divides into 2 posterior cerebral arteries (PCAs).

The vertebrobasilar arterial system provides blood supply to the medulla, cerebellum, pons, midbrain, thalamus, and occipital cortex. Occlusion of large vessels in this system usually leads to major disability or death (mortality rate>85%). Because of involvement of the brainstem and cerebellum, most survivors have multisystem dysfunction such as quadriplegia or hemiplegia, ataxia, dysphagia, dysarthria, gaze abnormalities, and cranial neuropathies. The clinical correlates of these smaller lesions consist of a variety of focal neurologic deficits, depending on their location within the brainstem.

Vertebrobasilar strokes have distinct characteristic features that differentiate them from hemispheric strokes caused by lesions of anterior or middle cerebral arteries or internal carotid artery. These features include:
1. When cranial nerves or their nuclei are involved, the corresponding clinical signs are ipsilateral to the lesion and the corticospinal signs are crossed, involving the opposite arm and leg.
2. Cerebellar signs (e.g. dysmetria, ataxia) are frequently observed.
3. Involvement of the ascending sensory pathways may affect the spinothalamic pathway or the medial lemniscus (dorsal columns), resulting in dissociated sensory loss, which is loss of one sensory modality on one side and preservation of other sensory modalities in the opposite limbs (dissociative sensory loss).
4. Dysarthria and dysphagia are typically present.
5. Vertigo, nausea, and vomiting, along with nystagmus, represent involvement of the vestibular system and are seen in vertebrobasilar strokes.
6. Unilateral Horner syndrome occurs with brainstem lesions.
7. Occipital lobe lesions result in visual field loss or visuospatial deficits.
8. Cortical deficits, such as aphasia and cognitive impairments, are absent

OPTION A : Horner syndrome is a constellation of signs (miosis, ptosis, and anhidrosis) and a feature of vertebrobasilar stroke. It does not explain this constellation of symptoms.

OPTION B : With left vertebral artery lesions, left-sided Horner syndrome, numbness of the left half of the face and that of the right limbs and left-sided hearing loss would have been expected.

OPTION D : Anterior communicating artery is an artery in the circle of Willis that connects right and left anterior cerebral arteries. Aneurysms of this artery are the most common circle of Willis aneurysms. However, ischemic strokes due to occlusion of this artery are not that common, but if they occur presentation will be similar to that of anterior cerebral artery strokes with personality changes, motor weakness and sensory disturbances more prominent in the lower limbs.

OPTION E : The posterior cerebral arteries (PCAs) are paired arteries, usually branching from the top of the basilar artery. They curving laterally, posteriorly, and superiorly around the midbrain. The PCAs supply parts of the midbrain, subthalamic nucleus, basal nucleus, thalamus, mesial inferior temporal lobe, and occipital and occipitoparietal cortices. In addition, the PCAs, via the posterior communicating arteries (PCOM), may become important source of collateral circulation for the middle cerebral artery (MCA) territory.

Although posterior cerebral artery is a branch of the basilar artery from the vertebrobasilar system, lesions at this rather terminal branch are associated with a narrower range of symptoms, mostly including:
-Acute vision loss
-Confusion
-New onset posterior cranium headache
-Paresthesia
-Limb weakness
-Dizziness
-Nausea
-Memory loss
-Language dysfunction

With the presence of vertigo, Horner syndrome, hearing loss and ataxia indicating lesions of cerebellum and medulla, the lesion is expected to be more proximal to the PCA.

56
Q

A 56-year-old man is brought to the Emergency Department with presenting complaint of vertigo. He describes that he had mild dizziness and nausea before he hears a popping sound in his right ear. The dizziness intensified afterwards, and he felt a severe spinning sensation associated with nausea and two episodes of vomiting. On examination, he has a blood pressure of 145/105 mmHg, pulse rate of 100 bpm, respiratory rate of 18 breaths per minute, and a temperature of 36.7°C. He also has right-sided Horner’s syndrome, complete sensorineural deafness of the right ear and nystagmus. There is paresthesia of the left upper and lower limbs. Which one of the following would be the most likely cause of this clinical presentation?

A. Meniere’s disease.
B. Acoustic neuroma.
C. Acute labyrinthitis.
D. Vertebrobasilar insufficiency.
E. Vestibular neuronitis.

A

D. Vertebrobasilar insufficiency

This patient’s symptoms—vertigo, Horner’s syndrome, sensorineural deafness, and nystagmus on the right side, along with left-sided sensory impairment—suggest a problem in the brainstem, specifically related to blood flow issues in the vertebrobasilar artery.

  1. What’s Happening:
    • The vertebrobasilar arteries supply blood to important parts of the brain like the brainstem and cerebellum. When blood flow is disrupted (vertebrobasilar insufficiency), it can cause a stroke or similar symptoms.
  2. Why These Symptoms?:
    • Vertigo, Nystagmus, Nausea, Vomiting: These are caused by involvement of the vestibular system in the brainstem, which controls balance and eye movements.
    • Horner’s Syndrome: A sign of damage in the brainstem, leading to drooping eyelid, small pupil, and reduced sweating on one side of the face.
    • Sensorineural Deafness: Hearing loss on the right side due to involvement of the auditory pathways.
    • Contralateral Sensory Impairment: Sensory loss on the opposite side of the body (left side) because the brainstem pathways cross over.
  3. Most Likely Diagnosis:
    • This pattern suggests occlusion (blockage) of the left inferior cerebellar artery (PICA), leading to a condition known as lateral medullary syndrome (Wallenberg syndrome).
  • Meniere’s Disease: Causes vertigo, tinnitus, and hearing loss but doesn’t involve neurological symptoms like Horner’s syndrome or sensory loss in the body.
  • Acoustic Neuroma: Usually causes gradual hearing loss, not sudden onset vertigo or sensory changes on the opposite side of the body.
  • Acute Labyrinthitis: Causes vertigo and hearing loss, often after a viral infection, but doesn’t involve Horner’s syndrome or sensory changes in the body.
  • Vestibular Neuronitis: Involves vertigo and imbalance without hearing loss or neurological symptoms like Horner’s syndrome.

This patient’s symptoms are best explained by a blockage in a blood vessel (PICA) that supplies the brainstem, leading to a lateral medullary stroke. This condition causes a mix of symptoms on the same side of the body as the stroke (like vertigo and Horner’s syndrome) and the opposite side (like sensory loss). Other conditions, like Meniere’s disease or acoustic neuroma, don’t match because they don’t cause this specific pattern of symptoms.

With vertigo, Horner’s syndrome and sensorineural deafness and nystagmus on the right side after a popping sound in the ear (which indicates sudden onset tinnitus), a central cause for this presentation should be thought of first. Considering the fact that this patient has also contralateral (left-sided) sensory impairment, vertebrobasilar insufficiency is the most likely explanation.

The vertebrobasilar arterial system provides blood supply to the medulla, cerebellum, pons, midbrain, thalamus, and occipital cortex.

Characteristic features of vertebrobasilar insufficiency/ stroke include the following:
-Vertigo, nausea, and vomiting, along with nystagmus, represent involvement of the vestibular system, and are seen in vertebrobasilar strokes.
-Unilateral Horner syndrome occurs with brainstem lesions.
-Cerebellar signs (e.g., dysmetria, ataxia) are frequently observed.
-Dysarthria and dysphagia are typically present.
-When cranial nerves or their nuclei are involved, the corresponding clinical signs are ipsilateral to the lesion and the corticospinal signs are crossed, involving the opposite arm and leg.
-Involvement of the ascending sensory pathways may affect the spinothalamic pathway or the medial lemniscus (dorsal column), resulting in dissociated sensory loss, which is loss of one sensory modality on one side and preservation of other sensory modalities in the opposite limbs (dissociative sensory loss).
-Occipital lobe lesions result in visual field loss or visuospatial deficits.
-Cortical deficits, such as aphasia and cognitive impairments, are absent.

In the absence of visual problems, this patient is more likely to have developed occlusion of the left inferior cerebellar artery (PICA) occlusion. The following diagram depicts the vertebrobasilar circulation: see anatomy of circle of willis below.

PICA is the largest branch of the vertebral artery and is one of the three main arteries that supply the cerebellum (the other two are superior cerebellar artery and anterior inferior cerebellar artery). Occlusion of PICA (A) results in a clinical syndrome called lateral medullary syndrome, Wallenberg syndrome or PICA syndrome.

This syndrome is characterized by sensory deficits that affect the trunk and extremities contralaterally (opposite to the lesion), and sensory deficits of the face and cranial nerves ipsilaterally (same side as the lesion).
The cross body finding is the chief symptom leading to PICA syndrome as the diagnosis.

Other possible symptoms include:
1. Difficulty walking or maintaining balance (ataxia) or difference in temperature sensation between left and right sides of the body
2. Attacks of vertigo and nystagmus
3. Difficulty swallowing (dysphagia) caused by the involvement of the nucleus ambiguus (this nucleus supplies the vagus and glossopharyngeal nerves).
4. Dysarthria and dysphonia
5. Ataxia (imbalance on walking) due to damage to the cerebellum or the inferior cerebellar peduncle
6. Horner’s syndrome caused by damage to hypothalamospinal fibers and disruption of sympathetic nervous system
7. Palatal myoclonus
8. Hoarseness
9. Nausea and vomiting
10. Bradycardia and dysregulation of blood pressure (due to injuries of nucleus ambiguous)

OPTION A : Meniere’s disease is caused by abnormal accumulation of endolymph in the labyrinth, and presents with vertigo, tinnitus and hearing loss lasting for hours each time. Patients often mention a sensation of fullness in the ear. There is often a positive family history. In Meniere’s disease, the pathology is contained within the labyrinth, and there is no neurological abnormalities such as Horner’s syndrome or sensory deficits of the body otherwise.

OPTION B : Acoustic neuromas are intracranial tumors arising from the Schwann cell sheath of either the vestibular or cochlear nerve. As acoustic neuromas increase in size, they eventually occupy a large portion of the cerebellopontine angle. Acoustic neuromas account for approximately 80% of tumors found within the cerebellopontine angle. The remaining 20% are principally meningiomas.

Unilateral hearing loss is the most common symptom present at the time of diagnosis and is generally the symptom that leads to diagnosis. The tumor can produce hearing loss through at least two mechanisms: (1) direct injury to the cochlear nerve or (2) interruption of cochlear blood supply. Progressive injury to cochlear fibers probably accounts for slow progressive neurosensory hearing loss observed in majority of patients with acoustic neuromas. Sudden and fluctuating hearing loss is seen in 5-15% of patients, probably due to disruption of cochlear blood supply by the tumor.
Other symptoms of acoustic neuroma include headaches (50-60% of patients at the time of diagnosis, but in less than 10% as the presenting symptom) and facial numbness (in 25% of patients but in less than 1% as the presenting symptom).
Vertigo and disequilibrium are uncommon presenting symptoms among patients with acoustic tumors. Overall, approximately 40-50% of patients with an acoustic neuroma might have some balance disturbance. However, balance disturbance is the presenting symptom in less than 10% of patients. This is due to the fact that destruction of vestibular fibers by the tumor is slow and allows for compensation.
Sudden onset of vertigo and hearing loss and contralateral sensory disturbances are strongly against acoustic neuroma as the diagnosis. In majority of patients with acoustic neuroma, the presenting symptom is progressive unilateral hearing loss.

OPTION C : Acute labyrinthitis presents with acute vertigo often followed by nausea and vomiting, tinnitus, and hearing loss. There is often a history of preceding viral upper respiratory tract infection. Change in head position provokes vertigo. Each episode of vertigo lasts from few seconds to minutes. Although there are similar symptoms of vertigo and hearing loss between labyrinthitis and vertebrobasilar insufficiency, the former is not associate with Horner’s syndrome and abnormal neurological findings such as sensory impairment.

OPTION E : Vestibular neuronitis is the inflammation of the vestibular nerve, often by a viral infection. Patients may have a preceding viral upper respiratory infection or herpes zoster. Vertigo and imbalance are the prominent features of vestibular neuronitis and there is no hearing loss or tinnitus. Loss of balance is more prominent in vestibular neuronitis compared to other causes of vertigo, and patient commonly presents with vertigo and falls. Similar to benign paroxysmal positional vertigo (BPPV), symptoms in vestibular neuronitis are aggravated by change in the position of the head. Neurological examination in patients with vestibular neuronitis is otherwise normal and there is no Horner’s syndrome.

57
Q

A 54-year-old woman presents to the emergency department with complaints of sudden onset vertigo, nausea, vomiting, and hearing loss in her left ear. On examination, her vital signs are within normal ranges. Hearing is decreased on the left side on whisper test. Rinne and Weber tests establish sensorineural deafness of the left ear. She has also nystagmus with the rapid eye to the left side. The rest of the examination is inconclusive. Which one of the following could be the most likely diagnosis?

A. Labyrinthitis.
B. Vestibular neuronitis.
C. Acoustic neuroma.
D. Meniere’sdisease.
E. Lateral medullary syndrome.

A

A. Labyrinthitis

Of the options, labyrinthitis, Meniere’s disease and lateral medullary syndrome can cause acute onset vertigo, tinnitus and hearing loss. Ataxia can be a presentation in patients with cerebellar or vestibular disease. Of these three, and given the inconclusive neurological examination, labyrinthitis is the most likely diagnosis.
Acute labyrinthitis presents with acute vertigo often followed by nausea and vomiting, tinnitus, and hearing loss. A history of preceding viral upper respiratory tract infection is present in up to 50% of patients. Change in head position provokes vertigo. Each episode of vertigo lasts from few seconds to minutes.

OPTION D : Meniere’s disease also presents with episodes of acute onset vertigo, tinnitus and hearing loss. However, patients with Meniere’s disease often complain of ear fullness because the pathophysiology is excess endolymph in the labyrinth. Patients are usually middle-aged women with a positive family history for the condition. Finally, Meniere’s disease is much less common compared to labyrinthitis. Given these, Meniere’s disease in this patient is a less likely diagnosis compared to labyrinthitis.

OPTION B : Vestibular neuronitis is the inflammation of the vestibular nerve often by a viral infection. Patients usually have a preceding viral upper respiratory infection or herpes zoster. Vertigo and imbalance are the prominent features of vestibular neuronitis and there is no hearing loss or tinnitus. Loss of balance is more prominent in vestibular neuronitis compared to other causes of vertigo, and patient commonly present with vertigo and falls. Symptoms in vestibular neuronitis are aggravated by change in the position of the head. Neurological examination in patients with vestibular neuronitis is otherwise normal.

OPTION C : Acoustic neuromas are intracranial tumors that arise from the Schwann cell sheath of either the vestibular or cochlear nerve. As acoustic neuromas increase in size, they eventually occupy a large portion of the cerebellopontine angle. Although 5-15% of patients with acoustic neuroma present with acute onset of unilateral hearing loss, deafness has an insidious onset in this condition, making it a less likely diagnosis. Gradual hearing loss is overwhelmingly the most common presenting symptom of patients with acoustic neuroma. Imbalance and vertigo is not a prominent feature because as the tumor growth disrupts the vestibular nerve function slowly, there is enough time for compensation. Other features that may be present in patients with acoustic neuroma are headache and facial sensory impairment.

OPTION E : Lateral medullary syndrome, also known as Wallenberg syndrome or posterior inferior cerebellar artery (PICA) syndrome has other clinical features in addition to vertigo, hearing loss, and tinnitus. Such features include cross-body sensory impairment (sensory impairment of the face on the affected side and that of the body on the other side), Horner’s syndrome, and signs and symptoms indicative of the involvement of cranial nerves or their nucleus. Such signs and symptoms may include dysphagia (due to involvement of nucleus ambiguus that supplies the vagus and glossopharyngeal nerves), dysarthria, dysphonia, disrupted temperature and pain sensation, palatal clonus and heart rate and blood pressure dysregulation (due to involvement of the vagus nerve).

58
Q

A 22-year-old man presents to your GP clinic for assessment. On waking up this morning, he felt pain in his left ear, associated with severe dizziness and loss of balance. He describes that the dizziness was felt as spinning sensation and giddiness followed by nausea and vomiting. The symptoms significantly improve when he lies down. He denies any hearing loss or ringing in the ears. Physical examination reveals normal vital signs. There is horizontal nystagmus with the rapid component to the right side. Caloric test shows markedly reduced response on the left side. The rest of the exam, including a full assessment of cranial nerves, is unremarkable. Which one of the following could be the most likely diagnosis?

A. Meniere’s disease.
B. Vestibular neuronitis.
C. Vertebrobasilar ischemia.
D. Paroxysmal positional vertigo.
E. Acute labyrinthitis.

A

B. Vestibular neuronitis

Acute onset of vertigo, nausea and vomiting, and being unable to keep balance in the absence of hearing loss and/or tinnitus favors either BPPV or vestibular neuronitis as the most likely etiologies. Spontaneous horizontal nystagmus with the fast part towards the unaffected site and a reduced caloric test of the left side (see TOPIC REVIEW), on the other hand, strongly suggests vestibular neuronitis as the most likely diagnosis. Another clue to vestibular neuronitis is the significant imbalance in this patient. Although all causes of vertigo can give rise to imbalance, this feature is more prominent and disabling in vestibular neuronitis compared to other causes of vertigo such as labyrinthitis, BPPV, or Meniere’s disease.

Vestibular neuronitis is the inflammation of the vestibular nerve often by a viral infection. Approximately, 50% of Patients have a preceding viral upper respiratory infection or herpes zoster infection. Vertigo and imbalance are the prominent features of vestibular neuronitis, and there is no hearing loss or tinnitus. Loss of balance is more prominent in vestibular neuronitis compared to other causes of vertigo, and patients commonly present with vertigo and falls. Symptoms in vestibular neuronitis are aggravated by change in head position. Neurological examination in these patients is otherwise normal.

Spontaneous, unidirectional, horizontal nystagmus is the most important physical finding. Fast phase oscillations beat toward the healthy ear. Nystagmus may be positional and apparent only when gazing toward the healthy ear, or during Hallpike maneuvers. Patients may suppress their nystagmus by visual fixation (a differentiating feature in nystagmus caused by central causes). Patient tends to fall toward his or her affected side when attempting ambulation or during Romberg tests. The affected side has impaired or no response to caloric stimulation.

Patients with BPPV (option D) usually wake up with the condition (like in this patient) and notice the vertigo while trying to sit up suddenly. Thereafter, propensity for positional vertigo may last for days to weeks. In many, the symptoms periodically wax and wane.

The physical examination findings in patients affected by BPPV are generally unremarkable. All neurotologic examination findings except those from the Dix-Hallpike maneuver is normal.

The Dix-Hallpike maneuver (see TOPIC REVIEW) is the standard clinical test for BPPV. The finding of classic rotatory nystagmus with latency and limited duration is considered pathognomonic. A negative test result is meaningless except to indicate that active canalithiasis is not present at that moment.

OPTION A : Meniere’s disease presents with episodes of acute onset vertigo, tinnitus, and hearing loss. Patients often complain of ear fullness. Absence of tinnitus and hearing loss in this patient make Meniere’s disease an unlikely diagnosis.

OPTION C : It is very unusual for a vertebrobasilar ischemic episode to present with vertigo as the sole symptom. There are always other clinical features in addition to vertigo such as hearing loss/ tinnitus, cross-body sensory impairment, Horner’s syndrome, and signs and symptoms indicative of the involvement of cranial nerves or their nucleus. Such signs and symptoms may include dysphagia (due to involvement of nucleus ambiguus that supplies the vagus and glossopharyngeal nerves), dysarthria, dysphonia, disrupted temperature and pain sensation, palatal clonus and heart rate and blood pressure dysregulation (due to involvement of the vagus nerve).

OPTION E : Acute labyrinthitis presents with acute vertigo often followed by nausea and vomiting, tinnitus, and hearing loss. Absence of hearing loss and tinnitus in this patient is against such diagnosis.

TOPIC REVIEW

  1. Caloric test (vestibuocular reflex test)

Caloric reflex test also termed as vestibular caloric stimulation is a test of the vestibule-ocular reflex. Ice cold or warm water or air is irrigated into the external auditory canal, usually using a syringe. The temperature difference between the body and the injected water creates a convective current in the endolymph of the nearby horizontal semicircular canal. In patients with an intact brain stem, hot and cold water produce currents in opposite directions and therefore a horizontal nystagmus in opposite directions.

If the water is warm (44 °C or above) endolymph in the ipsilateral horizontal canal rises, causing an increased rate of firing in the vestibular afferent nerve. This situation mimics a head turn to the ipsilateral side. Both eyes will turn toward the contralateral ear, with horizontal nystagmus (quick horizontal eye movements) to the ipsilateral ear.

If the water is cold, relative to body temperature (30 °C or below), the endolymph falls within the semicircular canal, decreasing the rate of vestibular afferent firing. This situation mimics a head turn to the contralateral side. The eye then turn toward the ipsilateral ear, with horizontal nystagmus to the contralateral ear.

Absent reactive eye movement suggests vestibular weakness of the horizontal semicircular canal of the side being stimulated.

One mnemonic used to remember the FAST direction of nystagmus is COWS:

COWS: Cold Opposite, Warm Same.
Cold water: FAST phase of nystagmus to the Opposite side from the cold water filled ear
Warm water: FAST phase of nystagmus to the Same side as the warm water filled ear

  1. Dix-Hallpike test (Maneuver)
    Dix – Hallpike maneuver is performed on all patients who complain of vertigo but do not exhibit nystagmus on routine examination of the extraocular muscles.
    Hallpike maneuver requires patient to lie back from sitting to supine position 3 times. The first time, have the patient lie back with the head facing forward and the neck slightly extended; repeat this movement with the patient’s head turned 45 degrees to the right and a third time with the head turned 45 degrees to the left.
    Instruct patient to keep both eyes open each time he or she lies back.
    Check for nystagmus and ask patient about any symptoms of vertigo.

Among the characteristics of an elicited nystagmus that would suggest disease of peripheral origin are a pause before nystagmus appears (latency), unidirectional nystagmus, and fatiguing of nystagmus after approximately 1 minute or repeated inductions.

Failure either to observe or to provoke unidirectional nystagmus casts doubt on whether the process is localized to the peripheral vestibular system. Either finding suggests a need to consider other diagnostic alternatives.

59
Q

A 55-year-old man presents to your GP clinic with complaints of tinnitus and hearing loss in his right ear. According to him, he suddenly felt a severe spinning sensation associated with ringing in and hearing loss of his right ear while working. He felt intensely nauseous but did not vomit. He could not stand so he lied down in bed. After five hours, the spinning partially improved and only has been coming on with head movements, but he still has the ringing ear and deafness. Except for the sensorineural deafness of the right ear and nystagmus on ocular movements, physical examination, including a full neurological exam, is unremarkable. Which one of the following could be the most likely diagnosis?

A. Labyrinthitis.
B. Vestibular neuronitis.
C. Acoustic neuroma.
D. Benign paroxysmal positional vertigo.
E. Meniere’s disease.

A

A. Labyrinthitis

The clinical picture is a classic description of acute labyrinthitis. Acute labyrinthitis is characterized by sudden, unilateral loss of vestibular function and hearing. The acute onset of severe, often incapacitating, vertigo, frequently associated with nausea and vomiting, is characteristic of this condition. The patient is often bedridden while the symptoms gradually subside. Each episode of vertigo lasts from seconds to minutes. Vertigo eventually resolves after several days to weeks; however, unsteadiness and positional vertigo may persist for several months. Hearing loss and tinnitus is common and may be the primary presenting symptom in many patients. Up to 50% of patients with acute labyrinthitis report a preceding viral upper respiratory tract infection.

OPTION B : Vestibular neuronitis and benign paroxysmal positional vertigo (BPPV) also present with vertigo and imbalance; however, tinnitus and hearing loss are not features seen in these conditions.

OPTION C : Acoustic neuromas are intracranial tumors that arise from the Schwann cell sheath of either the vestibular or cochlear nerve. As acoustic neuromas increase in size, they eventually occupy a large portion of the cerebellopontine angle. Acoustic neuromas account for approximately 80% of tumors found within the cerebellopontine angle.

Although 5-15% of patients with acoustic neuroma present with acute onset of unilateral hearing loss, deafness has an insidious onset in most cases. This fact, makes acoustic neuroma a less likely diagnosis for this patient who has presented with acute hearing loss. Gradual hearing loss is overwhelmingly the most common presenting symptom of patients with acoustic neuroma. Imbalance and vertigo is not a prominent feature because as the tumor disrupts the vestibular nerve function slowly, there is enough time for compensation. Other features that may be present in patients with acoustic neuroma are headache and facial sensory impairment.

OPTION E : Meniere’s disease can also cause vertigo, tinnitus and hearing loss; however, it is an uncommon cause of vertigo. Furthermore, it is more common in middle-aged women. Family history is often positive in patients with Meniere’s disease.

60
Q

A 67-year-old man is brought to the Emergency Department by his wife after he suddenly blacked out in the bathroom while urinating and regained consciousness in few seconds. He is diabetic and on antihyperglycemic medications. On assessment, there is no remarkable point except urinary symptoms of hesitation and terminal dribbling. Which one of the following could be the most likely cause of this presentation?

A. Epilepsy.
B. Vasovagal syncope.
C. Micturition syncope.
D. Hypoglycemia.
E. Transient ischemic attack (TIA).

A

C. Micturition syncope

The scenario describes an episode of syncope. Syncope is defined as an episode of sudden loss of consciousness and postural tone with spontaneous recovery. Presyncope refers to symptoms of faintness or lightheadedness without loss of consciousness. Any condition that leads to loss of oxygen or glucose to the brain can lead to syncope. Almost always, syncope results from loss of cerebral blood flow. Even syncope secondary to hyperventilation results from hypocapnia-induced cerebral vasoconstriction.

Although other conditions such as epilepsy or cerebral trauma can also cause sudden loss of consciousness and postural tone with spontaneous recovery, in practice the term syncope is reserved for transient reduction in cerebral blood flow, while other causes are considered ‘syncope mimics’.

Reflex syncope, also termed as neurally-mediated or neurocardiogenic syncope, accounts for at least one-third of syncopal episodes, and is the most common cause of syncope. As these terms imply, the nervous system rather than cardiac problems is the root cause of the problem. Orthostatic syncope is the second most common cause of syncope.

Reflex syncope includes:
1. Vasovagal syncope (common faint)
2. Carotid sinus hypersensitivity
3. Situational syncope i.e., cough syncope, micturition syncope, swallowing syncope and defecation syncope
4. Receptor syncope

With urinary symptoms of difficulty starting urination and terminal dribbling, micturition syncope is the most likely diagnosis. Micturition syncope or post-micturition syncope is fainting shortly after or during urination.

Micturition syncope is commonly seen in elderly men with a large prostate that prevents normal urine flow. When the patient strains to overcome the obstruction, intraabdominal pressure rises and leads to increased vagal tone and consequent bradycardia, drop in blood pressure, and cerebral hypoperfusion.

Micturition syncope is categorized under the entity ‘situational syncope’ which belongs to ‘reflex-mediated’ causes of syncope.

Situational syncope is an uncommon cause of syncope related to specific situations such as urinating, coughing or swallowing. The mechanism through which syncope is triggered may or may not differ from that of vasovagal syncope. Cough syncope is postulated to be caused be decreased blood flow to the brain due to increased intrathoracic pressure during bouts of coughing, while micturition syncope is caused by vasovagal hyperstimulation.

OPTION A : Epilepsy is the term used for seizures for which no apparent cause is found.

OPTION B: Although micturition syncope is caused by vasovagal response, it is independently categorized as ‘situational’ rather than ‘vasovagal’ syncope.

OPTION D : In fainting due to hypoglycemia, the regain of consciousness is not quick. Furthermore, there is usually presyncopal prodrome.

OPTION E : TIA is associated with focal neurological deficits rather than syncope.

TOPIC REVIEW

Causes of syncope (including ‘syncope mimics’):

  1. Reflex-mediated (most common)
    -Vasovagal syncope (common faint)
    -Carotid sinus hypersensitivity
    -Situational syncope (cough, micturition, swallowing, defecation)
    -Receptor stimulation (cranial nerve irritation, bladder distension, airway stimulation, neck tumor)
  2. Orthostatic hypotension (second most common)
    -Drug induced
    -Hypovolemia/hemorrhage
    -Deconditioning
    -Autonomic dysfunction: primary (pure autonomic failure, multiple system atrophy, etc.), secondary (diabetic, paraneoplastic neuropathies, amyloidosis)
  3. Cardiac (electrical)
    -Bradycardias (sinus node dysfunction or AV-conduction-system disease)
    -Tachycardias (supraventricular or ventricular)
  4. Cardiac (mechanical)
    -Valvular disease: aortic, mitral or pulmonary stenosis
    -Hypertrophic obstructive cardiomyopathy
    -Aortic dissection
    -Pulmonary embolus
    -Pericardial disease/cardiac tamponade
    -Primary pulmonary hypertension -Cardiac tumors
    -Critical myocardial ischemia
  5. Non-cardiac (predominantly syncope mimics)
    -Neurological/cerebrovascular: seizure, migraine, subarachnoid hemorrhage, subclavian steal syndrome, vertebrobasilar insufficiency
    -Metabolic disturbances: hypoglycemia or hypoxemia, pheochromocytoma, Addison’s disease, drug intoxication including alcohol, hyperventilation (via hypocapnia)
  6. Psychogenic pseudo-syncope
    -In up to one-third of cases, no diagnosis is made despite thorough investigation. It is very likely that most of these undiagnosed cases are due to reflex-mediated syncope or orthostatic hypotension.
61
Q

A 21-year-old woman is in your GP clinic with complaint of recurrent episodes of vertigo. She describes that she has had these episode every month for the past five years. The vertigo is followed by imbalance and sometimes nausea and vomiting. She denies any hearing impairment or ringing in the ear, or ear fullness during the attacks. She is otherwise healthy and does not drink alcohol or smoke. Physical examination is unremarkable. Which one of the following would be the most appropriate management for her?

A. Refer her to ENT specialist.
B. Reassure her that this is a benign condition.
C. Tempanostomy.
D. Prescribe decongestants.
E. MRI of the posterior fossa.

A

B. Reassure her that this is a benign condition.

Recurrent acute attacks of vertigo associated with nausea and vomiting without tinnitus or hearing impairment in a young otherwise healthy young woman is most likely due to benign paroxysmal positional vertigo (BPPV). Although BPPV is more common in middle-aged people, it can affect any age group.

BPPV is the most common cause of vertigo in clinical practice. The condition is caused by an accumulation of calcium crystals in the posterior semicircular canal. These crystals affect the movement of the endolymph in the semicircular canals and causes vertigo. The classic symptoms of BPPV are brief episodes of vertigo (often lasting for few seconds), associated with nausea and nystagmus. There is no hearing impairment or tinnitus.

Episodes of vertigo are triggered by rapid changes in the position of the head. Symptoms may persist for weeks and recur after remission. Nystagmus seen in BPPV has a rotational nature. The Dix−Hallpike maneuver is used to diagnose BPPV. The Epley manoeuvre can be used to treat it.
As the name implies, BPPV is a benign condition and the patient can be reassured. Treatment of BPPV is with Epley maneuver, vestibular rehabilitation and watchful waiting.

OPTION A: Referral to ENT specialist is not required at this stage as the diagnosis is almost certain and no further action other than simple measures is required.

OPTION D: Although antihistamines such as betahistine can provide modest relief in patients with BPPV and are often prescribed for short term use, they are not the mainstay of the treatment and do not have a significnt role in treatment of BPPV.

OPTION C: Tempanostomy is used for relief of pressure in the middle ear. This procedure is not useful for treatment of BPPV, the etiology of which lies withing the inner ear.

OPTION E : With a diagnosis of BPPV as a peripheral cause of the vertigo in this patient, MRI or other imaging studies are not indicated. Imaging studies are often considered when a central nervous system lesion is suspected.

62
Q

A 30-year-old woman presents for evaluation of vertigo of three months duration. She also complains of constant ringing of her ears. She is very fond of music and has her earphones on through the day. She has no significant past medical or surgical history, is not on any regular medications, and otherwise enjoys a healthy life. Her family history is remarkable for hearing impairment in one of her maternal aunts. Whispering test shows reduced hearing in the left ear. Which one of the following is the most appropriate option to reach a diagnosis?

A. Rinne and Weber tests.
B. Speech discrimination test.
C. Electrocochleography.
D. Pure tone audiometry.
E. MRI.

A

D. Pure tone audiometry

For every patient presenting with suspected hearing loss, tinnitus, vertigo and other ear symptoms, an audiogram is indicated to evaluate the condition, especially if a clear diagnosis of a benign self-limiting condition such as vestibular neuronitis, acute labyrinthitis, or benign paroxysmal positional vertigo (BPPV) cannot be made based on the history and physical examination alone. Audiometry is also useful for screening for hearing loss in people regularly exposed to loud noises, and for certain patients on ototoxic medications (e.g., gentamicin).

Pure tone audiometry is the standard test to assess hearing loss in adult patients. It allows for independent thresholds to be determined in each ear, for air conduction (conductive hearing) and bone conduction (sensorineural hearing). Air conduction measures the ability of the external and middle ear to transmit sound to the cochlea. Conductive hearing loss (CHL) results from any barriers that inhibit sound transmission along this pathway, which may include cerumen impaction, middle ear fluid, or a tympanic membrane perforation. On the audiogram, CHL is depicted as an air-bone gap. Sensorineural hearing loss (SNHL) is defined by equal air and bone conduction thresholds higher than 25 Db. Tympanometry can complement the pure tone audiogram by assessing the compliance and mobility of the middle ear system.

OPTION A: Rinne and Weber test are office tests for a provisional assessment of hearing loss. In Rinne test bone and air conductions are compared. A vibrating tuning fork is placed over the mastoid bone and the patient is asked to signal the examiner once he/she cannot hear the vibration. On the signal, the examiner brings the fork in front of the ear and asks the patient if he/she could still hear the vibration. In a normal person air conduction (AC) is better than (BC); therefore, the vibration should still be heard when the fork is brought to in front of the ear. If not, BC is better than AC in that ear and CHL is suspected.

In Weber test, a vibrating tuning fork is placed in the middle of the patient’s forehead. In CHL the sound is lateralized to the affected ear (louder in the affected ear) while in SNHL the sound is louder in unaffected ear.

Although these tests can assess the hearing loss and differentiate CHL from SNHL to some extend, they are not very reliable compared to formal audiometry because for example in patients with bilateral and equal SNHL Rinne and Weber tests could be completely normal as no lateralization occurs. None of these tests replace the need for pure tone audiometry for assessment of CHL versus SNHL and the extent of impairment.

OPTION B: Speech discrimination test is performed as a part of formal audiometry. Speech discrimination scores are a measure of the patient’s ability to hear words correctly and are a good indication of the integrity of the cochlear nerve. There is a predictable attenuation with a conductive or a cochlear loss, and if there is an acoustic neuroma the speech discrimination is much worse than expected from the pure tone audiogram. Improvement in speech discrimination scores with increased intensity is also a good way of predicting if a patient will benefit from a hearing aid or other amplification.

OPTION C : Electrocochleography is an objective measure of the electrical potentials generated in the inner ear as a result of sound stimulation. This test is most often used to determine if the cochlea has an excessive amount of fluid pressure. Excessive fluid pressure in the cochlea can cause symptoms such as hearing loss, aural fullness, dizziness, and/or tinnitus. These symptoms are sometimes indicative of certain ear pathologies such as Meniere’s disease or endolymphatic hydrops. This test might be later considered for this patient if her symptoms are suspected to have been caused by such pathology.

OPTION E: Imaging studies such as magnetic resonance imaging (MRI), computed tomography (CT) or magnetic resonance angiography (MRA) in patients with vertigo and/or hearing loss are indicated if:
-the examination is not consistent with a peripheral lesion prominent risk factors for CVA are present
neurological signs and symptoms are present, or symptoms of vertigo are accompanied by a headache
At this stage, this patient does not have any of the above to necessitate MRI.

63
Q

Ben is in your clinic for assessment. He is 60 years old, and at several occasions in the past month has had vertigo, right sided hearing loss and double vision as well as numbness of his left limbs. These episodes last for up to 5-10 minutes and resolve spontaneously. He smokes 10-15 cigarettes and drinks 2-3 standard drinks every day. His past medical or surgical history is insignificant and he is not on any regular medication. He also denies any recent flu-like symptoms. His family history is remarkable for Meniere’s disease in his mother. Which one of the following could be the most likely diagnosis?

A. Multiple sclerosis.
B. Vertebrobasilar insufficiency.
C. Meniere’s disease.
D. Labyrinthitis.
E. Meningioma.

A

B. Vertebrobasilar insufficiency

Episodes of vertigo, right-sided hearing loss, double vision and most importantly simultaneous numbness of the left limbs are highly suggestive of vertebrobasilar insufficiency as the most likely diagnosis. Vertebrobasilar circulation supplies the medulla, cerebellum, pons, midbrain, thalamus, and occipital cortex.

Characteristic features of vertebrobasilar insufficiency/ stroke include the following:
-Vertigo, nausea, and vomiting, along with nystagmus, represent involvement of the vestibular system and are seen in vertebrobasilar strokes.
-Unilateral Horner syndrome occurs with brainstem lesions.
-Cerebellar signs (e.g., dysmetria, ataxia) are frequently observed.
-Dysarthria and dysphagia are typically present.
-When cranial nerves or their nuclei are involved, the corresponding clinical signs are ipsilateral to the lesion and the corticospinal signs are crossed, involving the opposite arm and leg.
-Involvement of the ascending sensory pathways may affect the spinothalamic pathway or the medial lemniscus (dorsal columns), resulting in dissociated sensory loss, which is loss of one sensory modality on one side and preservation of other sensory modalities in the opposite limbs (dissociative sensory loss).
-Occipital lobe lesions result in visual field loss or visuospatial deficits.
-Cortical deficits, such as aphasia and cognitive impairments, are absent.

OPTION A : Multiple sclerosis (MS) can cause a variety of neurological symptoms; however, it is unusual for MS to affect the same part of the nervous system with each relapse. MS relapses characteristically are described as CNS demyelinating lesions that are separate in time and location.

With MS, there is involvement of one particular part of the CNS and the corresponding symptomatology at one time that completely or partially resolves and involvement of another part of the CNS with next relapse. Furthermore, symptoms last for days to weeks before they improve. This clinical scenario is not consistent with MS.

OPTION B : Meniere’s disease present with recurrent episodes of hearing loss, tinnitus, and vertigo as well as a sense of ear fullness that last for hours. Additionally, there are no other focal neurological findings such as numbness because the pathology is within the inner ear (increased endolymph pressure in labyrinth).

OPTION D: Labyrinthitis causes vertigo, hearing loss and tinnitus in episodes lasting from seconds to minutes. However, there is no focal neurological findings in labyrinthitis.

OPTION E : Meningiomas are non-cancerous CNS tumors that arise from meningeal lining within the cranium or spinal canal. The symptoms are caused by irritation of the underlying brain or spinal tissue and/or compression effect. Symptoms have an insidious onset and the presentation depends on the specific location they arise from within the CNS. Headache is a common feature in intracranial meningiomas.

Of the cerebellopontine angle tumors, almost 20% are meningiomas (80% are acoustic neuromas). Unilateral gradual sensorineural deafness is overwhelmingly the cardinal presenting symptom. However, imbalance, vertigo, or facial numbness can be the presenting feature in a minority of patients. Limb paresthesia is not a presentation. More importantly, patients with intracranial tumors have continuous symptoms that progress with time. The presence of this patient’s symptom in short periods and being asymptomatic in between is strongly against meningioma as the diagnosis.

64
Q

A 30-year-old woman presents to your general practice with complaint of blurred vision and pain in the right eye for the past 3 days. She is otherwise healthy and has not had any major medical or surgical history. On examination, her visual acuity of the right and left eyes are 6/12 and 6/6, respectively. She cannot read the numbers in Ishihara chart and confuses red and green. Other significant finding on examination is worsening of the right eye pain with eye movements that are normal and symmetric in all directions with no diplopia. Which one of the following could be the most likely diagnosis?

A. Intracranial hemorrhage.
B. Orbital tumor.
C. Optic neuritis.
D. Temporal arteritis.
E. Retinal detachment.

A

C. Optic neuritis

Sudden onset of decreased vision and retro-orbital pain in a young female is highly suggestive of optic neuritis as the most likely diagnosis.

Optic neuritis (ON) is a demyelinating inflammation of the optic nerve that often occurs in association with multiple sclerosis (MS) and neuromyelitis optica (NMO). A gradual recovery of visual acuity with time is characteristic of ON; however, permanent residual deficits in color vision and contrast and brightness sensitivity are common.

Typically, patients with first time acute optic neuritis (ON) are otherwise healthy young adults, who experience rapidly developing impairment of vision in one eye or, less commonly, both eyes during an acute attack. Dyschromatopsia (change in color perception) in the affected eye occasionally may be more prominent than the decreased vision. Decreased color vision is tested with Ishihara chart shown in the following picture. Patients with impaired color vision cannot read the numbers in the chart due to color blindness.

In almost all cases, the visual changes are associated with a retro-orbital or ocular pain, usually aggravated by eye movement. The pain may precede the visual loss.

Patients may complain of vision loss exacerbated by heat or exercise (Uhthoff phenomenon). Objects moving in a straight line may appear to have a curved trajectory (Pulfrich phenomenon), presumably due to asymmetrical conduction between the optic nerves.

Physical examination findings in patient with ON may include:
-Decreased visual acuity
-Decreased color vision
-Presence of relative afferent pupillary defect
-Reduction in subjective brightness
-Visual field defect
-Abnormal contrast sensitivity
-Certain appearances of the optic nerve (swelling, peripapillary hemorrhage or pallor)

OPTION A : Intracranial hemorrhage is not likely to present with isolated findings of reduced vision and retro-orbital pain worsening on eye movement. In patients with intracranial hemorrhage, more significant motor and/or sensory disturbances would be expected.

OPTION B : An orbital tumor presents differently with probable findings of eye bulging, decreased vision, impaired visual fields, and manifestations of raised intraocular pressure. It is less likely that an intraorbital tumor is missed on funduscopic examination in experienced hands. Vision loss and retro-orbital pain are unlikely to be the sole manifestations of an eye tumor.

NOTE - Uveal melanoma is the most common intraocular malignancy in adult.

OPTION D : Temporal arteritis is a vasculitis of large vessels of middle-age onset and characterized by a variety of symptoms including headache, scalp tenderness, jaw claudication and vision loss. Polymyalgia rheurmatica can be an association. Neither the patient’s age, nor the clinical presentation is consistent with temporal arteritis as the diagnosis.

OPTION E: Although retinal detachment is associated with vision loss, absence of other features like flashes and floaters in the visual fields and the presence of pain makes such diagnosis unlikely.

65
Q

A 38-year-old woman presents with complaints of decreased visual acuity and retro-orbital pain worse on eye movements on the left side for the past four days. On examination, there is vision loss, color vision in particular, and painful eye movements. Which one of the following is the most appropriate next investigation to consider for her?

A. Temporal artery biopsy.
B. MRI.
C. CT scan of the head.
D. Visual evoked potentials.
E. Gonioscopy.

A

B. MRI

Clinical manifestations of decreased visual acuity, especially the color vision, and retro-orbital pain worse on eye movements are highly suggestive of optic neuritis (ON) as the most likely diagnosis.

ON is acute inflammation of the optic nerve, and the most commonly optic neuropathy encountered in general practice. Demyelination associated with multiple sclerosis (MS) is the most common cause of ON in adults. Other causes include infections, parainfections (infection source elsewhere other than the optic nerve) and rheumatologic diseases.

Decreased vision, especially color vision, and retro-orbital pain aggravated by eye movements are the cardinal symptoms of ON. On physical examination of the eye, the following features can be present:
-Decreased visual acuity
-Decreased color vision
-Presence of relative afferent pupillary defect
-Reduction in subjective brightness
-Visual field defect
-Abnormal contrast sensitivity, and
-Certain appearances of the optic nerve (swelling, peripapillary hemorrhage or pallor)

For patients with suspected ON, magnetic resonance imaging (MRI) is the most appropriate investigation to consider. MRI is highly sensitive and specific in assessing inflammatory changes in the optic nerves and helps to rule out structural lesions. In addition, MRI may have a value in predicting future development of MS in patients presenting with first-time, acute optic neuritis. Patients with optic neuritis, who also have demyelinating brain lesions on MRI, are more likely to develop MS in the future. MRI should be considered as the most important initial investigation for all patients presenting with clinical manifestations of optic neuritis.

OPTION E : Visual evoked potentials (VEP) is another important test to consider, especially if the diagnosis is uncertain. Patients with ON or MS will have a delay in latency with preserved waveforms in conventional VEP. In ON, VEP can be abnormal when MRI is negative and inconclusive. VEP can confer the highest diagnostic yield in patients with suspected ON and is often performed in such patients. However, MRI is always the first diagnostic modality to consider.

OPTION A : Temporal artery biopsy is the definitive diagnostic test for temporal arteritis (giant cell arteritis). Temporal arteritis presents with scalp tenderness, jaw claudication and vision loss (due to involvement of the retinal branch). The condition usually occurs in older patients (>50 years). This patient is young and does not have other features of temporal arteritis; hence, temporal artery biopsy is not necessary for her.

OPTION C : CT scan of the head has no diagnostic value for suspected ON.

OPTION E : Gonioscopy is the examination of the anterior chamber, most commonly for assessment of the anterior chamber angle. It adds no diagnostic benefit for patients with ON.

66
Q

A 35-year-old woman presents with complaints of decreased vision and pain of her right eye for the past three days. Eye examination reveals decreased visual acuity of the right eye and retro-orbital pain worse on eye movements. Which one of the following investigations is most likely to establish a diagnosis for her?

A. Temporal artery biopsy.
B. Lumbar Puncture(LP).
C. CT scan of the head.
D. Visual evoked potentials.
E. Fundoscopy.

A

D. Visual evoked potentials

Sudden-onset decreased vision and retro-orbital pain in a young female is highly suggestive of optic neuritis (ON) as the most likely diagnosis.

For patients with suspected ON, MRI of the brain is always the most initial investigation to consider. MRI is highly sensitive and specific in assessing inflammatory changes in the optic nerve and helps to rule out structural lesions. In addition, MRI may have a value in predicting future development of multiple sclerosis (MS) in patients presenting with first-time acute optic neuritis. Patients with optic neuritis, who also have demyelinating brain lesions on MRI, are more likely to develop MS in the future. MRI should be considered as the most important initial investigation for all patients presenting with clinical manifestations of optic neuritis. However, when it comes to diagnostic accuracy, visual evoked potentials (VEP) can give a more sensitive diagnostic yield compared to MRI. In ON, VEP can be abnormal when MRI is negative and inconclusive. Patients with ON or MS will have a delay in latency with preserved waveforms in conventional VEP.

OPTION A : Temporal artery biopsy is the definite diagnostic test for patients with suspected temporal arteritis. Temporal arteritis presents with scalp tenderness, jaw claudication, and vision loss (due to involvement of the retinal branch). The condition usually occurs in older patients (>50 years). This patient is young and does not have other features of temporal arteritis; hence, temporal artery biopsy is not necessary for her.

OPTION B : Lumbar puncture (LP) is now used less frequently and its use is controversial. However, cerebrospinal fluid (CSF) examination is useful in patients with normal or atypical brain MRI. The typical finding is the presence of oligoclonal bands (69% of patients with ON).

OPTION C : CT scan of the head has no diagnostic value in patients with suspected ON.

OPTION E : In almost 65% of patients, the pattern of ON is retrobulbar; therefore, no abnormality is noted on fundoscopic examination. In the remaining 35% of patients with anterior ON (involvement of the optic nerve head), optic nerve swelling or papillitis can be seen. Other possible findings in these patients are retinal exudates or peripapillary hemorrhage. Given the low sensitivity of funduscopy in ON, it cannot be relied on for diagnosis.

67
Q

A 26-year-old woman presents with history of recurrent visual problems. She describes that she frequently experiences seeing glimmering flashes in her vision for about 10 to 15 minutes each time which are completely resolved every time. She is also experiencing global headaches which worsens on exertion. On examination, visual acuity, visual fields, eye movements and pupillary reflexes are normal. Fundoscopic examination is inconclusive. There are no focal neurological deficits on central nervous system examination either. Which one of the following could be the most likely diagnosis?

A. Focal epilepsy.
B. Occipital lobe tumor.
C. Optic neuritis.
D. Migraine with aura.
E. Carotid artery stenosis.

A

D. Migraine with aura

The history of visual symptoms lasting 10-15 minutes with complete resolution and headache aggravated by exercise are consistent with migraine with aura as the most likely diagnosis among the options. Although migraine headache often tends to be unilateral first, some patients may have generalized (global) headache. Significant pieces of information in the history suggesting migraine with aura as the probable diagnosis are the presence of visual disturbances in form of glimmering flashes indicating aura and exacerbation of the headache with exertion. Exertion can trigger and/or aggravate migraine headache.

In migraine, auras most commonly consist of visual symptoms, which may be negative or positive. Negative symptoms include negative scotoma or negative visual phenomena, such as the following:
-Homonymous hemianopic or quadrantic field defects
-Central scotomas
-Tunnel vision
-Altitudinal visual defects
-Complete blindness

The most common positive visual phenomenon is the scintillating scotoma. This consists of an arc or band of absent vision with a shimmering or glittering zigzag border. The disturbance begins in the paracentral area, and gradually enlarges and moves across the hemifield, eventually breaking up and resolving. It is often combined with photopsias (uniform flashes of light) or visual hallucinations, which may take various shapes.

Other forms of aura include paresthesias (the next most common form of aura and seen in 40% of patients) and motor symptoms. The typical pattern is numbness that starts at hands, migrates to the arm, and then involves the face, lips, and tongue.

Motor symptoms may occur, and usually are associated with sensory symptoms. Motor symptoms often are described as a sense of heaviness of the limbs before a headache but without any true weakness. Speech and language disturbances have been reported in 17–20% of patients. These disturbances are commonly associated with upper extremity heaviness or weakness.

NOTE - Auras may or may not be followed by the headache.

The headache often is initially unilateral and localized in the frontotemporal and ocular area, but pain can be felt anywhere around the head or neck. The pain typically builds up over a period of 1–2 hours, progressing posteriorly and becoming global. The headache typically lasts from 4–72 hours. Among females, more than two thirds of patients report attacks lasting longer than 24 hours.

Pain intensity is moderate to severe and intensifies with movement or physical activity. Many patients prefer to lie quietly in a dark room. The pain usually subsides gradually within a day and after a period of sleep. Most patients report feeling tired and weak after the attack.

OPTION A : Focal epilepsies can present with a variety of symptoms. Visual auras such as flashes, glimmering lights, etc. are common aural phenomena. With the focus of the epilepsy in the visual cortex, visual disturbances or hallucinations can be the presentation of the epilepsy itself and not just the aura. However, a headache that worsens with exertion more favors the diagnosis of migraine with aura rather than focal epilepsy. Moreover, migraine attacks are more common than focal epilepsies in terms of commonality.

OPTION B : In brain tumors, the neurological findings are usually fixed and not intermittent. Although occipital lobe tumor can cause flashes, the intermittent nature of the flashes in this patients makes an occipital tumor a less likely explanation.

OPTION C : Decreased visual acuity, impairment in color perception, and retro-orbital pain that worsens with eye movements are typical clinical presentations in optic neuritis (ON). Patients with ON may also experience diminished perception of contrast, visual field deficits and flashes in the eye. Intermittent flashes and headaches that are worse with exertion. Intact visual acuity and visual fields, as well as no painful eye movements in this patient are against ON as the diagnosis.

OPTION E : The most common visual disturbance in patients with carotid artery stenosis is amaurosis fugax, which is transient unilateral vision loss.
Amaurosis fugax is described by the patients as a curtain coming down in front of their eye. While flashes can be a symptom seen in patients with vertebrobasilar insufficiency, it is not a feature in carotid artery stenosis.

68
Q

A 65-year-old man presents to a GP clinic for what he believe is vertigo. At work, when he stood up from a sitting position, he felt dizzy and nauseated. He feels ok now and does not have the symptoms anymore. He is a known case of diabetes and ischemic heart disease, and is currently on valsartan, metoprolol, atorvastatin, metformin and glimepiride. On physical examination, his vital signs are within normal limits and the rest of the examination including a full neurological exam is unremarkable. Laboratory tests are only significant for a random serum glucose level of 11 mmol/L and HbA1C level of 7%. An ECG is obtained which is normal. Which one of the following is the most likely diagnosis?

A. Stroke.
B. Myocardial infarction.
C. Diabetes mellitus.
D. Drug-induced hypotension.
E. Ear problems.

A

D. Drug-induced hypotension

Vertigo is a common, distressing presentation in general practice and constitutes approximately 54% of cases presenting with dizziness. Vertigo in a sensation of movement of the environment around the patient. Often, patients describe a ‘spinning’ sensation of either their body or their surroundings. This sensation can be confused with dizziness, which is a non-specific term; therefore, adequate history is required to differentiate dizziness from true vertigo.

Dizziness can be classified into four categories:
1. Vertigo (spinning sensation)
2. Disequilibrium (feeling of imbalance)
3. Light-headedness (sensation of giddiness)
4. Presyncope (faint).

In this man, what he believes was vertigo has occurred after a change in position from sitting to standing. On the other hand, he is on the blood pressure lowering agents valsartan (angiotensin receptor blocker [ABR]) and metoprolol (beta blocker). With all these in the history and the fact that there are no other symptoms at present, this patient is very likely to have experienced light-headedness or presyncope due to a sudden drop in his blood pressure while standing up from a sitting position because of his antihypertensive medications.

For this patient, blood pressure should be recorded in both supine and standing positions to see if a postural drop in hypotension (orthostatic hypotension) follows when he stands from supine position. A drop of at least 20 mmHg in systolic or 10 mmHg in diastolic blood pressure confirms this diagnosis with high certainty.

Common symptoms of orthostatic hypotension can include dizziness, lightheadedness, blurred vision, weakness, fatigue, nausea, palpitations, and headache. Syncope, dyspnea, chest pain and neck and shoulder pain are less common symptoms.

OPTION A : With stroke, the patient should have focal neurological abnormalities on physical examination. There is no limb weakness, paresthesia, clinical features of involvement of cranial nerves or any other indicators of stroke.

OPTION B : Myocardial infarction presents with chest pain as the cardinal symptom. Other symptoms may include shortness of breath and lightheadedness due to impaired cardiac output. Although myocardial infarction (MI) can be painless in patients with diabetes mellitus, absence of chest pain in history and the normal ECG exclude this diagnosis.

OPTION C : Longstanding diabetes can result in autonomic dysregulation. Patients with autonomic dysregulation from diabetes experience symptoms related to heart rate and blood pressure and bowel motility abnormalities. These symptoms may include bradycardia, tachycardia, orthostatic hypotension, diarrhea or constipation.

Although diabetes can result in impaired blood pressure regulation independently, in this patient and in the presence of blood pressure lowering agents, hypotension from medications seems more likely as the cause of such episode. Autonomic dysregulation from the diabetes, if present, can act as a predisposing factor.

OPTION E : Inner ear problems can cause true vertigo. Benign paroxysmal positional vertigo, vestibular neuronitis, labyrinthitis and Meniere’s disease are diseases originating from inner ear that can cause vertigo. Episodes of vertigo in vestibular neuronitis and labyrinthitis last from seconds to minutes. Vestibular neuronitis is associated with severe and almost debilitating spinning sensation. The history of dizziness for a few seconds with complete resolution and no residual symptoms is against vestibular neuronitis. Absence of hearing impairment and ringing ears (tinnitus) strongly excludes labyrinthitis and Meniere’s disease. BPPV can be a possibility as it presents with acute episode of vertigo often followed by nausea and intact hearing with every episode of vertigo lasting only few seconds. However, given the drug history and the fact that the dizziness has occurred just once and was not present afterwards or reproduced during the physical examination, this diagnosis is less likely compared to drug-induced postural hypotension.

NOTE - If in history, the dizziness was described as true vertigo (rotation or spinning sensation), BPPV would be a better diagnosis because postural hypotension cannot cause a spinning sensation.

69
Q

A 40-year-old man presents to your office with complaints of vertigo and shaky hands every morning after he wakes up. His medical history is significant for long-standing heavy alcohol drinking. When you ask him about the nature of what he describes as vertigo, he says: “like I am floating, and my head is light”. He denies any sensation of spinning or moving around. Which one of the following could be the most likely diagnosis?

A. Temporal lobe epilepsy.
B. Anxiety.
C. Cerebellar degeneration.
D. Benign paroxysmal positional vertigo(BPPV).
E. Optic neuritis.

A

B. Anxiety

One of the chief complaints of this patient, is dizziness. It is very important to differentiate between dizziness or lightheadedness (pseudovertigo) and true vertigo.

Vertigo is caused by a variety of etiologies, widely categorized as peripheral and central. While central vertigo is caused by a central nervous system (CNS) pathology, peripheral vertigo is a presentation of the dysfunction of the peripheral component of the balance system such as vestibular nerve, labyrinths, and utricle and saccule. The majority of vertigoes are have a peripheral etiology.

This patient mentions dizziness but denies any spinning or movement sensation. This indicates that he has dizziness rather than true vertigo. Of the options, only anxiety can justify such presentation. This patient is alcohol dependent and is likely to be experiencing withdrawal symptoms including anxiety after waking up in the morning. Anxiety can cause dizziness due to hyperventilation and also hand tremors.

OPTION A : Temporal lobe epilepsy is characterized by a preceding aura (now called focal aware) including auditory, olfactory or gustatory illusions, hot-cold sensation, and somatosensory, vestibular or visual disturbances. Other feature of focal aware include heart rate regulation abnormalities (tachycardia, bradycardia r asystole), pallor, piloerection, and flushing, or gastrointestinal disturbances (e.g. the sense of a rising of or presence of butterflies in the stomach as explained by patients). Cognitive/ psychic phenomena (e.g., Déjà vu or jamais vu, dissociation, depersonalization or derealization, forced thinking, and aphasia/dysphasia), mood and affect lability (e.g., agitation, aggression, anger, anxiety, fear, paranoia, pleasure, crying
or laughing) may be present. This aural phase is followed by motionless stare, dilated pupils, and behavioral arrest. During an attack there can be eye blinking, alimentary, manual or unilateral dystonic limb posturing. There is often a postictal period with confusion, aphasia, or amnesia. This patient is unlikely to be suffering from temporal lobe epilepsy.

OPTION C : Cerebellar degeneration associated with alcohol misuse mainly presents with ataxia. Almost all patients have gait abnormalities which is typically characterized as weakness, unsteadiness, or incoordination in legs. Later, a minority of patients may develop incoordination and tremor in the upper arm, dysarthria, and intermittent diplopia, or blurred vision. Vertigo, tinnitus, and hearing loss have not been reported in patients with alcohol-related cerebellar degeneration. With dizziness and hand tremors in the morning as the only complaints and in the absence of gait abnormalities, this patient is unlikely to have cerebellar degeneration.

OPTION D : BPPV is characterized by spells of vertigo on head movements and body position. This patient does not have vertigo. Moreover, tremor is not a feature of BPPV. Presence of tremor in the history makes BPPV an unlikely diagnosis.

OPTION E : Optic neuritis is the demyelinating inflammation of the optic nerve. Multiple sclerosis is the most common associated condition, but it can be seen in other conditions such as diabetes as well. Symptoms include retro-orbital pain brought on by eye movements, partial or complete vision loss and blurred vision. None of these features are present in this patient. On the other hand, optic neuritis does not cause dizziness/vertigo and/or tremors.

Different causes of true vertigo are as follows:

  1. Peripheral causes
    -Labyrinthitis.
    -Vestibule neuronitis.
    -Meniere’s disease
    -Benign paroxysmal positional vertigo (BPPV)
    -Chronic suppurative otitis media (CSOM)
    -Cervical spondylosis with or without radiculopathy
  2. Central causes
    -Transient ischemic attack (TIA) / stroke e.g. PICA syndrome Complex migraine
    -Multiple sclerosis (MS)
    -CNS tumors (e.g. acoustic neuroma)
  3. Other causes
    -Medications/ drugs Idiopathic/ iatrogenic
    -Psychogenic
    -Trauma
70
Q

A 6-year-old boy is brought to your practice by his parents with recurrent episodes of headache for the past few weeks. He has frequent episodes almost every week. The headache is often associated with visual disturbances and accompanied by vomiting at times. After sleeping, the headache resolves. Which one of the following is the most appropriate drug for management of acute episodes?

A. Aspirin.
B. Paracetamol.
C. Sumatriptan.
D. Non-steroidalanti-inflammatorydrugs (NSAIDs).
E. Propranolol.

A

B. Paracetamol

With recurrent episodes of headache associated with visual disturbances (aura), nausea and vomiting, and resolution after sleep, migraine with aura (classic migraine) is the most likely diagnosis.

In children, a stepwise management of migraine headaches starts with simple analgesic. Paracetamol either alone or in combination with NSAIDs such as ibuprofen as the most appropriate option. Over recent years there has been an increasing use of medications from the triptans group, most commonly sumatriptan. These agents work through 5-HT (1B/1D) receptors by three mechanisms:
1. Promoting vasoconstriction of painfully dilated cerebral blood vessels
2. Inhibiting the release of vasoactive neuropeptides by trigeminal nerves
3. Inhibiting nociceptive neurotransmission.

The efficacy of triptans such as sumatriptan has been established in several large studies and recently there has been more evidence for their safety and effectiveness in the adolescent population. In the pediatric population (less than 12 years of age), triptans have been shown to be reasonably safe but not very effective in the termination of recurrent migraines. Different formulations are available, including nasal sprays and sublingual wafers, and it is likely that these will become a mainstay of migraine termination treatment, especially in the adolescent population.

As this is the first presentation, and the patient is a child less than 12 years of age, and not an adolescent, the best choice would be paracetamol. However, if paracetamol and NSAIDs fail to control the pain, triptans would be the next best option to consider.

Some studies have shown that aspirin may be more effective than some NSAIDs such as ibuprofen, but it should not be used in the pediatric group due to the risk of Reye syndrome.

OPTION E : Propranolol is often used to prevent migraine attacks in children but it is not an appropriate option for treatment of an acute attack.

71
Q

A 72-year-old man is being assessed for memory loss. On mini mental status exam (MMSE), he cannot answer two questions because of sight impairment. At the end of the test, his score is 23. Which one of the following would be the most appropriate next step in management?

A. Start him in donezepil.
B. Perform another cognitive assessment.
C. CT scan of the head.
D. MRI of the head.
E. Reassure.

A

B. Perform another cognitive assessment

MMSE is a practical screening test to evaluate cognitive impairment in older adults. A score of 24 or above rules out cognitive impairment.

Sight impairment has an effect in the results of MMSE (or some similar tools) and may lead to falsely decreased scores. Sight impairment not only can affect completion of tasks requiring vision such as copying a pentagon, but it also has been shown to be associated with impairment in performing tasks that do not require vision.

In those with sight impairment, it is suggested that the test be repeated after the sight is corrected, or other tests in which the vision is not essential for reliable results are used. With an intact sight, the MMSE score of this patient would increase to at least 25; a score that excludes cognitive impairment and overrules the previous score of 23 that is indicative of mild impairment; therefore, this patient can be safely reassured as to not having cognitive impairment.

72
Q

A 67-year-old woman presents to your practice with complaint of headaches for the past 2 weeks. She describes that the headache is nagging in nature and is present on waking up, worse in the morning and associated with vomiting but no nausea. She has taken some over-the-counter painkillers for relief to no avail. Which one of the following could be the most likely diagnosis?

A. Migraine headache.
B. Cluster headache.
C. Sinusitis.
D. A brain tumor.
E. Tension-type headache.

A

D. A brain tumor

Common things are more common. Generally, in a patient presenting with headache, benign causes such as tension-type or migraine headache are more likely diagnoses. However, the presence or absence of some features excludes some of these common and benign pathologies and suggest more serious and less common diagnoses as more likely. In this patient nagging headaches in the morning and the associated vomiting not preceded by nausea (projectile vomiting) are highly suggestive of a space occupying lesion in the brain such as a tumor or an abscess.

Of the options, a brain tumor is most likely to have given rise to such presentation. Other physical exam findings if present can include focal neurological deficits, papilledema, or personality changes.

OPTION A : Migraine headache is throbbing in nature, felt unilateral, and may occur in the morning as well. Nausea and sometimes vomiting are often present. Migraine headache and headache caused by space occupying brain lesions tend to aggravate by head movement. An untreated migraine headache can last between 4-72 hours. In this patient however, the nagging rather than throbbing nature of the pain, absence of previous migraine headache, and also the fact that there is no nausea makes migraine less likely.

OPTION B : Cluster headaches present completely different. Cluster headache is usually unilateral and tend to occur at night or early in the morning waking the patient from sleep. The pain attack is usually a deep, extremely intensive pain around or behind the eye or in the temple. Tearing, runny nose, eye congestion, and ptosis of affected side. The patient is agitated and restless with a tendency to pace around and/or push the painful area with his/her hand. Attacks occur about the same time on each and there may be up to 8 attacks a day. Each attack lasts between 15 and 180 minutes. The nagging nature of the pain in this patient, and that aches do not occur in episodic clusters make such diagnosis less likely.

OPTION C : Headaches caused by sinusitis are nagging and associated with a sense of pressure. There might be tenderness over maxillary or frontal sinuses. Other features are nasal discharge and cough due to postnasal discharge. However, sinusitis headache is not associate with nausea and vomiting.

OPTION E : Tension-type headaches is the most common cause of headache. Such headaches though often occur in the evening hours and are not associated with nausea and vomiting. Tension headache responds well to rest and analgesia with simple painkillers such as paracetamol or NSAIDs.

  • RACGP – AFP – Management of chronic headache
  • RACGP – Theme – Diagnosing headache
  • RACGP – Clinical guidance for MRI referral: Unexplained chronic headache
73
Q

You are assessing Tim, a 65-year-old man, who is brough to the Emergency Department after his wife suspected there may be something wrong with him. She noticed that he had been shaving only the right side of his face for the past 2 days. Assessment suggests ischemic stroke and you are performing a neurological examination on him. In the physical examination, when you ask him to lift his left arm, he raises his right arm instead. Which one of the following areas is most likely to have been affected by the stroke?

A. Right frontal cortex.
B. Left temporal cortex.
C. Right parietal cortex.
D. Left parietal cortex.
E. Right occipital cortex.

A

C. Right parietal cortex

The scenario represents a typical case of hemineglect, also known as unilateral spatial neglect, or unilateral inattention. Hemineglect is a disabling condition that classically presents as non-dominant (often left-sided) spatial disorientation after a pathological event in the right cerebral hemisphere, classically the right posterior parietal cortex. Patients with syndrome can entirely fail to attend to the side opposite (contralateral) to lesion. The patient often collides into their surroundings, ignores food on one side of the plate, and attends to only one side their bodies.

This patient is neglecting the left side evident by not shaving the left side of his face (as if it does not exist) and lifting the right arm instead of the left arm because he is unaware of his left arm. Right parietal lobe pathology is the most likely explanation. Ischemic injuries to the parietal lobe are often caused by stroke in the territory of middle cerebral artery.

NOTE - Spatial neglect not only can cause sensory and motor impairment, but can also affect other components like perceptual, representational, visuospatial, behavioral deficits, etc.

OPTION A : Frontal cortex lesions may include disinhibited speech and behavior, primitive reflexes (e.g., grasping, sucking reflexes), altered mental status, impaired judgment, contralateral weakness (greater in legs than arms), contralateral cortical sensory deficits, and gait apraxia.

OPTION B : Temporal cortex lesions may cause different symptoms which are less noticeable by others. These symptoms include problems with language and speech, forgetfulness, and visual disturbances.

OPTION D : Left parietal lobe involvement results in right side hemineglect. This patient has left side hemineglect; therefore, the lesion is in the right parietal lobe.

OPTION E : Occipital lobe lesions usually present with contralateral homonymous hemianopsia, cortical blindness, visual agnosia (not recognizing familiar objects), altered mental status, and impaired memory.

74
Q

A

Ten days ago, Jenna, 65-year-old, received Covid19 vaccine. Now she has presented with complaints of persistent headaches, anorexia, and nausea. The headache has been resistant to painkillers and progressive in nature. He has no other symptoms and has been otherwise in good health. She only takes thyroxine for hypothyroidism.
On examination, she is systemically well and has a blood pressure (BP) of 130/75 mmHg while lying down, a regular pulse rate of 78 beats per minute, a respiratory rate of 16 breaths per minute, and oxygen saturation of 98% on room air. Ear, nose, and throat examination is unremarkable as is the cardiovascular exam. A full neurological exam is performed with no concerning findings. She has a Glasgow coma scale (GCS) of 15. Which one of the following is the most important investigation to perform?

A. 24-hour BP monitoring.
B. CT scan of the head.
C. D-Dimers, fibrinogen, and platelet levels.
D. Urgent retinal scans.
E. Radionucleotide imaging of the thyroid.

A

C. D-Dimers, fibrinogen, and platelet levels

Persistent headaches in recipients of adenovirus-based vaccines such as Covid19 vaccines should always alarm cerebral venous thrombosis (CVST) until proven otherwise.

CVST is a recognized complication of the ChAdOx1 nCoV-19 vaccine that should elicit a high index of clinical suspicion when patients of any age present with persistent headaches 4–42 days post-administration of the vaccine.

Headache is the most frequent symptom in CVST, and it may occur isolated or accompanied by other symptoms. As in other secondary headache disorders, CVST may be recognized by the presence of red flags.

CVST and other thrombotic complications in adenovirus-based vaccine recipients share a unique feature, delayed presentation. In most of the reported cases, symptoms started one week after immunization.

Although headache is a common symptom after vaccination, it typically presents and resolves within the same day or a few days later.

All patients with symptoms consistent with CVST (persistent, severe, and exacerbated by recumbency and Valsalva) within the relevant time criteria should be assessed initially by blood investigation assessing for (relative) thrombocytopenia and elevated D-dimer (>5 times the upper limit of normal), and subsequently by imaging such as CT scan if the screening tests yield positive results. Consideration for hematological advice should be sought early for possible CVST cases.

It is crucial that all general practitioners are aware of the signs/symptoms of CVST and other thrombotic complications of the ChAdOx1 nCoV-19 vaccine and have a low threshold for consideration of an investigation of thrombosis and thrombocytopenia syndrome (TTS).

OPTION A : 24-hour BP monitoring would be indicated if there were high fluctuations in blood pressure, which is not the case here.

OPTION D : Retinal scans are indicated in cases of acute visual loss which is not a complaint in this scenario.

Option E: Radionucleotide imaging would be considered in case of some thyroid disorders.

75
Q

A 71-year-old man presents to the Emergency Department with weakness of the left arm and leg. His past medical history is significant for coronary angioplasty 5 years ago for which he is currently on daily low-dose aspirin. A non-contrast brain CT scan excludes cerebral hemorrhage. Doppler ultrasonography shows bilateral carotid artery stenosis of 50%. Which one of the following would be the most appropriate action to takef or long-term secondary prevention?

A. Add warfarin.
B. Add clopidogrel.
C. Carotid endarterectomy.
D. Continue the same dose of aspirin.
E. Increase the dose of aspirin.

A

C. Carotid endarterectomy

It is recommended that asymptomatic patients with carotid artery stenosis of equal to or greater than 60% and symptomatic patients with stenosis of equal to or greater than 50% undergo endarterectomy in an attempt to prevent further strokes as the most appropriate management. With a cerebrovascular event and a stenosis of 50% this patent requires carotid endarterectomy as the most appropriate action to prevent further strokes.

OPTION A : Warfarin is the preferred anticoagulant to prevent stroke in the setting of atrial fibrillation.

OPTION B : Although short-term combination theray with aspirin and clopidogrel for 3 weeks is recommended for initial pharmacological treatment of patients with stroke or high-risk TIA, long-term use is discouraged and should be avoided unless they are indicated for acute coronary syndrome or coronary artery stents. If an individual has a stroke while on aspirin, dipyridamole should be added to aspirin or aspirin be switched to clopidogrel.

OPTION D : With the stenotic carotid artery, continuing the same dose of aspirin is necessary but will not prevent further cerebrovascular events.

OPTION E : Increasing the dose of aspirin has not proven effective in prevention of further strokes/TIAs compared to lower doses.

76
Q

A 4-year-old black Sudanese boy from a refugee camp presents with irritability and lethargy. Laboratory assessment is significant for a decreased 25-hydroxy vitamin D level, hemoglobin of 48 g/L, platelet count of 170x106/L, and white cell count of 8x106/L. Which one of the following can be the cause of his clinical picture of irritability and lethargy?

A. Acute lymphocytic leukemia (ALL).
B. Acute myelocytic leukemia (AML).
C. Low level of 25-hydroxy vitamin D
D. Low hemoglobin.
E. Infection.

A

C. Low level of 25-hydroxy vitamin D

Vitamin D deficiency is a problem seen in people of low socioeconomic status, particularly those living in camps such as refugees, those living near Poles where there is not enough sun exposure, and dark-skinned individuals.

Moreover, according to an article recently published in the Medical Journal of Australia (MJA), vitamin D deficiency is prevalent among adult Aboriginal Australians, particularly in autumn, spring and winter. The article, however, is not clear about the prevalence of vitamin D deficiency among Aboriginal children.

Vitamin D insufficiency leads to decreased absorption of dietary calcium and hypocalcemia. The earliest effects of hypocalcemia on CNS include irritability and lethargy. Severe cases may develop into seizures.

OPTION A and B : With acute leukemia, fatigue would be the most common presenting symptom. Moreover, this patient has normal WBC and platelet counts, making leukemia a less likely diagnosis.

OPTION D : Anemia commonly presents with fatigue and pallor rather than irritability and lethargy.

OPTION E : In the absence of fever and a normal WBC count, infection is very unlikely.

77
Q

Jane, 53 years, presents to your GP practice with facial weakness, right ptosis, and drooling from the right corner of her mouth since this morning. She mentions a history of right ear pain a few days earlier. On examination, an erythematous vesicular rash is noted in the right ear canal. Which one of the following would be the most appropriate next step in management?

A. Oral aciclovir.
B. Oral prednisone.
C. Methotrexate.
D. Clindamycin.
E. Gabapentin.

A

A. Oral aciclovir.

The clinical findings and history suggest Ramsay Hunt syndrome as the most likely diagnosis. The syndrome is defined as an acute peripheral facial neuropathy caused by the reactivation of a latent varicella-zoster virus infection. The syndrome is associated with an erythematous vesicular rash of the skin of the ear canal, auricle (also known as herpes zoster oticus), and/or oropharyngeal mucous membrane.

Varicella zoster virus (VZV) infection causes two distinct clinical syndromes. Primary infection, also known as varicella or chickenpox, is a common pediatric viral disease. After a chickenpox infection, VZV remains latent in the neurons of cranial nerves and dorsal root ganglia. Subsequent reactivation of latent VZV can result in a vesicular rash in a dermatomal distribution, known as herpes zoster. VZV infection or reactivation involving the geniculate ganglion of the 7th cranial nerve (CN VII) within the temporal bone is the main pathophysiological mechanism of Ramsay-Hunt syndrome.

Ramsay-Hunt syndrome is estimated to account for 16% of all causes of unilateral facial palsies in children, and 18% of facial palsies in adults. Ramsay Hunt syndrome is thought to be the cause of as many as 20% of clinically diagnosed cases of Bell palsy. Although VZV reactivation is the cause in most cases of this syndrome, herpes simplex virus (HSV) II has been implicated in some cases.

The syndrome usually presents with paroxysmal pain deep within the ear. The pain often radiates outward into the pinna of the ear and may be associated with a more constant, diffuse, and dull background pain. The onset of pain usually precedes the rash by several hours or even days.

The classic triad includes (1) ipsilateral facial palsy, (2) ear pain, and (3) vesicles in the auditory canal and auricle. In some patients taste perception, hearing (tinnitus, hyperacusis), and lacrimation may occur.

Ramsay-Hunt syndrome is generally considered a polycranial neuropathy, with frequent involvement of cranial nerves V, IX, and X. With the involvement of the geniculate ganglion, the inflammation can spread to the 8th cranial nerve (CN VIII), resulting in auditory and vestibular disorders.

Clinical manifestations of Ramsay Hunt syndrome may include:

  • Vesicular rash of the ear or mouth (as many as 80% of cases)
  • The rash might precede the onset of facial paresis/palsy (involvement of the seventh cranial nerve [CN VII])
  • Ipsilateral lower motor neuron facial paresis/palsy (CN VII)
  • Vertigo and ipsilateral hearing loss (CN VIII)
  • Tinnitus
  • Hyperacusis
  • Otalgia (ear pain)
  • Headaches
  • Dysarthria
  • Gait ataxia
  • Fever
  • Cervical lymphadenopathy
  • Facial weakness usually reaches maximum severity by one week after the onset of symptoms.
  • Other cranial neuropathies might be present and may involve cranial nerves (CNs) VIII, IX, X, V, and VI.
  • Ipsilateral hearing loss has been reported in as many as 50% of cases.
  • Blisters of the skin of the ear canal, auricle, or both may become secondarily infected, causing cellulitis

Antiviral treatment should be started in all patients within 72 hours of the onset of the rash. This time limit does not apply to patients with herpes zoster ophthalmicus and immunocompromised patients, meaning that such patients should receive antiviral therapy even beyond 72 hours of presentation.

Treatment options include famciclovir, valaciclovir, and aciclovir.

NOTE - Aciclovir is the preferred medication in children and pregnant women.

For this patient, oral acyclovir is the most appropriate option to consider as the next step in management.

Option A: Therapy with oral corticosteroids (e.g. prednisolone daily for 7 days, tapering to 5 mg daily over the next 2 weeks) can reduce symptoms in the acute (inflammatory) phase of herpes zoster, but only when used in combination with an antiviral agent. Corticosteroids are recommended for patients over the age of 50 if not contraindicated and should be used with caution in patients with comorbid conditions such as diabetes.

Option C: Methotrexate has no role in the treatment of herpes zoster.

Option D: Antibiotics other than mentioned antiviral agents are ineffective in treating herpes zoster.

Option E: Gabapentin and tricyclic antidepressants are the most effective treatment options for neuropathic pain associated with herpes zoster infection, but given the time of presentation, antiviral therapy should be considered as the cornerstone of therapy first.

78
Q

A 70-year-old man is being evaluated for further management after he recovered from an episode of transient ischemic attack (TIA). On examination, his blood pressure is 180/110 mmHg and a bruit is heard over the right carotid artery. A Duplex Doppler ultrasonography of carotid arteries is performed, which is significant for 50% and 60% stenosis of the right and left carotid arteries, respectively. Which one of the following options is the most appropriate management for prevention of further thromboembolic phenomena?

A. Hypertension control.
B. Aspirin.
C. Warfarin.
D. Carotid endarterectomy.
E. Clopidogrel.

A

B. Aspirin

The scenario is about measures for secondary prevention of cerebrovascular accident (CVA) after the patient sustained a TIA. Several preventive measures must be considered for him.

By far, hypertension is the most important preventable risk factor for CVA, accounting for 50% of cases. Unless contraindicated by symptomatic hypotension, all patients should receive blood pressure lowering therapy after stroke or TIA regardless of being hypertensive or normotensive. With the exception of beta blockers, all antihypertensive medications have proven effective; however, ACE inhibitors and diuretics, either in combination or alone, have shown to be more effective.

This patient should also receive antiplatelet therapy. Antiplatelet therapy significantly reduces further incidences of stroke, MI, or vascular death.

Long-term antiplatelet medications should be prescribed to all individuals with ischemic stroke or TIA, who are not already on anticoagulation therapy (e.g., warfarin). The antiplatelet therapy of choice is low-dose aspirin plus modified release dipyridamole. However, aspirin alone is the most used regimen in practice.

Clopidogrel is indicated in patients who are not tolerant of aspirin, those with contraindications to aspirin, or those who experience another CVA despite being on aspirin.

This patient has also a 50% stenosis in the right and 60% stenosis in the left carotid artery. Current guidelines recommend that carotid endarterectomy (CEA) should be performed in patients with non-disabling carotid artery territory ischemic stroke or TIA with ipsilateral carotid stenosis measured at 70-99% if surgery can be performed by a specialist surgeon with low (< 6%) perioperative mortality/morbidity. For stenoses of 50-69%, CEA is performed in selected patients (considering age, gender, and comorbidities) with non-disabling carotid artery territory ischemic stroke or TIA if surgery can be performed with very low (< 3%) perioperative mortality/morbidity.

Symptomatic stenosis means TIA or stroke attributable to the territory of the stenotic artery. For example, if a patient has stenosis of both left and right carotid arteries, and develops right hemiparesis and left facial weakness, only the stenosis of left carotid artery is symptomatic.

Other measures to consider for this patient are regular exercise, statins and glycemic control if diabetic.

Of the given options, however, the most appropriate one for prevention of a thromboembolic event is aspirin.

Option A: Lowering the blood pressure has a significant effect on prevention of CVAs in the future, and reduces the risk by approximately 25%, but it is unlikely to prevent thromboembolism; not directly at least.

Option C: Warfarin is the standard of care in patients with CVA in the setting of atrial fibrillation and is preferred over aspirin or other antiplatelet agents. Unless this patient has AF, warfarin is not the preferred option for him.

Option D: Although CEA in patients with symptomatic carotid stenosis of ≥70% is associated with decreased risk of thromboembolism, benefits to those with stenosis at 50-69% seems marginal, when compared to aspirin.

Option E: Clopidogrel is the antiplatelet medication of choice if the patient has co-existing ischemic heart disease or has recently undergone coronary artery stenting. It is also replaced with aspirin is the patient is intolerant of it, or develops another ischemic event while on it.

Generaly do CEA if > 70-80% stenosed! Case by case.

79
Q

A 65-year-old man presents to the Emergency Department with left-sided hemiplegia starting 30 minutes ago. On arrival, his blood pressure is 140/95 mmHg and pulse rate 82 bpm and regular. A non-contrast CT scan of the head excludes intracranial hemorrhage. With a diagnosis of ischemic cerebrovascular accident (CVA), which one of the following could have the most significant effect on recovery of this patient?

A. Immediate lowering of his blood pressure.
B. Carotid endarterectomy.
C. Fibrinolytic therapy.
D. Aspirin.
E. Expert nurse care.

A

C. Fibrinolytic therapy

If not contraindicated, fibrinolytic (thrombolytic) therapy is the treatment of choice in carefully selected patients, who present within 4.5 hours after symptoms onset. Several trials have shown significant reduction in death and disability. Recombinant tissue plasminogen activator (rTPA) is approved for this purpose in Australia. Despite the benefits, access to thrombolysis remains low in Australia (~3%).

Option A: Acute lowering of the blood pressure is harmful for patients with stroke due to risk of cerebral hypoperfusion. The only indication is extremely high blood pressures (systolic> 22mmHg, diastolic>11 mmHg).

Option B: Carotid endarterectomy (CEA) is considered for secondary prevention of ischemic stroke and TIA in selected patients. It prevents further ischemic events but does not affect the outcomes of the patient’s current condition.

Option D: Aspirin (150-300mg) is the most common medication used for initial treatment of ischemic stroke (contraindicated in hemorrhagic stroke) if thrombolysis is not considered and should be given as soon as possible (within 48 hours). If thrombolysis is done, aspirin should be delayed for 24 hours.

NOTE - Current guidelines recommend dual antiplatelet therapy with aspirin and clopidogrel for 3 weeks.

Option E: Expert nurse care provides rehabilitation and improvement of the quality of life of patients with stroke in and established debilitation but has nothing to do with acute management of stroke.

80
Q

A 60-year-old man has had three episodes of transient monocular left blindness and right hemianesthesia. On examination, a soft bruit is heard over the left carotid artery. A Duplex Doppler ultrasound reveals a 25% stenosis of the left carotid artery at the level of the carotid bifurcation. Based on the current recommendations, which one of the following would be the best step in management of this patient?

A. Immediate carotid endarterectomy.
B. Carotid endarterectomy in 2 months.
C. Aspirin100mg daily.
D. Warfarin.
E. Intracarotid streptokinase infusion.

A

C. Aspirin100mg daily

The clinical picture is remarkable for episodes of transient ischemic attacks (TIA) in the territory of the left carotid artery. Impaired blood flow to this artery can cause ipsilateral visual problems, facial weakness, and facial sensory deficits, as well as contralateral motor and sensory disturbances below the level of the injury.

Antiplatelet therapy is the pharmacological intervention of choice for secondary prevention of ischemic strokes and TIAs. The most effective treatment option is either low-dose aspirin or combination therapy with aspirin and dipyridamole. Although the latter appears more effective, aspirin alone is the most commonly used regimen due to adverse effects of dipyridamole (e.g., headache, flushing, nausea, gastrointestinal upset).

Option A and B: Carotid endarterectomy (CEA) has shown no benefit in patients with symptomatic carotid stenoses of less than 50%. It is not indicated in this patient with a stenosis of 25%.

Option D: Warfarin in used for prevention of ischemic stroke or TIA in the presence of atrial fibrillation (AF).

Option E: Intra-carotid streptokinase infusion is not a preventive measure for ischemic stroke or TIA. It is used for treatment of stroke within 4.5 hours of symptoms onset.

81
Q

A 71-year-old woman presents to the Emergency Department with dysarthria and right sided hemiplegia which started one hour ago. She had a myocardial infarction 18 moths back and is on aspirin 80 mg, daily. A CT scan of the head is obtained which is negative for intracranial hemorrhage. Which one of the following is the most appropriate next step in management?

A. rTPA.
B. Clopidogrel and aspirin.
C. Clopidogrel.
D. Warfarin.
E. Heparin.

A

A. rTPA

The constellation of signs and symptoms in the absence of blood on brain CT scan suggests ischemic stroke or transient ischemic attack (TIA) as the diagnosis.

In carefully selected patients with ischemic stroke or TIA, thrombolytic therapy is the treatment of choice if the patient presents within 4.5 hours of symptoms onset. Recombinant tissue plasminogen activator (rTPA) is approved for this purpose in Australia.

Current guidelines also recommend dual therapy with aspirin and clopidogrel for ONLY 3 weeks. This treatment should be started immediately. If thrombolytic therapy has been performed, dual therapy should be started 24 hours after it. For this patient, thrombolytic therapy is the first step followed by combination therapy after 24 hours.

Option C: Clopidogrel alone is used as stand-alone therapy in patients who are intolerant of or allergic to aspirin or develop another cardiovascular event while on aspirin. Addition of dipyridamole to aspirin is another option for the latter.

Option D and E: Anticoagulation is the secondary preventive method of choice in patients with ischemic stroke and TIA in the setting of AF. Heparin is not recommended for acute management of stroke or TIA if the cause is not AF.

82
Q

A 67-year-old woman presents to the Emergency Department after she developed a transient left hemiparesis at home 20 minutes ago which has resolved completely at the time of physical examination. She had an acute myocardial infarction 12 months ago and underwent angioplasty. She is currently on aspirin 100 mg, daily. Which one of the following is the most appropriate management option for her?

A. Add warfarin.
B. Add clopidogrel.
C. Continue the same dose of aspirin.
D. Increase the dose of aspirin.
E. Switch to dipyridamole.

A

B. Add clopidogrel

Antiplatelet therapy is one of the most important measures taken for secondary prevention of ischemic cerebrovascular accidents in patients with the history of such events. Although addition of dipyridamole to aspirin has shown increased benefit compared to aspirin alone, aspirin alone is more used due to the adverse effects of dipyridamole and the consequent patients’ non-compliance.

Current guidelines recommend dual therapy with aspirin and clopidogrel for 3 weeks for initial pharmacological management of strokes and high-risk transient ischemic attacks (TIA). This patient is already on aspirin; therefore, addition of clopidogrel is the most appropriate next step. Unless also indicated for acute coronary disease or coronary artery stents, extended combination therapy with aspirin and clopidogrel beyond 3 weeks is not recommended.

Option A: Warfarin is the antithrombotic drug of choice in the presence of atrial fibrillation (AF) as the likely underlying cause of the cerebrovascular event.

Option C and D: Continuation of aspirin at the same dose does not prevent more episodes of CVA.

Option E: Dipyridamole has no considerable benefit over aspirin. However, adding it to aspirin can be used for long-term secondary prevention of cerebrovascular events, Dipyridamole adverse effects include headache, flushing, nausea and gastrointestinal upset. Such adverse effects are a major cause of non-compliance and abandoning the treatment by patients. Dipyridamole does not cause bleeding, nor does it increase the incidence of aspirin-related bleeding.

dipyridamole side effects: headache, flushing, nausea, GI upset

83
Q

A 72-year-old woman, who is a known case of diabetes and hypertension and on multiple medications, is brought to the Emergency Department with complaint of left-sided weakness for the past 48 hours. On examination, she has a blood pressure of 150/100 mmHg and an irregular pulse of 98 bpm. Her blood sugar is 8 mmol/L. Three years ago, she had an episode of sudden-onset right vision loss for few hours before she completely recovers. She, however, did not seek any medical attention at that time. Which one of the following is the most appropriate treatment option for her?

A. Warfarin.
B. Aspirin.
C. Dipyridamole.
D. rTPA.
E. Aspirin and clopidogrel.

A

A. Warfarin

If not contraindicated, antithrombotic therapy should be started for all patients with ischemic stroke. In patients with atrial fibrillation (AF), warfarin is the antithrombotic of choice to start. The need for anticoagulation is assessed based on CHA2DS2- VASc system.

Option B and C: Aspirin or dipyridamole alone does not appear to provide adequate anticoagulation in patients with AF.

Option D: Thrombolytic therapy is indicated within the first 4.5 hours of symptoms onset. In this patient, 48 hours has passed since the start of symptom; therefore, thrombolytic therapy with rTPA is not beneficial for her.

Option E: Combination therapy with aspirin and clopidogrel for 3 weeks is recommended for patients with stroke or high-risk transient ischemic attacks in other settings than AF.

This patient has irregular pulse of 98bpm! read properly

84
Q

A 60-year-old woman presents to your clinic for evaluation. She has the history of a recent transient ischemic attack (TIA), and was diagnosed with atrial fibrillation (AF) 3 months ago. Other significant points in her history are cigarette smoking and hypercholesterolemia. A Duplex Doppler sonography of the carotid arteries has been performed which shows 40% stenosis of the left and 50% stenosis of the right carotid arteries. On examination, she is found to have a blood pressure of 180/110 mmHg on two occasions. Which one of the following would be the most appropriate long-term management?

A. Left carotid endarterectomy.
B. Smoking cessation.
C. Hypertension control.
D. Warfarin.
E. Aspirin.

A

D. Warfarin

By far, hypertension is the most important preventable risk factor for CVA, accounting for 50% of cases. Unless contraindicated by symptomatic hypotension, all patients should receive blood pressure lowering therapy after stroke or TIA, regardless of their blood pressure (option C). With the exception of beta blockers, all antihypertensive medications have proven effective; however, ACE inhibitors and diuretics, either in combination or alone have shown to be more effective.

This patient should also receive antiplatelet therapy. Antiplatelet therapy significantly reduces stroke, MI or vascular death. Long term antiplatelet therapy should be prescribed to all patients with ischemic stroke or TIA. The antiplatelet therapy of choice is low dose aspirin plus modified release dipyridamole.

In this patient, however, in the presence of AF, aspirin (option E) will not provide adequate protection against thromboembolism, and warfarin should be used for such purpose. Warfarin is the standard of care in patients with CVA in the setting of AF, and is preferred over aspirin or other antiplatelet agents.

This patient has also a 50% stenosis in the right and 40% stenosis in the left carotid arteries. Current guidelines recommend that carotid endarterectomy (CEA) (option A) should be performed in patients with non-disabling carotid artery territory ischemic stroke or TIA with ipsilateral carotid stenosis measured at 70-99% if surgery can be performed by a specialist surgeon with < 5% perioperative mortality/morbidity, or stenoses of 50-69% in selected patients (considering age, gender and comorbidities) with < 3% risk of perioperative mortality/morbidity. The question is not specific about which side was involved in the previous TIA, but if occurred on the right side, CEA would be considered. For symptomatic stenoses of less than 50%, or asymptomatic stenoses of less than 60%, CEA is not recommended.

Option B: Smoking cessation is another important step that reduces the chance of having another incidence of ischemic stroke or TIA and all-cause mortality/morbidity. It is not, however, as important as warfarin therapy at this stage.

This patient should also be started on statins regardless of her cholesterol level.